Test 5 Med surg

¡Supera tus tareas y exámenes ahora con Quizwiz!

An adult patient has sought care for the treatment of headaches that have become increasingly severe and frequent over the past several months. Which of the following questions addresses potential etiological factors? Select all that apply?

-Are you exposed to any toxins or chemicals at work? -How would you describe your ability to cope with stress? -What medications are you currently taking? -Does anyone else in your family struggle with headaches?

The nurse caring for a patient in a persistent vegetative state is regularly assessing for potential complications. Complications of neurologic dysfunction for which the nurse should assess include which of the following? Select all that apply.

-Contractures -Pressure ulcers -Venous thromboembolism -Pneumonia

During a patients recovery from stroke, the nurse should be aware of predictors of stroke outcome in order to help patients and families set realistic goals. What are the predictors of stroke outcome? Select all that apply.

-National Institutes of Health Stroke Scale (NIHSS) score -LOC at time of admission -Age

As a member of the stroke team, the nurse knows that thrombolytic therapy carries the potential for benefit and for harm. The nurse should be cognizant of what contraindications for thrombolytic therapy? Select all that apply.

-Recent intracranial pathology -Current anticoagulation therapy -Symptom onset greater than 3 hours prior to admission

The nurse is admitting a patient to the unit who is scheduled for removal of an intracranial mass. What diagnostic procedures might be included in this patients admission orders? Select all that apply.

-Transcranial Doppler flow study -Cerebral angiography -MRI

Assessment is crucial to the care of patients with neurologic dysfunction. What does accurate and appropriate assessment require? Select all that apply.

-Understanding of the tests used to diagnose neurologic disorders -Knowledge of nursing interventions related to assessment and diagnostic testing -Knowledge of the anatomy of the nervous system

The nurse educator is reviewing the assessment of cranial nerves. What should the educator identify as the specific instances when cranial nerves should be assessed? Select all that apply.

-When level of consciousness is decreased -With brain stem pathology -In the presence of peripheral nervous system disease

The public health nurse is planning a health promotion campaign that reflects current epidemiologic trends. The nurse should know that hemorrhagic stroke currently accounts for what percentage of total strokes in the United States?

13%

The pathophysiology of an ischemic stroke involves the ischemic cascade, which includes the following steps: 1. Change in pH 2. Blood flow decreases 3. A switch to anaerobic respiration 4. Membrane pumps fail 5. Cells cease to function 6. Lactic acid is generated Put these steps in order in which they occur.

236145

A neurologic nurse is reviewing seizures with a group of staff nurses. How should this nurse best describe the cause of a seizure?

A dysrhythmia in the nerve cells in one section of the brain

The nurse is preparing to provide care for a patient diagnosed with myasthenia gravis. The nurse should know that the signs and symptoms of the disease are the result of what?

A lower motor neuron lesion

An elderly patient is being discharged home. The patient lives alone and has atrophy of his olfactory organs. The nurse tells the patients family that it is essential that the patient have what installed in the home?

A smoke detector

A case manager is responsible for ensuring that patients meet the criteria for diagnoses of chronic conditions in order to ensure their eligibility for federal programs. Which of these definitions may not apply for legal purposes? A) A person who is temporarily disabled but later return to full functioning. B) A person who is disabled and cannot expect a return to full functioning. C) A person whose disability is the result of a developmental disorder. D) A person whose disability is the result of a traumatic injury.

A) A person who is temporarily disabled but later return to full functioning.

A man with a physical disability uses a wheelchair. The individual wants to attend a support group for the parents of autistic children, which is being held in the basement of a church. When the individual arrives at the church, he realizes there are no ramps or elevators to the basement so he will not be able to attend the support group. What type of barrier did this patient encounter? A) A structural barrier B) A barrier to health care C) An institutional barrier D) A transportation barrier

A) A structural barrier

A patient who is legally blind is being admitted to the hospital. The patient informs the nurse that she needs to have her guide dog present during her hospitalization. What is the nurse's best response to the patient? A) Arrangements can be made for your guide dog to be at the hospital with you during your stay. B) I will need to check with the care team before that decision can be made. C) Because of infection control, your guide dog will likely not be allowed to stay in your room during your hospitalization. D) Your guide dog can stay with you during your hospitalization, but he will need to stay in a cage or crate that you will need to provide.

A) Arrangements can be made for your guide dog to be at the hospital with you during your stay.

The community nurse is caring for a patient who has paraplegia following a farm accident when he was an adolescent. This patient is now 64 years old and has just been diagnosed with congestive heart failure. The patient states, Im so afraid about what is going to happen to me. What would be the best nursing intervention for this patient? A) Assist the patient in making suitable plans for his care. B) Take him to visit appropriate long-term care facilities. C) Give him pamphlets about available community resources. D) Have him visit with other patients who have congestive heart failure.

A) Assist the patient in making suitable plans for his care.

A patient tells the nurse that her doctor told her that her new diagnosis of rheumatoid arthritis is considered to be a chronic condition. She asks the nurse what chronic condition means. What would be the nurse's best response? A) Chronic conditions are defined as health problems that require management of several months or longer. B) Chronic conditions are diseases that come and go in a relatively predictable cycle. C) Chronic conditions are medical conditions that culminate in disabilities that require hospitalization. D) Chronic conditions are those that require short-term management in extended-care facilities.

A) Chronic conditions are defined as health problems that require management of several months or longer.

In your role as a school nurse, you are presenting at a high school health fair and are promoting the benefits of maintaining a healthy body weight. You should refer to reductions in the risks of what diseases? Select all that apply. A) Heart disease B) Stroke C) Cancer D) Diabetes E) Hypertension

A) Heart Disease B) Stroke D) Diabetes E) Hypertension

An international nurse has noted that a trend in developing countries is a decrease in mortality from some acute conditions. This has corresponded with an increase in the incidence and prevalence of chronic diseases. What has contributed to this decrease in mortality from some acute conditions? A) Improved nutrition B) Integration of alternative health practices C) Stronger international security measures D) Decrease in obesity

A) Improved Nutrition

A home care nurse is making an initial visit to a 68-year-old man. The nurse finds the man tearful and emotionally withdrawn. Even though the man lives alone and has no family, he has been managing well at home until now. What would be the most appropriate action for the nurse to take? A) Reassess the patients psychosocial status and make the necessary referrals B) Have the patient volunteer in the community for social contact C) Arrange for the patient to be reassessed by his social worker D) Encourage the patient to focus on the positive aspects of his life

A) Reassess the patient's psychosocial status and make the necessary referrals

You are caring for a patient with a history of chronic angina. The patient tells you that after breakfast he usually takes a shower and shaves. It is at this time, the patient says that he tends to experience chest pain. What might you counsel the patient to do to decrease the likelihood of angina in the morning? A) Shower in the evening and shave before breakfast. B) Skip breakfast and eat an early lunch. C) Take a nitro tab prior to breakfast. D) Shower once a week and shave prior to breakfast.

A) Shower in the evening and shave before breakfast.

A patient with a spinal cord injury is being assessed by the nurse prior to his discharge home from the rehabilitation facility. The nurse is planning care through the lens of the interface model of disability. Within this model the nurse will plan care based on what belief? A) The patient has the potential to function effectively despite his disability. B) The patients disabling condition does not have to affect his lifestyle. C) The patient will not require care from professional caregivers in the home setting. D) The patients disability is the most salient aspect of his personal identity.

A) The patient has the potential to function effectively despite his disability.

An elderly patient presented to the clinic with a new diagnosis of osteoarthritis. The patients daughter is accompanying him and you have explained why the incidence of chronic diseases tends to increase with age. What rationale for this phenomenon should you describe? A) With age, biologic changes reduce the efficiency of body systems. B) Older adults often have less support and care from their family, resulting in illness. C) There is an increased morbidity of peers in this age group, and this leads to the older adults desire to also assume the sick role. D) Chronic illnesses are diagnosed more often in older adults because they have more contact with the health care system.

A) With age, biological changes reduce the efficiency of body systems.

A school nurse is called to the playground where a 6-year-old girl has been found unresponsive and staring into space, according to the playground supervisor. How would the nurse document the girls activity in her chart at school?

Absence seizure

A patient with a cerebral aneurysm exhibits signs and symptoms of an increase in intracranial pressure (ICP). What nursing intervention would be most appropriate for this patient?

Absolute bed rest in a quiet, nonstimulating environment

The nurse is caring for a patient with permanent neurologic impairments resulting from a traumatic head injury. When working with this patient and family, what mutual goal should be prioritized?

Achieve as high a level of function as possible.

The nurse is caring for a patient who exhibits abnormal results of the Weber test and Rinne test. The nurse should suspect dysfunction involving what cranial nerve?

Acoustic

A patient with herpes simplex virus encephalitis (HSV) has been admitted to the ICU. What medication would the nurse expect the physician to order for the treatment of this disease process?

Acyclovir (Zovirax)

A patient is recovering from intracranial surgery performed approximately 24 hours ago and is complaining of a headache that the patient rates at 8 on a 10-point pain scale. What nursing action is most appropriate?

Administer morphine sulfate as ordered.

A patient who has been on long-term phenytoin (Dilantin) therapy is admitted to the unit. In light of the adverse of effects of this medication, the nurse should prioritize which of the following in the patients plan of care?

Administration of thorough oral hygiene

A community health nurse is giving an educational presentation about stroke and heart disease at the local senior citizens center. What nonmodifiable risk factor for stroke should the nurse cite?

Advanced age

The nurse caring for an 80 year-old patient knows that she has a pre-existing history of dulled tactile sensation. The nurse should first consider what possible cause for this patients diminished tactile sensation?

Age-related neurologic changes

A clinic nurse is caring for a patient diagnosed with migraine headaches. During the patient teaching session, the patient questions the nurse regarding alcohol consumption. What would the nurse be correct in telling the patient about the effects of alcohol?

Alcohol causes vasodilation of the blood vessels.

When caring for a patient who had a hemorrhagic stroke, close monitoring of vital signs and neurologic changes is imperative. What is the earliest sign of deterioration in a patient with a hemorrhagic stroke of which the nurse should be aware?

Alteration in level of consciousness (LOC)

26. The nurse is planning the care of a patient who has been recently diagnosed with a cerebellar tumor. Due to the location of this patient's tumor, the nurse should implement measures to prevent what complication? A) Falls B) Audio hallucinations C) Respiratory depression D) Labile BP

Ans: A Feedback: A cerebellar tumor causes dizziness, an ataxic or staggering gait with a tendency to fall toward the side of the lesion, and marked muscle incoordination. Because of this, the patient faces a high risk of falls. Hallucinations and unstable vital signs are not closely associated with cerebellar tumors.

A patient with a C5 spinal cord injury is tetraplegic. After being moved out of the ICU, the patient complains of a severe throbbing headache. What should the nurse do first? A) Check the patient's indwelling urinary catheter for kinks to ensure patency. B) Lower the HOB to improve perfusion. C) Administer analgesia. D) Reassure the patient that headaches are expected after spinal cord injuries.

Ans: A Feedback: A severe throbbing headache is a common symptom of autonomic dysreflexia, which occurs after injuries to the spinal cord above T6. The syndrome is usually brought on by sympathetic stimulation, such as bowel and bladder distention. Lowering the HOB can increase ICP. Before administering analgesia, the nurse should check the patient's catheter, record vital signs, and perform an abdominal assessment. A severe throbbing headache is a dangerous symptom in this patient and is not expected.

A nurse is assessing a patient with an acoustic neuroma who has been recently admitted to an oncology unit. What symptoms is the nurse likely to find during the initial assessment? A) Loss of hearing, tinnitus, and vertigo B) Loss of vision, change in mental status, and hyperthermia C) Loss of hearing, increased sodium retention, and hypertension D) Loss of vision, headache, and tachycardia

Ans: A Feedback: An acoustic neuroma is a tumor of the eighth cranial nerve, the cranial nerve most responsible for hearing and balance. The patient with an acoustic neuroma usually experiences loss of hearing, tinnitus, and episodes of vertigo and staggering gait. Acoustic neuromas do not cause loss of vision, increased sodium retention, or tachycardia.

The school nurse has been called to the football field where player is immobile on the field after landing awkwardly on his head during a play. While awaiting an ambulance, what action should the nurse perform? A) Ensure that the player is not moved. B) Obtain the player's vital signs, if possible. C) Perform a rapid assessment of the player's range of motion. D) Assess the player's reflexes.

Ans: A Feedback: At the scene of the injury, the patient must be immobilized on a spinal (back) board, with the head and neck maintained in a neutral position, to prevent an incomplete injury from becoming complete. This is a priority over determining the patient's vital signs. It would be inappropriate to test ROM or reflexes.

The nurse is caring for a patient whose spinal cord injury has caused recent muscle spasticity. What medication should the nurse expect to be ordered to control this? A) Baclofen (Lioresal) B) Dexamethasone (Decadron) C) Mannitol (Osmitrol) D) Phenobarbital (Luminal)

Ans: A Feedback: Baclofen is classified as an antispasmodic agent in the treatment of muscles spasms related to spinal cord injury. Decadron is an anti-inflammatory medication used to decrease inflammation in both SCI and head injury. Mannitol is used to decrease cerebral edema in patients with head injury. Phenobarbital is an anticonvulsant that is used in the treatment of seizure activity.

A patient who is being treated in the hospital for a spinal cord injury is advocating for the removal of his urinary catheter, stating that he wants to try to resume normal elimination. What principle should guide the care team's decision regarding this intervention? A) Urinary retention can have serious consequences in patients with SCIs. B) Urinary function is permanently lost following an SCI. C) Urinary catheters should not remain in place for more than 7 days. D) Overuse of urinary catheters can exacerbate nerve damage.

Ans: A Feedback: Bladder distention, a major cause of autonomic dysreflexia, can also cause trauma. For this reason, removal of a urinary catheter must be considered with caution. Extended use of urinary catheterization is often necessary following SCI. The effect of a spinal cord lesion on urinary function depends on the level of the injury. Catheter use does not cause nerve damage, although it is a major risk factor for UTIs.

A patient is admitted to the neurologic ICU with a suspected diffuse axonal injury. What would be the primary neuroimaging diagnostic tool used on this patient to evaluate the brain structure? A) MRI B) PET scan C) X-ray D) Ultrasound

Ans: A Feedback: CT and MRI scans, the primary neuroimaging diagnostic tools, are useful in evaluating the brain structure. Ultrasound would not show the brain nor would an x-ray. A PET scan shows brain function, not brain structure.

19. A patient with Huntington disease has just been admitted to a long-term care facility. The charge nurse is creating a care plan for this patient. Nutritional management for a patient with Huntington disease should be informed by what principle? A) The patient is likely to have an increased appetite. B) The patient is likely to required enzyme supplements. C) The patient will likely require a clear liquid diet. D) The patient will benefit from a low-protein diet.

Ans: A Feedback: Due to the continuous involuntary movements, patients will have a ravenous appetite. Despite this ravenous appetite, patients usually become emaciated and exhausted. As the disease progresses, patients experience difficulty in swallowing and thin liquids should be avoided. Protein will not be limited with this disease. Enzyme supplements are not normally required.

A patient with a T2 injury is in spinal shock. The nurse will expect to observe what assessment finding? A) Absence of reflexes along with flaccid extremities B) Positive Babinski's reflex along with spastic extremities C) Hyperreflexia along with spastic extremities D) Spasticity of all four extremities

Ans: A Feedback: During the period immediately following a spinal cord injury, spinal shock occurs. In spinal shock, all reflexes are absent and the extremities are flaccid. When spinal shock subsides, the patient demonstrates a positive Babinski's reflex, hyperreflexia, and spasticity of all four extremities

The nurse is planning the care of a patient with a T1 spinal cord injury. The nurse has identified the diagnosis of "risk for impaired skin integrity." How can the nurse best address this risk? A) Change the patient's position frequently. B) Provide a high-protein diet. C) Provide light massage at least daily. D) Teach the patient deep breathing and coughing exercises.

Ans: A Feedback: Frequent position changes are among the best preventative measures against pressure ulcers. A high-protein diet can benefit wound healing, but does not necessarily prevent skin breakdown. Light massage and deep breathing do not protect or restore skin integrity.

23. A gerontologic nurse is advocating for diagnostic testing of an 81-year-old patient who is experiencing personality changes. The nurse is aware of what factor that is known to affect the diagnosis and treatment of brain tumors in older adults? A) The effects of brain tumors are often attributed to the cognitive effects of aging. B) Brain tumors in older adults do not normally produce focal effects. C) Older adults typically have numerous benign brain tumors by the eighth decade of life. D) Brain tumors cannot normally be treated in patient over age 75.

Ans: A Feedback: In older adult patients, early signs and symptoms of intracranial tumors can be easily overlooked or incorrectly attributed to cognitive and neurologic changes associated with normal aging. Brain tumors are not normally benign and they produce focal effects in all patients. Treatment options are not dependent primarily on age.

25. A male patient presents at the free clinic with complaints of impotency. Upon physical examination, the nurse practitioner notes the presence of hypogonadism. What diagnosis should the nurse suspect? A) Prolactinoma B) Angioma C) Glioma D) Adrenocorticotropic hormone (ACTH)-producing adenoma

Ans: A Feedback: Male patients with prolactinomas may present with impotence and hypogonadism. An ACTH-producing adenoma would cause acromegaly. The scenario contains insufficient information to know if the tumor is an angioma, glioma, or neuroma.

30. An older adult has encouraged her husband to visit their primary care provider, stating that she is concerned that he may have Parkinson's disease. Which of the wife's descriptions of her husband's health and function is most suggestive of Parkinson's disease? A) "Lately he seems to move far more slowly than he ever has in the past." B) "He often complains that his joints are terribly stiff when he wakes up in the morning." C) "He's forgotten the names of some people that we've known for years." D) "He's losing weight even though he has a ravenous appetite."

Ans: A Feedback: Parkinson's disease is characterized by bradykinesia. It does not manifest as memory loss, increased appetite, or joint stiffness.

37. A patient with a new diagnosis of amyotrophic lateral sclerosis (ALS) is overwhelmed by his diagnosis and the known complications of the disease. How can the patient best make known his wishes for care as his disease progresses? A) Prepare an advance directive. B) Designate a most responsible physician (MRP) early in the course of the disease. C) Collaborate with representatives from the Amyotrophic Lateral Sclerosis Association. D) Ensure that witnesses are present when he provides instruction.

Ans: A Feedback: Patients with ALS are encouraged to complete an advance directive or "living will" to preserve their autonomy in decision making. None of the other listed actions constitutes a legally binding statement of end-of-life care.

27. A patient has been admitted to the neurologic ICU with a diagnosis of a brain tumor. The patient is scheduled to have a tumor resection/removal in the morning. Which of the following assessment parameters should the nurse include in the initial assessment? A) Gag reflex B) Deep tendon reflexes C) Abdominal girth D) Hearing acuity

Ans: A Feedback: Preoperatively, the gag reflex and ability to swallow are evaluated. In patients with diminished gag response, care includes teaching the patient to direct food and fluids toward the unaffected side, having the patient sit upright to eat, offering a semisoft diet, and having suction readily available. Deep tendon reflexes, abdominal girth, and hearing acuity are less commonly affected by brain tumors and do not affect the risk for aspiration.

40. A nurse is planning discharge education for a patient who underwent a cervical diskectomy. What strategies would the nurse assess that would aid in planning discharge teaching? A) Care of the cervical collar B) Technique for performing neck ROM exercises C) Home assessment of ABGs D) Techniques for restoring nerve function

Ans: A Feedback: Prior to discharge, the nurse should assess the patient's use and care of the cervical collar. Neck ROM exercises would be contraindicated and ABGs cannot be assessed in the home. Nerve function is not compromised by a diskectomy.

The nurse is caring for a patient with increased intracranial pressure (ICP) caused by a traumatic brain injury. Which of the following clinical manifestations would suggest that the patient may be experiencing increased brain compression causing brain stem damage? A) Hyperthermia B) Tachycardia C) Hypertension D) Bradypnea

Ans: A Feedback: Signs of increasing ICP include slowing of the heart rate (bradycardia), increasing systolic BP, and widening pulse pressure. As brain compression increases, respirations become rapid, BP may decrease, and the pulse slows further. A rapid rise in body temperature is regarded as unfavorable. Hyperthermia increases the metabolic demands of the brain and may indicate brain stem damage.

33. The nurse caring for a patient diagnosed with Parkinson's disease has prepared a plan of care that would include what goal? A) Promoting effective communication B) Controlling diarrhea C) Preventing cognitive decline D) Managing choreiform movements

Ans: A Feedback: The goals for the patient may include improving functional mobility, maintaining independence in ADLs, achieving adequate bowel elimination, attaining and maintaining acceptable nutritional status, achieving effective communication, and developing positive coping mechanisms. Constipation is more likely than diarrhea and cognition largely remains intact. Choreiform movements are related to Huntington disease.

The nurse is providing health education to a patient who has a C6 spinal cord injury. The patient asks why autonomic dysreflexia is considered an emergency. What would be the nurse's best answer? A) "The sudden increase in BP can raise the ICP or rupture a cerebral blood vessel." B) "The suddenness of the onset of the syndrome tells us the body is struggling to maintain its normal state." C) "Autonomic dysreflexia causes permanent damage to delicate nerve fibers that are healing." D) "The sudden, severe headache increases muscle tone and can cause further nerve damage."

Ans: A Feedback: The sudden increase in BP may cause a rupture of one or more cerebral blood vessels or lead to increased ICP. Autonomic dysreflexia does not directly cause nerve damage.

A patient with suspected Parkinson's disease is initially being assessed by the nurse. When is the best time to assess for the presence of a tremor? A) When the patient is resting B) When the patient is ambulating C) When the patient is preparing his or her meal tray to eat D) When the patient is participating in occupational therapy

Ans: A Feedback: The tremor is present while the patient is at rest; it increases when the patient is walking, concentrating, or feeling anxious. Resting tremor characteristically disappears with purposeful movement, but is evident when the extremities are motionless. Consequently, the nurse should assess for the presence of a tremor when the patient is not performing deliberate actions.

An 82-year-old man is admitted for observation after a fall. Due to his age, the nurse knows that the patient is at increased risk for what complication of his injury? A) Hematoma B) Skull fracture C) Embolus D) Stroke

Ans: A Feedback: Two major factors place older adults at increased risk for hematomas. First, the dura becomes more adherent to the skull with increasing age. Second, many older adults take aspirin and anticoagulants as part of routine management of chronic conditions. Because of these factors, the patient's risk of a hematoma is likely greater than that of stroke, embolism, or skull fracture.

A patient with spinal cord injury is ready to be discharged home. A family member asks the nurse to review potential complications one more time. What are the potential complications that should be monitored for in this patient? Select all that apply. A) Orthostatic hypotension B) Autonomic dysreflexia C) DVT D) Salt-wasting syndrome E) Increased ICP

Ans: A, B, C Feedback: For a spinal cord-injured patient, based on the assessment data, potential complications that may develop include DVT, orthostatic hypotension, and autonomic dysreflexia. Salt-wasting syndrome or increased ICP are not typical complications following the immediate recovery period.

36. A family member of a patient diagnosed with Huntington disease calls you at the clinic. She is requesting help from the Huntington's Disease Society of America. What kind of help can this patient and family receive from this organization? Select all that apply. A) Information about this disease B) Referrals C) Public education D) Individual assessments E) Appraisals of research studies

Ans: A, B, C Feedback: The Huntington's Disease Society of America helps patients and families by providing information, referrals, family and public education, and support for research. It does not provide individual assessments or appraisals of individual research studies.

29. A patient with an inoperable brain tumor has been told that he has a short life expectancy. On what aspects of assessment and care should the home health nurse focus? Select all that apply. A) Pain control B) Management of treatment complications C) Interpretation of diagnostic tests D) Assistance with self-care E) Administration of treatments

Ans: A, B, D, E Feedback: Home care needs and interventions focus on four major areas: palliation of symptoms and pain control, assistance in self-care, control of treatment complications, and administration of specific forms of treatment, such as parenteral nutrition. Interpretation of diagnostic tests is normally beyond the purview of the nurse.

The school nurse is giving a presentation on preventing spinal cord injuries (SCI). What should the nurse identify as prominent risk factors for SCI? Select all that apply. A) Young age B) Frequent travel C) African American race D) Male gender E) Alcohol or drug use

Ans: A, D, E Feedback: The predominant risk factors for SCI include young age, male gender, and alcohol and drug use. Ethnicity and travel are not risk factors.

34. The nurse is caring for a patient diagnosed with Parkinson's disease. The patient is having increasing problems with rising from the sitting to the standing position. What should the nurse suggest to the patient to use that will aid in getting from the sitting to the standing position as well as aid in improving bowel elimination? A) Use of a bedpan B) Use of a raised toilet seat C) Sitting quietly on the toilet every 2 hours D) Following the outlined bowel program

Ans: B Feedback: A raised toilet seat is useful, because the patient has difficulty in moving from a standing to a sitting position. A handicapped toilet is not high enough and will not aid in improving bowel elimination. Sitting quietly on the toilet every 2 hours will not aid in getting from the sitting to standing position; neither will following the outlined bowel program.

18. A patient has just returned to the unit from the PACU after surgery for a tumor within the spine. The patient complains of pain. When positioning the patient for comfort and to reduce injury to the surgical site, the nurse will position to patient in what position? A) In the high Fowler's position B) In a flat side-lying position C) In the Trendelenberg position D) In the reverse Trendelenberg position

Ans: B Feedback: After spinal surgery, the bed is usually kept flat initially. The side-lying position is usually the most comfortable because this position imposes the least pressure on the surgical site. The Fowler's position, Trendelenberg position, and reverse Trendelenberg position are inappropriate for this patient because they would result in increased pain and complications.

An elderly woman found with a head injury on the floor of her home is subsequently admitted to the neurologic ICU. What is the best rationale for the following physician orders: elevate the HOB; keep the head in neutral alignment with no neck flexion or head rotation; avoid sharp hip flexion? A) To decrease cerebral arterial pressure B) To avoid impeding venous outflow C) To prevent flexion contractures D) To prevent aspiration of stomach contents

Ans: B Feedback: Any activity or position that impedes venous outflow from the head may contribute to increased volume inside the skull and possibly increase ICP. Cerebral arterial pressure will be affected by the balance between oxygen and carbon dioxide. Flexion contractures are not a priority at this time. Stomach contents could still be aspirated in this position.

A nurse is caring for a critically ill patient with autonomic dysreflexia. What clinical manifestations would the nurse expect in this patient? A) Respiratory distress and projectile vomiting B) Bradycardia and hypertension C) Tachycardia and agitation D) Third-spacing and hyperthermia

Ans: B Feedback: Autonomic dysreflexia is characterized by a pounding headache, profuse sweating, nasal congestion, piloerection ("goose bumps"), bradycardia, and hypertension. It occurs in cord lesions above T6 after spinal shock has resolved; it does not result in vomiting, tachycardia

38. The nurse is caring for a patient who is scheduled for a cervical discectomy the following day. During health education, the patient should be made aware of what potential complications? A) Vertebral fracture B) Hematoma at the surgical site C) Scoliosis D) Renal trauma

Ans: B Feedback: Based on all the assessment data, the potential complications of diskectomy may include hematoma at the surgical site, resulting in cord compression and neurologic deficit and recurrent or persistent pain after surgery. Renal trauma and fractures are unlikely; scoliosis is a congenital malformation of the spine.

14. A patient diagnosed with a pituitary adenoma has arrived on the neurologic unit. When planning the patient's care, the nurse should be aware that the effects of the tumor will primarily depend on what variable? A) Whether the tumor utilizes aerobic or anaerobic respiration B) The specific hormones secreted by the tumor C) The patient's pre-existing health status D) Whether the tumor is primary or the result of metastasis

Ans: B Feedback: Functioning pituitary tumors can produce one or more hormones normally produced by the anterior pituitary and the effects of the tumor depend largely on the identity of these hormones. This variable is more significant than the patient's health status or whether the tumor is primary versus secondary. Anaerobic and aerobic respiration is not relevant.

A patient is brought to the trauma center by ambulance after sustaining a high cervical spinal cord injury 1½ hours ago. Endotracheal intubation has been deemed necessary and the nurse is preparing to assist. What nursing diagnosis should the nurse associate with this procedure? A) Risk for impaired skin integrity B) Risk for injury C) Risk for autonomic dysreflexia D) Risk for suffocation

Ans: B Feedback: If endotracheal intubation is necessary, extreme care is taken to avoid flexing or extending the patient's neck, which can result in extension of a cervical injury. Intubation does not directly cause autonomic dysreflexia and the threat to skin integrity is a not a primary concern. Intubation does not carry the potential to cause suffocation.

Following a spinal cord injury a patient is placed in halo traction. While performing pin site care, the nurse notes that one of the traction pins has become detached. The nurse would be correct in implementing what priority nursing action? A) Complete the pin site care to decrease risk of infection. B) Notify the neurosurgeon of the occurrence. C) Stabilize the head in a lateral position. D) Reattach the pin to prevent further head trauma.

Ans: B Feedback: If one of the pins became detached, the head is stabilized in neutral position by one person while another notifies the neurosurgeon. Reattaching the pin as a nursing intervention would not be done due to risk of increased injury. Pin site care would not be a priority in this instance. Prevention of neurologic injury is the priority.

16. The nurse in an extended care facility is planning the daily activities of a patient with postpolio syndrome. The nurse recognizes the patient will best benefit from physical therapy when it is scheduled at what time? A) Immediately after meals B) In the morning C) Before bedtime D) In the early evening

Ans: B Feedback: Important activities for patients with postpolio syndrome should be planned for the morning, as fatigue often increases in the afternoon and evening.

A patient with spinal cord injury has a nursing diagnosis of altered mobility and the nurse recognizes the increased the risk of deep vein thrombosis (DVT). Which of the following would be included as an appropriate nursing intervention to prevent a DVT from occurring? A) Placing the patient on a fluid restriction as ordered B) Applying thigh-high elastic stockings C) Administering an antifibrinolyic agent D) Assisting the patient with passive range of motion (PROM) exercises

Ans: B Feedback: It is important to promote venous return to the heart and prevent venous stasis in a patient with altered mobility. Applying elastic stockings will aid in the prevention of a DVT. The patient should not be placed on fluid restriction because a dehydrated state will increase the risk of clotting throughout the body. Antifibrinolytic agents cause the blood to clot, which is absolutely contraindicated in this situation. PROM exercises are not an effective protection against the development of DVT.

The staff educator is precepting a nurse new to the critical care unit when a patient with a T2 spinal cord injury is admitted. The patient is soon exhibiting manifestations of neurogenic shock. In addition to monitoring the patient closely, what would be the nurse's most appropriate action? A) Prepare to transfuse packed red blood cells. B) Prepare for interventions to increase the patient's BP. C) Place the patient in the Trendelenberg position. D) Prepare an ice bath to lower core body temperature.

Ans: B Feedback: Manifestations of neurogenic shock include decreased BP and heart rate. Cardiac markers would be expected to rise in cardiogenic shock. Transfusion, repositioning, and ice baths are not indicated interventions.

A neurologic flow chart is often used to document the care of a patient with a traumatic brain injury. At what point in the patient's care should the nurse begin to use a neurologic flow chart? A) When the patient's condition begins to deteriorate B) As soon as the initial assessment is made C) At the beginning of each shift D) When there is a clinically significant change in the patient's condition

Ans: B Feedback: Neurologic parameters are assessed initially and as frequently as the patient's condition requires. As soon as the initial assessment is made, the use of a neurologic flowchart is started and maintained. A new chart is not begun at the start of every shift.

A patient who has sustained a nondepressed skull fracture is admitted to the acute medical unit. Nursing care should include which of the following? A) Preparation for emergency craniotomy B) Watchful waiting and close monitoring C) Administration of inotropic drugs D) Fluid resuscitation

Ans: B Feedback: Nondepressed skull fractures generally do not require surgical treatment; however, close observation of the patient is essential. A craniotomy would not likely be needed if the fracture is nondepressed. Even if treatment is warranted, it is unlikely to include inotropes or fluid resuscitation.

A 25-year-old female patient with brain metastases is considering her life expectancy after her most recent meeting with her oncologist. Based on the fact that the patient is not receiving treatment for her brain metastases, what is the nurse's most appropriate action? A) Promoting the patient's functional status and ADLs B) Ensuring that the patient receives adequate palliative care C) Ensuring that the family does not tell the patient that her condition is terminal D) Promoting adherence to the prescribed medication regimen

Ans: B Feedback: Patients with intracerebral metastases who are not treated have a steady downhill course with a limited survival time, whereas those who are treated may survive for slightly longer periods, but for most cure is not possible. Palliative care is thus necessary. This is a priority over promotion of function and the family should not normally withhold information from the patient. Adherence to medications such as analgesics is important, but palliative care is a high priority.

13. A patient has been admitted to the neurologic unit for the treatment of a newly diagnosed brain tumor. The patient has just exhibited seizure activity for the first time. What is the nurse's priority response to this event? A) Identify the triggers that precipitated the seizure. B) Implement precautions to ensure the patient's safety. C) Teach the patient's family about the relationship between brain tumors and seizure activity. D) Ensure that the patient is housed in a private room.

Ans: B Feedback: Patients with seizures are carefully monitored and protected from injury. Patient safety is a priority over health education, even though this is appropriate and necessary. Specific triggers may or may not be evident; identifying these is not the highest priority. A private room is preferable, but not absolutely necessary.

6. The nurse is caring for a boy who has muscular dystrophy. When planning assistance with the patient's ADLs, what goal should the nurse prioritize? A) Promoting the patient's recovery from the disease B) Maximizing the patient's level of function C) Ensuring the patient's adherence to treatment D) Fostering the family's participation in care

Ans: B Feedback: Priority for the care of the child with muscular dystrophy is the need to maximize the patient's level of function. Family participation is also important, but should be guided by this goal. Adherence is not a central goal, even though it is highly beneficial, and the disease is not curable.

17. A patient newly diagnosed with a cervical disk herniation is receiving health education from the clinic nurse. What conservative management measures should the nurse teach the patient to implement? A) Perform active ROM exercises three times daily. B) Sleep on a firm mattress. C) Apply cool compresses to the back of the neck daily. D) Wear the cervical collar for at least 2 hours at a time.

Ans: B Feedback: Proper positioning on a firm mattress and bed rest for 1 to 2 days may bring dramatic relief from pain. The patient may need to wear a cervical collar 24 hours a day during the acute phase of pain from a cervical disk herniation. Hot, moist compresses applied to the back of the neck will increase blood flow to the muscles and help relax the spastic muscles.

A nurse is reviewing the trend of a patient's scores on the Glasgow Coma Scale (GCS). This allows the nurse to gauge what aspect of the patient's status? A) Reflex activity B) Level of consciousness C) Cognitive ability D) Sensory involvement

Ans: B Feedback: The Glasgow Coma Scale (GCS) examines three responses related to LOC: eye opening, best verbal response, and best motor response.

5. The clinic nurse caring for a patient with Parkinson's disease notes that the patient has been taking levodopa and carbidopa (Sinemet) for 7 years. For what common side effect of Sinemet would the nurse assesses this patient? A) Pruritus B) Dyskinesia C) Lactose intolerance D) Diarrhea

Ans: B Feedback: Within 5 to 10 years of taking levodopa, most patients develop a response to the medication characterized by dyskinesia (abnormal involuntary movements). Another potential complication of long-term dopaminergic medication use is neuroleptic malignant syndrome characterized by severe rigidity, stupor, and hyperthermia. Side effects of long-term Sinemet therapy are not pruritus, lactose intolerance, or diarrhea.

The nurse is caring for a patient who is rapidly progressing toward brain death. The nurse should be aware of what cardinal signs of brain death? Select all that apply. A) Absence of pain response B) Apnea C) Coma D) Absence of brain stem reflexes E) Absence of deep tendon reflexes

Ans: B, C, D Feedback: The three cardinal signs of brain death upon clinical examination are coma, the absence of brain stem reflexes, and apnea. Absences of pain response and deep tendon reflexes are not necessarily indicative of brain death.

A patient is admitted to the neurologic ICU with a C4 spinal cord injury. When writing the plan of care for this patient, which of the following nursing diagnoses would the nurse prioritize in the immediate care of this patient? A) Risk for impaired skin integrity related to immobility and sensory loss B) Impaired physical mobility related to loss of motor function C) Ineffective breathing patterns related to weakness of the intercostal muscles D) Urinary retention related to inability to void spontaneously

Ans: C Feedback: A nursing diagnosis related to breathing pattern would be the priority for this patient. A C4 spinal cord injury will require ventilatory support, due to the diaphragm and intercostals being affected. The other nursing diagnoses would be used in the care plan, but not designated as a higher priority than ineffective breathing patterns.

8. A patient with Parkinson's disease is undergoing a swallowing assessment because she has recently developed adventitious lung sounds. The patient's nutritional needs should be met by what method? A) Total parenteral nutrition (TPN) B) Provision of a low-residue diet C) Semisolid food with thick liquids D) Minced foods and a fluid restriction

Ans: C Feedback: A semisolid diet with thick liquids is easier for a patient with swallowing difficulties to consume than is a solid diet. Low-residue foods and fluid restriction are unnecessary and counterproductive to the patient's nutritional status. The patient's status does not warrant TPN.

The ED nurse is caring for a patient who has been brought in by ambulance after sustaining a fall at home. What physical assessment finding is suggestive of a basilar skull fracture? A) Epistaxis B) Periorbital edema C) Bruising over the mastoid D) Unilateral facial numbness

Ans: C Feedback: An area of ecchymosis (bruising) may be seen over the mastoid (Battle's sign) in a basilar skull fracture. Numbness, edema, and epistaxis are not directly associated with a basilar skull fracture.

A patient is brought to the ED by her family after falling off the roof. A family member tells the nurse that when the patient fell she was "knocked out," but came to and "seemed okay." Now she is complaining of a severe headache and not feeling well. The care team suspects an epidural hematoma, prompting the nurse to prepare for which priority intervention? A) Insertion of an intracranial monitoring device B) Treatment with antihypertensives C) Emergency craniotomy D) Administration of anticoagulant therapy

Ans: C Feedback: An epidural hematoma is considered an extreme emergency. Marked neurologic deficit or respiratory arrest can occur within minutes. Treatment consists of making an opening through the skull to decrease ICP emergently, remove the clot, and control the bleeding. Antihypertensive medications would not be a priority. Anticoagulant therapy should not be ordered for a patient who has a cranial bleed. This could further increase bleeding activity. Insertion of an intracranial monitoring device may be done during the surgery, but is not priority for this patient.

28. A patient with a brain tumor has begun to exhibit signs of cachexia. What subsequent assessment should the nurse prioritize? A) Assessment of peripheral nervous function B) Assessment of cranial nerve function C) Assessment of nutritional status D) Assessment of respiratory status

Ans: C Feedback: Cachexia is a wasting syndrome of weight loss, muscle atrophy, fatigue, weakness, and significant loss of appetite. Consequently, nutritional assessment is paramount.

31. A patient, brought to the clinic by his wife and son, is diagnosed with Huntington disease. When providing anticipatory guidance, the nurse should address the future possibility of what effect of Huntington disease? A) Metastasis B) Risk for stroke C) Emotional and personality changes D) Pathologic bone fractures

Ans: C Feedback: Huntington disease causes profound changes to personality and behavior. It is a nonmalignant disease and stroke is not a central risk. The disease is not associated with pathologic bone fractures.

39. The nurse responds to the call light of a patient who has had a cervical diskectomy earlier in the day. The patient states that she is having severe pain that had a sudden onset. What is the nurse's most appropriate action? A) Palpate the surgical site. B) Remove the dressing to assess the surgical site. C) Call the surgeon to report the patient's pain. D) Administer a dose of an NSAID.

Ans: C Feedback: If the patient experiences a sudden increase in pain, extrusion of the graft may have occurred, requiring reoperation. A sudden increase in pain should be promptly reported to the surgeon. Administration of an NSAID would be an insufficient response and the dressing should not be removed without an order. Palpation could cause further damage.

Paramedics have brought an intubated patient to the RD following a head injury due to acceleration-deceleration motor vehicle accident. Increased ICP is suspected. Appropriate nursing interventions would include which of the following? A) Keep the head of the bed (HOB) flat at all times. B) Teach the patient to perform the Valsalva maneuver. C) Administer benzodiazepines on a PRN basis. D) Perform endotracheal suctioning every hour.

Ans: C Feedback: If the patient with a brain injury is very agitated, benzodiazepines are the most commonly used sedatives and do not affect cerebral blood flow or ICP. The HOB should be elevated 30 degrees. Suctioning should be done a limited basis, due to increasing the pressure in the cranium. The Valsalva maneuver is to be avoided. This also causes increased ICP.

20. A patient with amyotrophic lateral sclerosis (ALS) is being visited by the home health nurse who is creating a care plan. What nursing diagnosis is most likely for a patient with this condition? A) Chronic confusion B) Impaired urinary elimination C) Impaired verbal communication D) Bowel incontinence

Ans: C Feedback: Impaired communication is an appropriate nursing diagnosis; the voice in patients with ALS assumes a nasal sound and articulation becomes so disrupted that speech is unintelligible. Intellectual function is marginally impaired in patients with late ALS. Usually, the anal and bladder sphincters are intact because the spinal nerves that control muscles of the rectum and urinary bladder are not affected.

A patient is admitted to the neurologic ICU with a spinal cord injury. When assessing the patient the nurse notes there is a sudden depression of reflex activity in the spinal cord below the level of injury. What should the nurse suspect? A) Epidural hemorrhage B) Hypertensive emergency C) Spinal shock D) Hypovolemia

Ans: C Feedback: In spinal shock, the reflexes are absent, BP and heart rate fall, and respiratory failure can occur. Hypovolemia, hemorrhage, and hypertension do not cause this sudden change in neurologic function.

35. A patient with Parkinson's disease is experiencing episodes of constipation that are becoming increasingly frequent and severe. The patient states that he has been achieving relief for the past few weeks by using OTC laxatives. How should the nurse respond? A) "It's important to drink plenty of fluids while you're taking laxatives." B) "Make sure that you supplement your laxatives with a nutritious diet." C) "Let's explore other options, because laxatives can have side effects and create dependency." D) "You should ideally be using herbal remedies rather than medications to promote bowel function."

Ans: C Feedback: Laxatives should be avoided in patients with Parkinson's disease due to the risk of adverse effects and dependence. Herbal bowel remedies are not necessarily less risky.

The nurse recognizes that a patient with a SCI is at risk for muscle spasticity. How can the nurse best prevent this complication of an SCI? A) Position the patient in a high Fowler's position when in bed. B) Support the knees with a pillow when the patient is in bed. C) Perform passive ROM exercises as ordered. D) Administer NSAIDs as ordered.

Ans: C Feedback: Passive ROM exercises can prevent muscle spasticity following SCI. NSAIDs are not used for this purpose. Pillows and sitting upright do not directly address the patient's risk of muscle spasticity.

A patient is admitted to the neurologic ICU with a spinal cord injury. In writing the patient's care plan, the nurse specifies that contractures can best be prevented by what action? A) Repositioning the patient every 2 hours B) Initiating range-of-motion exercises (ROM) as soon as the patient initiates C) Initiating (ROM) exercises as soon as possible after the injury D) Performing ROM exercises once a day

Ans: C Feedback: Passive ROM exercises should be implemented as soon as possible after injury. It would be inappropriate to wait for the patient to first initiate exercises. Toes, metatarsals, ankles, knees, and hips should be put through a full ROM at least four, and ideally five, times daily. Repositioning alone will not prevent contractures.

The nurse is writing a care plan for a patient with brain metastases. The nurse decides that an appropriate nursing diagnosis is "anxiety related to lack of control over the health circumstances." In establishing this plan of care for the patient, the nurse should include what intervention? A) The patient will receive antianxiety medications every 4 hours. B) The patient's family will be instructed on planning the patient's care. C) The patient will be encouraged to verbalize concerns related to the disease and its treatment. D) The patient will begin intensive therapy with the goal of distraction.

Ans: C Feedback: Patients need the opportunity to exercise some control over their situation. A sense of mastery can be gained as they learn to understand the disease and its treatment and how to deal with their feelings. Distraction and administering medications will not allow the patient to gain control over anxiety. Delegating planning to the family will not help the patient gain a sense of control and autonomy.

22. The nurse is caring for a patient newly diagnosed with a primary brain tumor. The patient asks the nurse where his tumor came from. What would be the nurse's best response? A) "Your tumor originated from somewhere outside the CNS." B) "Your tumor likely started out in one of your glands." C) "Your tumor originated from cells within your brain itself." D) "Your tumor is from nerve tissue somewhere in your body."

Ans: C Feedback: Primary brain tumors originate from cells and structures within the brain. Secondary brain tumors are metastatic tumors that originate somewhere else in the body. The scenario does not indicate that the patient's tumor is a pituitary tumor or a neuroma.

Splints have been ordered for a patient who is at risk of developing footdrop following a spinal cord injury. The nurse caring for this patient knows that the splints are removed and reapplied when? A) At the patient's request B) Each morning and evening C) Every 2 hours D) One hour prior to mobility exercises

Ans: C Feedback: The feet are prone to footdrop; therefore, various types of splints are used to prevent footdrop. When used, the splints are removed and reapplied every 2 hours.

The nurse planning the care of a patient with head injuries is addressing the patient's nursing diagnosis of "sleep deprivation." What action should the nurse implement? A) Administer a benzodiazepine at bedtime each night. B) Do not disturb the patient between 2200 and 0600. C) Cluster overnight nursing activities to minimize disturbances. D) Ensure that the patient does not sleep during the day.

Ans: C Feedback: To allow the patient longer times of uninterrupted sleep and rest, the nurse can group nursing care activities so that the patient is disturbed less frequently. However, it is impractical and unsafe to provide no care for an 8-hour period. The use of benzodiazepines should be avoided.

The nurse caring for a patient with a spinal cord injury notes that the patient is exhibiting early signs and symptoms of disuse syndrome. Which of the following is the most appropriate nursing action? A) Limit the amount of assistance provided with ADLs. B) Collaborate with the physical therapist and immobilize the patient's extremities temporarily. C) Increase the frequency of ROM exercises. D) Educate the patient about the importance of frequent position changes.

Ans: C Feedback: To prevent disuse syndrome, ROM exercises must be provided at least four times a day, and care is taken to stretch the Achilles tendon with exercises. The patient is repositioned frequently and is maintained in proper body alignment whether in bed or in a wheelchair. The patient must be repositioned by caregivers, not just taught about repositioning. It is inappropriate to limit assistance for the sole purpose of preventing disuse syndrome.

A patient with a spinal cord injury has experienced several hypotensive episodes. How can the nurse best address the patient's risk for orthostatic hypotension? A) Administer an IV bolus of normal saline prior to repositioning. B) Maintain bed rest until normal BP regulation returns. C) Monitor the patient's BP before and during position changes. D) Allow the patient to initiate repositioning.

Ans: C Feedback: To prevent hypotensive episodes, close monitoring of vital signs before and during position changes is essential. Prolonged bed rest carries numerous risks and it is not possible to provide a bolus before each position change. Following the patient's lead may or may not help regulate BP.

24. A patient who has been experiencing numerous episodes of unexplained headaches and vomiting has subsequently been referred for testing to rule out a brain tumor. What characteristic of the patient's vomiting is most consistent with a brain tumor? A) The patient's vomiting is accompanied by epistaxis. B) The patient's vomiting does not relieve his nausea. C) The patient's vomiting is unrelated to food intake. D) The patient's emesis is blood-tinged.

Ans: C Feedback: Vomiting is often unrelated to food intake if caused by a brain tumor. The presence or absence of blood is not related to the possible etiology and vomiting may or may not relieve the patient's nausea.

21. The nurse educator is discussing neoplasms with a group of recent graduates. The educator explains that the effects of neoplasms are caused by the compression and infiltration of normal tissue. The physiologic changes that result can cause what pathophysiologic events? Select all that apply. A) Intracranial hemorrhage B) Infection of cerebrospinal fluid C) Increased ICP D) Focal neurologic signs E) Altered pituitary function

Ans: C, D, E Feedback: The effects of neoplasms are caused by the compression and infiltration of tissue. A variety of physiologic changes result, causing any or all of the following pathophysiologic events: increased ICP and cerebral edema, seizure activity and focal neurologic signs, hydrocephalus, and altered pituitary function.

A patient who suffered a spinal cord injury is experiencing an exaggerated autonomic response. What aspect of the patient's current health status is most likely to have precipitated this event? A) The patient received a blood transfusion. B) The patient's analgesia regimen was recent changed. C) The patient was not repositioned during the night shift. D) The patient's urinary catheter became occluded.

Ans: D Feedback: A distended bladder is the most common cause of autonomic dysreflexia. Infrequent positioning is a less likely cause, although pressure ulcers or tactile stimulation can cause it. Changes in mediations or blood transfusions are unlikely causes.

9. While assessing the patient at the beginning of the shift, the nurse inspects a surgical dressing covering the operative site after the patients' cervical diskectomy. The nurse notes that the drainage is 75% saturated with serosanguineous discharge. What is the nurse's most appropriate action? A) Page the physician and report this sign of infection. B) Reinforce the dressing and reassess in 1 to 2 hours. C) Reposition the patient to prevent further hemorrhage. D) Inform the surgeon of the possibility of a dural leak.

Ans: D Feedback: After a cervical diskectomy, the nurse will monitor the operative site and dressing covering this site. Serosanguineous drainage may indicate a dural leak. This constitutes a risk for meningitis, but is not a direct sign of infection. This should be reported to the surgeon, not just reinforced and observed.

The ED is notified that a 6-year-old is in transit with a suspected brain injury after being struck by a car. The child is unresponsive at this time, but vital signs are within acceptable limits. What will be the primary goal of initial therapy? A) Promoting adequate circulation B) Treating the child's increased ICP C) Assessing secondary brain injury D) Preserving brain homeostasis

Ans: D Feedback: All therapy is directed toward preserving brain homeostasis and preventing secondary brain injury, which is injury to the brain that occurs after the original traumatic event. The scenario does not indicate the child has increased ICP or a secondary brain injury at this point. Promoting circulation is likely secondary to the broader goal of preserving brain homeostasis.

The nurse has implemented interventions aimed at facilitating family coping in the care of a patient with a traumatic brain injury. How can the nurse best facilitate family coping? A) Help the family understand that the patient could have died. B) Emphasize the importance of accepting the patient's new limitations. C) Have the members of the family plan the patient's inpatient care. D) Assist the family in setting appropriate short-term goals.

Ans: D Feedback: Helpful interventions to facilitate coping include providing family members with accurate and honest information and encouraging them to continue to set well-defined, short-term goals. Stating that a patient's condition could be worse downplays their concerns. Emphasizing the importance of acceptance may not necessarily help the family accept the patient's condition. Family members cannot normally plan a patient's hospital care, although they may contribute to the care in some ways.

A 13-year-old was brought to the ED, unconscious, after being hit in the head by a baseball. When the child regains consciousness, 5 hours after being admitted, he cannot remember the traumatic event. MRI shows no structural sign of injury. What injury would the nurse suspect the patient has? A) Diffuse axonal injury B) Grade 1 concussion with frontal lobe involvement C) Contusion D) Grade 3 concussion with temporal lobe involvement

Ans: D Feedback: In a grade 3 concussion there is a loss of consciousness lasting from seconds to minutes. Temporal lobe involvement results in amnesia. Frontal lobe involvement can cause uncharacteristic behavior and a grade 1 concussion does not involve loss of consciousness. Diagnostic studies may show no apparent structural sign of injury, but the duration of unconsciousness is an indicator of the severity of the concussion. Diffuse axonal injury (DAI) results from widespread shearing and rotational forces that produce damage throughout the brain—to axons in the cerebral hemispheres, corpus callosum, and brain stem. In cerebral contusion, a moderate to severe head injury, the brain is bruised and damaged in a specific area because of severe acceleration-deceleration force or blunt trauma.

11. A patient has just been diagnosed with Parkinson's disease and the nurse is planning the patient's subsequent care for the home setting. What nursing diagnosis should the nurse address when educating the patient's family? A) Risk for infection B) Impaired spontaneous ventilation C) Unilateral neglect D) Risk for injury

Ans: D Feedback: Individuals with Parkinson's disease face a significant risk for injury related to the effects of dyskinesia. Unilateral neglect is not characteristic of the disease, which affects both sides of the body. Parkinson's disease does not directly constitute a risk for infection or impaired respiration.

10. A patient, diagnosed with cancer of the lung, has just been told he has metastases to the brain. What change in health status would the nurse attribute to the patient's metastatic brain disease? A) Chronic pain B) Respiratory distress C) Fixed pupils D) Personality changes

Ans: D Feedback: Neurologic signs and symptoms include headache, gait disturbances, visual impairment, personality changes, altered mentation (memory loss and confusion), focal weakness, paralysis, aphasia, and seizures. Pain, respiratory distress, and fixed pupils are not among the more common neurologic signs and symptoms of metastatic brain disease.

12. The nurse is caring for a patient with Huntington disease who has been admitted to the hospital for treatment of malnutrition. What independent nursing action should be implemented in the patient's plan of care? A) Firmly redirect the patient's head when feeding. B) Administer phenothiazines after each meal as ordered. C) Encourage the patient to keep his or her feeding area clean. D) Apply deep, gentle pressure around the patient's mouth to aid swallowing.

Ans: D Feedback: Nursing interventions for a patient who has inadequate nutritional intake should include the following: Apply deep gentle pressure around the patient's mouth to assist with swallowing, and administer phenothiazines prior to the patient's meal as ordered. The nurse should disregard the mess of the feeding area and treat the person with dignity. Stiffness and turning away by the patient during feeding are uncontrollable choreiform movements and should not be interrupted.

A nurse on the neurologic unit is providing care for a patient who has spinal cord injury at the level of C4. When planning the patient's care, what aspect of the patient's neurologic and functional status should the nurse consider? A) The patient will be unable to use a wheelchair. B) The patient will be unable to swallow food. C) The patient will be continent of urine, but incontinent of bowel. D) The patient will require full assistance for all aspects of elimination

Ans: D Feedback: Patients with a lesion at C4 are fully dependent for elimination. The patient is dependent for feeding, but is able to swallow. The patient will be capable of using an electric wheelchair.

7. A 37-year-old man is brought to the clinic by his wife because he is experiencing loss of motor function and sensation. The physician suspects the patient has a spinal cord tumor and hospitalizes him for diagnostic testing. In light of the need to diagnose spinal cord compression from a tumor, the nurse will most likely prepare the patient for what test? A) Anterior-posterior x-ray B) Ultrasound C) Lumbar puncture D) MRI

Ans: D Feedback: The MRI scan is the most commonly used diagnostic procedure. It is the most sensitive diagnostic tool that is particularly helpful in detecting epidural spinal cord compression and vertebral bone metastases.

32. A patient who was diagnosed with Parkinson's disease several months ago recently began treatment with levodopa-carbidopa. The patient and his family are excited that he has experienced significant symptom relief. The nurse should be aware of what implication of the patient's medication regimen? A) The patient is in a "honeymoon period" when adverse effects of levodopa-carbidopa are not yet evident. B) Benefits of levodopa-carbidopa do not peak until 6 to 9 months after the initiation of treatment. C) The patient's temporary improvement in status is likely unrelated to levodopa-carbidopa. D) Benefits of levodopa-carbidopa often diminish after 1 or 2 years of treatment.

Ans: D Feedback: The beneficial effects of levodopa therapy are most pronounced in the first year or two of treatment. Benefits begin to wane and adverse effects become more severe over time. However, a "honeymoon period" of treatment is not known.

An ED nurse has just received a call from EMS that they are transporting a 17-year-old man who has just sustained a spinal cord injury (SCI). The nurse recognizes that the most common cause of this type of injury is what? A) Sports-related injuries B) Acts of violence C) Injuries due to a fall D) Motor vehicle accidents

Ans: D Feedback: The most common causes of SCIs are motor vehicle crashes (46%), falls (22%), violence (16%), and sports (12%).

15. A male patient with a metastatic brain tumor is having a generalized seizure and begins vomiting. What should the nurse do first? A) Perform oral suctioning. B) Page the physician. C) Insert a tongue depressor into the patient's mouth. D) Turn the patient on his side.

Ans: D Feedback: The nurse's first response should be to place the patient on his side to prevent him from aspirating emesis. Inserting something into the seizing patient's mouth is no longer part of a seizure protocol. Obtaining supplies to suction the patient would be a delegated task. Paging or calling the physician would only be necessary if this is the patient's first seizure.

A patient with a head injury has been increasingly agitated and the nurse has consequently identified a risk for injury. What is the nurse's best intervention for preventing injury? A) Restrain the patient as ordered. B) Administer opioids PRN as ordered. C) Arrange for friends and family members to sit with the patient. D) Pad the side rails of the patient's bed.

Ans: D Feedback: To protect the patient from self-injury, the nurse uses padded side rails. The nurse should avoid restraints, because straining against them can increase ICP or cause other injury. Narcotics used to control restless patients should be avoided because these medications can depress respiration, constrict the pupils, and alter the patient's responsiveness. Visitors should be limited if the patient is agitated.

The critical care nurse and the other members of the care team are assessing the patient to see if he is ready to be weaned from the ventilator. What are the most important predictors of successful weaning that the nurse should identify? A) Stable vital signs and ABGs B) Pulse oximetry above 80% and stable vital signs C) Stable nutritional status and ABGs D) Normal orientation and level of consciousness

Ans: A Feedback: Among many other predictors, stable vital signs and ABGs are important predictors of successful weaning. Pulse oximetry must greatly exceed 80%. Nutritional status is important, but vital signs and ABGs are even more significant. Patients who are weaned may or may not have full level of consciousness.

Postural drainage has been ordered for a patient who is having difficulty mobilizing her bronchial secretions. Before repositioning the patient and beginning treatment, the nurse should perform what health assessment? A) Chest auscultation B) Pulmonary function testing C) Chest percussion D) Thoracic palpation

Ans: A Feedback: Chest auscultation should be performed before and after postural drainage in order to evaluate the effectiveness of the therapy. Percussion and palpation are less likely to provide clinically meaningful data for the nurse. PFTs are normally beyond the scope of the nurse and are not necessary immediately before postural drainage.

A patient is exhibiting signs of a pneumothorax following tracheostomy. The surgeon inserts a chest tube into the anterior chest wall. What should the nurse tell the family is the primary purpose of this chest tube? A) To remove air from the pleural space B) To drain copious sputum secretions C) To monitor bleeding around the lungs D) To assist with mechanical ventilation

Ans: A Feedback: Chest tubes and closed drainage systems are used to re-expand the lung involved and to remove excess air, fluid, and blood. The primary purpose of a chest tube is not to drain sputum secretions, monitor bleeding, or assist with mechanical ventilation.

A critical care nurse is caring for a client with an endotracheal tube who is on a ventilator. The nurse knows that meticulous airway management of this patient is necessary. What is the main rationale for this? A) Maintaining a patent airway B) Preventing the need for suctioning C) Maintaining the sterility of the patients airway D) Increasing the patients lung compliance

Ans: A Feedback: Maintaining a patent (open) airway is achieved through meticulous airway management, whether in an emergency situation such as airway obstruction or in long-term management, as in caring for a patient with an endotracheal or a tracheostomy tube. The other answers are incorrect.

The nurse has admitted a patient who is scheduled for a thoracic resection. The nurse is providing preoperative teaching and is discussing several diagnostic studies that will be required prior to surgery. Which study will be performed to determine whether the planned resection will leave sufficient functioning lung tissue? A) Pulmonary function studies B) Exercise tolerance tests C) Arterial blood gas values D) Chest x-ray

Ans: A Feedback: Pulmonary function studies are performed to determine whether the planned resection will leave sufficient functioning lung tissue. ABG values are assessed to provide a more complete picture of the functional capacity of the lung. Exercise tolerance tests are useful to determine if the patient who is a candidate for pneumonectomy can tolerate removal of one of the lungs. Preoperative studies, such as a chest x-ray, are performed to provide a baseline for comparison during the postoperative period and to detect any unsuspected abnormalities.

A nurse practitioner has provided care for three different patients with chronic pharyngitis over the past several months. Which patients are at greatest risk for developing chronic pharyngitis? A) Patients who are habitual users of alcohol and tobacco B) Patients who are habitual users of caffeine and other stimulants C) Patients who eat a diet high in spicy foods D) Patients who have gastrointestinal reflux disease (GERD)

Ans: A Chronic pharyngitis is common in adults who live and work in dusty surroundings, use the voice to excess, suffer from chronic chough, and habitually use alcohol and tobacco. Caffeine and spicy foods have not been linked to chronic pharyngitis. GERD is not a noted risk factor.

You are creating a nursing care plan for a patient who is hospitalized following right total hip replacement. What nursing action should you specify to prevent inward rotation of the patient's hip when the patient is in a partial lateral position? A) Use of an abduction pillow between the patient's legs B) Alignment of the head with the spine using a pillow C) Support of the lower back with a small pillow D) Placement of trochanter rolls under the greater trochanter

Ans: A Feedback: Abduction pillows can be used to keep the hip in correct alignment if precautions are warranted following hip replacement. Trochanter rolls and back pillows do not achieve this goal.

The nurse is caring for an older adult patient who is receiving rehabilitation following an ischemic stroke. A review of the patient's electronic health record reveals that the patient usually defers her self-care to family members or members of the care team. What should the nurse include as an initial goal when planning this patient's subsequent care? A) The patient will demonstrate independent self-care. B) The patient's family will collaboratively manage the patient's care. C) The nurse will delegate the patient's care to a nursing assistant. D) The patient will participate in a life skills program.

Ans: A Feedback: An appropriate patient goal will focus on the patient demonstrating independent self-care. The rehabilitation process helps patients achieve an acceptable quality of life with dignity, self-respect, and independence. The other options are incorrect because an appropriate goal would not be for the family to manage the patient's care, the patient's care would not be delegated to a nursing assistant, and participating in a social program is not an appropriate initial goal.

The ED nurse is assessing a young gymnast who fell from a balance beam. The gymnast presents with a clear fluid leaking from her nose. What should the ED nurse suspect? A) Fracture of the cribriform plate B) Rupture of an ethmoid sinus C) Abrasion of the soft tissue D) Fracture of the nasal septum

Ans: A Feedback: Clear fluid from either nostril suggests a fracture of the cribriform plate with leakage of cerebrospinal fluid. The symptoms are not indicative of an abrasion of the soft tissue or rupture of a sinus. Clear fluid leakage from the nose would not be indicative of a fracture of the nasal septum.

The nurse is caring for a patient who needs education on his medication therapy for allergic rhinitis. The patient is to take cromolyn (Nasalcrom) daily. In providing education for this patient, how should the nurse describe the action of the medication? A) It inhibits the release of histamine and other chemicals. B) It inhibits the action of proton pumps. C) It inhibits the action of the sodium-potassium pump in the nasal epithelium. D) It causes bronchodilation and relaxes smooth muscle in the bronchi.

Ans: A Feedback: Cromolyn (Nasalcrom) inhibits the release of histamine and other chemicals. It is prescribed to treat allergic rhinitis. Beta-adrenergic agents lead to bronchodilation and stimulate beta-2adrenergic receptors in the smooth muscle of the bronchi and bronchioles. It does not affect proton pump action or the sodium-potassium pump in the nasal cells.

A school nurse is providing health promotion teaching to a group of high school seniors. The nurse should highlight what salient risk factor for traumatic brain injury? A) Substance abuse B) Sports participation C) Anger mismanagement D) Lack of community resources

Ans: A Feedback: Of spinal cord injuries, 50% are related to substance abuse, and approximately 50% of all patients with traumatic brain injury were intoxicated at the time of injury. This association exceeds the significance of sports participation, anger mismanagement, or lack of community resources.

A patient has had a nasogastric tube in place for 6 days due to the development of paralytic ileus after surgery. In light of the prolonged presence of the nasogastric tube, the nurse should prioritize assessments related to what complication? A) Sinus infections B) Esophageal strictures C) Pharyngitis D) Laryngitis

Ans: A Feedback: Patients with nasotracheal and nasogastric tubes in place are at risk for development of sinus infections. Thus, accurate assessment of patients with these tubes is critical. Use of a nasogastric tube is not associated with the development of the other listed pathologies.

A patient who is receiving rehabilitation following a spinal cord injury has been diagnosed with reflex incontinence. The nurse caring for the patient should include which intervention in this patient's plan of care? A) Regular perineal care to prevent skin breakdown B) Kegel exercises to strengthen the pelvic floor C) Administration of hypotonic IV fluid D) Limited fluid intake to prevent incontinence

Ans: A Feedback: Reflex incontinence is associated with a spinal cord lesion that interrupts cerebral control, resulting in no sensory awareness of the need to void. Total incontinence occurs in patients with a psychological impairment when they cannot control excreta. A patient who is paralyzed cannot perform Kegel exercises. Intravenous fluids would make no difference in reflex incontinence. Limited fluid intake would make no impact on a patient's inability to sense the need to void.

The OR nurse is setting up a water-seal chest drainage system for a patient who has just had a thoracotomy. The nurse knows that the amount of suction in the system is determined by the water level. At what suction level should the nurse set the system? A) 20 cm H2O B) 15 cm H2O C) 10 cm H2O D) 5 cm H2O

Ans: A Feedback: The amount of suction is determined by the water level. It is usually set at 20 cm H2O; adding more fluid results in more suction.

You are the nurse creating the care plan for a patient newly admitted to your rehabilitation unit. The patient is an 82-year-old patient who has had a stroke but who lived independently until this event. What is a goal that you should include in this patient's nursing care plan? A) Maintain joint mobility. B) Refer to social services. C) Ambulate three times every day. D) Perform passive range of motion twice daily.

Ans: A Feedback: The major goals may include absence of contracture and deformity, maintenance of muscle strength and joint mobility, independent mobility, increased activity tolerance, and prevention of further disability. The other listed actions are interventions, not goals.

The home care nurse is assessing the home environment of a patient who will be discharged from the hospital shortly after his laryngectomy. The nurse should inform the patient that he may need to arrange for the installation of which system in his home? A) A humidification system B) An air conditioning system C) A water purification system D) A radiant heating system

Ans: A Feedback: The nurse stresses the importance of humidification at home and instructs the family to obtain and set up a humidification system before the patient returns home. Air-conditioning may be too cool and too drying for the patient. A water purification system or a radiant heating system is not necessary.

An adult patient's current goals of rehabilitation focus primarily on self-care. What is a priority when teaching a patient who has self-care deficits in ADLs? A) To provide an optimal learning environment with minimal distractions B) To describe the evidence base for any chosen interventions C) To help the patient become aware of the requirements of assisted-living centers D) To ensure that the patient is able to perform self-care without any aid from caregivers

Ans: A Feedback: The nurse's role is to provide an optimal learning environment that minimizes distractions. Describing the evidence base is not a priority, though nursing actions should indeed be evidence-based. Assisted-living facilities are not relevant to most patients. Absolute independence in ADLs is not an appropriate goal for every patient.

The nurse is caring for a patient who is ready to be weaned from the ventilator. In preparing to assist in the collaborative process of weaning the patient from a ventilator, the nurse is aware that the weaning of the patient will progress in what order? A) Removal from the ventilator, tube, and then oxygen B) Removal from oxygen, ventilator, and then tube C) Removal of the tube, oxygen, and then ventilator D) Removal from oxygen, tube, and then ventilator

Ans: A Feedback: The process of withdrawing the patient from dependence on the ventilator takes place in three stages: the patient is gradually removed from the ventilator, then from the tube, and, finally, oxygen.

A patient has been discharged home after thoracic surgery. The home care nurse performs the initial visit and finds the patient discouraged and saddened. The client states, I am recovering so slowly. I really thought I would be better by now. What nursing action should the nurse prioritize? A) Provide emotional support to the patient and family. B) Schedule a visit to the patients primary physician within 24 hours. C) Notify the physician that the patient needs a referral to a psychiatrist. D) Place a referral for a social worker to visit the patient.

Ans: A Feedback: The recovery process may take longer than the patient had expected, and providing support to the patient is an important task for the home care nurse. It is not necessary, based on this scenario, to schedule a visit with the physician within 24 hours, or to get a referral to a psychiatrist or a social worker.

You are the nurse caring for an elderly patient who has been on a bowel training program due to the neurologic effects of a stroke. In the past several days, the patient has begun exhibiting normal bowel patterns. Once a bowel routine has been well established, you should avoid which of the following? A) Use of a bedpan B) Use of a padded or raised commode C) Massage of the patient's abdomen D) Use of a bedside toilet

Ans: A Feedback: Use of bedpans should be avoided once a bowel routine has been established. An acceptable alternative to a private bathroom is a padded commode or bedside toilet. Massaging the abdomen from right to left facilitates movement of feces in the lower tract.

The nurse is caring for a patient whose recent unexplained weight loss and history of smoking have prompted diagnostic testing for cancer. What symptom is most closely associated with the early stages of laryngeal cancer? A) Hoarseness B) Dyspnea C) Dysphagia D) Frequent nosebleeds

Ans: A Hoarseness is an early symptom of laryngeal cancer. Dyspnea, dysphagia, and lumps are later signs of laryngeal cancer. Alopecia is not associated with a diagnosis of laryngeal cancer.

The nurse is providing patient teaching to a patient diagnosed with acute rhinosinusitis. For what possible complication should the nurse teach the patient to seek immediate follow-up? A) Periorbital edema B) Headache unrelieved by OTC medications C) Clear drainage from nose D) Blood-tinged mucus when blowing the nose

Ans: A Patient teaching is an important aspect of nursing care for the patient with acute rhinosinusitis. The nurse instructs the patient about symptoms of complications that require immediate follow-up. Referral to a physician is indicated if periorbital edema and severe pain on palpation occur. Clear drainage and blood- tinged mucus do not require follow-up if the patient has acute rhinosinusitis. A persistent headache does not necessarily warrant immediate follow-up.

A patient has been brought to the ED by the paramedics. The patient is suspected of having ARDS. What intervention should the nurse first anticipate? A) Preparing to assist with intubating the patient B) Setting up oxygen at 5 L/minute by nasal cannula C) Performing deep suctioning D) Setting up a nebulizer to administer corticosteroids

Ans: A Feedback: A patient who has ARDS usually requires intubation and mechanical ventilation. Oxygen by nasal cannula would likely be insufficient. Deep suctioning and nebulizers may be indicated, but the priority is to secure the airway.

An x-ray of a trauma patient reveals rib fractures and the patient is diagnosed with a small flail chest injury. Which intervention should the nurse include in the patients plan of care? A) Suction the patients airway secretions. B) Immobilize the ribs with an abdominal binder. C) Prepare the patient for surgery. D) Immediately sedate and intubate the patient.

Ans: A Feedback: As with rib fracture, treatment of flail chest is usually supportive. Management includes clearing secretions from the lungs, and controlling pain. If only a small segment of the chest is involved, it is important to clear the airway through positioning, coughing, deep breathing, and suctioning. Intubation is required for severe flail chest injuries, and surgery is required only in rare circumstances to stabilize the flail segment.

The nurse at a long-term care facility is assessing each of the residents. Which resident most likely faces the greatest risk for aspiration? A) A resident who suffered a severe stroke several weeks ago B) A resident with mid-stage Alzheimers disease C) A 92-year-old resident who needs extensive help with ADLs D) A resident with severe and deforming rheumatoid arthritis

Ans: A Feedback: Aspiration may occur if the patient cannot adequately coordinate protective glottic, laryngeal, and cough reflexes. These reflexes are often affected by stroke. A patient with mid-stage Alzheimers disease does not likely have the voluntary muscle problems that occur later in the disease. Clients that need help with ADLs or have severe arthritis should not have difficulty swallowing unless it exists secondary to another problem.

The perioperative nurse is writing a care plan for a patient who has returned from surgery 2 hours prior. Which measure should the nurse implement to most decrease the patients risk of developing pulmonary emboli (PE)? A) Early ambulation B) Increased dietary intake of protein C) Maintaining the patient in a supine position D) Administering aspirin with warfarin

Ans: A Feedback: For patients at risk for PE, the most effective approach for prevention is to prevent deep vein thrombosis. Active leg exercises to avoid venous stasis, early ambulation, and use of elastic compression stocking are general preventive measures. The patient does not require increased dietary intake of protein directly related to prevention of PE, although it will assist in wound healing during the postoperative period. The patient should not be maintained in one position, but frequently repositioned, unless contraindicated by the surgical procedure. Aspirin should never be administered with warfarin because it will increase the patients risk for bleeding.

The nurse is assessing an adult patient following a motor vehicle accident. The nurse observes that the patient has an increased use of accessory muscles and is complaining of chest pain and shortness of breath. The nurse should recognize the possibility of what condition? A) Pneumothorax B) Anxiety C) Acute bronchitis D) Aspiration

Ans: A Feedback: If the pneumothorax is large and the lung collapses totally, acute respiratory distress occurs. The patient is anxious, has dyspnea and air hunger, has increased use of the accessory muscles, and may develop central cyanosis from severe hypoxemia. These symptoms are not definitive of pneumothorax, but because of the patients recent trauma they are inconsistent with anxiety, bronchitis, or aspiration.

The nurse is caring for a patient who is scheduled for a lobectomy for a diagnosis of lung cancer. While assisting with a subclavian vein central line insertion, the nurse notes the clients oxygen saturation rapidly dropping. The patient complains of shortness of breath and becomes tachypnic. The nurse suspects a pneumothorax has developed. Further assessment findings supporting the presence of a pneumothorax include what? A) Diminished or absent breath sounds on the affected side B) Paradoxical chest wall movement with respirations C) Sudden loss of consciousness D) Muffled heart sounds

Ans: A Feedback: In the case of a simple pneumothorax, auscultating the breath sounds will reveal absent or diminished breath sounds on the affected side. Paradoxical chest wall movements occur in flail chest conditions. Sudden loss of consciousness does not typically occur. Muffled or distant heart sounds occur in pericardial tamponade.

The school nurse is presenting a class on smoking cessation at the local high school. A participant in the class asks the nurse about the risk of lung cancer in those who smoke. What response related to risk for lung cancer in smokers is most accurate? A) The younger you are when you start smoking, the higher your risk of lung cancer. B) The risk for lung cancer never decreases once you have smoked, which is why smokers need annual chest x-rays. C) The risk for lung cancer is determined mostly by what type of cigarettes you smoke. D) The risk for lung cancer depends primarily on the other risk factors for cancer that you have.

Ans: A Feedback: Risk is determined by the pack-year history (number of packs of cigarettes used each day, multiplied by the number of years smoked), the age of initiation of smoking, the depth of inhalation, and the tar and nicotine levels in the cigarettes smoked. The younger a person is when he or she starts smoking, the greater the risk of developing lung cancer. Risk declines after smoking cessation. The type of cigarettes is a significant variable, but this is not the most important factor.

The nurse is caring for a patient at risk for atelectasis. The nurse implements a first-line measure to prevent atelectasis development in the patient. What is an example of a first-line measure to minimize atelectasis? A) Incentive spirometry B) Intermittent positive-pressure breathing (IPPB) C) Positive end-expiratory pressure (PEEP) D) Bronchoscopy

Ans: A Feedback: Strategies to prevent atelectasis, which include frequent turning, early ambulation, lung-volume expansion maneuvers (deep breathing exercises, incentive spirometry), and coughing, serve as the firstline measures to minimize or treat atelectasis by improving ventilation. In patients who do not respond to first-line measures or who cannot perform deep-breathing exercises, other treatments such as positive end-expiratory pressure (PEEP), continuous or intermittent positive-pressure breathing (IPPB), or bronchoscopy may be used.

An adult patient has tested positive for tuberculosis (TB). While providing patient teaching, what information should the nurse prioritize? A) The importance of adhering closely to the prescribed medication regimen B) The fact that the disease is a lifelong, chronic condition that will affect ADLs C) The fact that TB is self-limiting, but can take up to 2 years to resolve D) The need to work closely with the occupational and physical therapists

Ans: A Feedback: Successful treatment of TB is highly dependent on careful adherence to the medication regimen. The disease is not self-limiting; occupational and physical therapy are not necessarily indicated. TB is curable.

While planning a patients care, the nurse identifies nursing actions to minimize the patients pleuritic pain. Which intervention should the nurse include in the plan of care? A) Avoid actions that will cause the patient to breathe deeply. B) Ambulate the patient at least three times daily. C) Arrange for a soft-textured diet and increased fluid intake. D) Encourage the patient to speak as little as possible

Ans: A Feedback: The key characteristic of pleuritic pain is its relationship to respiratory movement. Taking a deep breath, coughing, or sneezing worsens the pain. A soft diet is not necessarily indicated and there is no need for the patient to avoid speaking. Ambulation has multiple benefits, but pain management is not among them.

The nurse is assessing a patient who has a 35 pack-year history of cigarette smoking. In light of this known risk factor for lung cancer, what statement should prompt the nurse to refer the patient for further assessment? A) Lately, I have this cough that just never seems to go away. B) I find that I dont have nearly the stamina that I used to. C) I seem to get nearly every cold and flu that goes around my workplace. D) I never used to have any allergies, but now I think Im developing allergies to dust and pet hair.

Ans: A Feedback: The most frequent symptom of lung cancer is cough or change in a chronic cough. People frequently ignore this symptom and attribute it to smoking or a respiratory infection. A new onset of allergies, frequent respiratory infections and fatigue are not characteristic early signs of lung cancer.

A critical-care nurse is caring for a patient diagnosed with pneumonia as a surgical complication. The nurses assessment reveals that the patient has an increased work of breathing due to copious tracheobronchial secretions. What should the nurse encourage the patient to do? A) Increase oral fluids unless contraindicated. B) Call the nurse for oral suctioning, as needed. C) Lie in a low Fowlers or supine position. D) Increase activity.

Ans: A Feedback: The nurse should encourage hydration because adequate hydration thins and loosens pulmonary secretions. Oral suctioning is not sufficiently deep to remove tracheobronchial secretions. The patient should have the head of the bed raised, and rest should be promoted to avoid exacerbation of symptoms.

The public health nurse is administering Mantoux tests to children who are being registered for kindergarten in the community. How should the nurse administer this test? A) Administer intradermal injections into the childrens' inner forearms. B) Administer intramuscular injections into each child's vastus lateralis. C) Administer a subcutaneous injection into each child's umbilical area. D) Administer a subcutaneous injection at a 45-degree angle into each childs deltoid.

Ans: A Feedback: The purified protein derivative (PPD) is always injected into the intradermal layer of the inner aspect of the forearm. The subcutaneous and intramuscular routes are not utilized.

6. The nurse is caring for a patient who has returned to the unit following a bronchoscopy. The patient is asking for something to drink. Which criterion will determine when the nurse should allow the patient to drink fluids? A) Presence of a cough and gag reflex B) Absence of nausea C) Ability to demonstrate deep inspiration D) Oxygen saturation of 92%

Ans: A Feedback: After the procedure, it is important that the patient takes nothing by mouth until the cough reflex returns because the preoperative sedation and local anesthesia impair the protective laryngeal reflex and swallowing for several hours. Deep inspiration, adequate oxygen saturation levels, and absence of nausea do not indicate that oral intake is safe from the risk of aspiration

40. The nurse has assessed a patients family history for three generations. The presence of which respiratory disease would justify this type of assessment? A) Asthma B) Obstructive sleep apnea C) Community-acquired pneumonia D) Pulmonary edema

Ans: A Feedback: Asthma is a respiratory illness that has genetic factors. Sleep apnea, pneumonia, and pulmonary edema lack genetic risk factors.

39. A patient on the medical unit has told the nurse that he is experiencing significant dyspnea, despite that he has not recently performed any physical activity. What assessment question should the nurse ask the patient while preparing to perform a physical assessment? A) On a scale from 1 to 10, how bad would rate your shortness of breath? B) When was the last time you ate or drank anything? C) Are you feeling any nausea along with your shortness of breath? D) Do you think that some medication might help you catch your breath?

Ans: A Feedback: Gauging the severity of the patients dyspnea is an important part of the nursing process. Oral intake and nausea are much less important considerations. The nurse must perform assessment prior to interventions such as providing medication.

14. The medical nurse who works on a pulmonology unit is aware that several respiratory conditions can affect lung tissue compliance. The presence of what condition would lead to an increase in lung compliance? A) Emphysema B) Pulmonary fibrosis C) Pleural effusion D) Acute respiratory distress syndrome (ARDS)

Ans: A Feedback: High or increased compliance occurs if the lungs have lost their elasticity and the thorax is overdistended, in conditions such as emphysema. Conditions associated with decreased compliance include pneumothorax, hemothorax, pleural effusion, pulmonary edema, atelectasis, pulmonary fibrosis, and ARDS.

23. The nurse is performing a respiratory assessment of an adult patient and is attempting to distinguish between vesicular, bronchovesicular, and bronchial (tubular) breath sounds. The nurse should distinguish between these normal breath sounds on what basis? A) Their location over a specific area of the lung B) The volume of the sounds C) Whether they are heard on inspiration or expiration D) Whether or not they are continuous breath sounds

Ans: A Feedback: Normal breath sounds are distinguished by their location over a specific area of the lung; they are identified as vesicular, bronchovesicular, and bronchial (tubular) breath sounds. Normal breath sounds are heard on both inspiration and expiration, and are continuous. They are not distinguished solely on the basis of volume.

21. The clinic nurse is caring for a patient who has been diagnosed with emphysema and who has just had a pulmonary function test (PFT) ordered. The patient asks, What exactly is this test for? What would be the nurses best response? A) A PFT measures how much air moves in and out of your lungs when you breathe. B) A PFT measures how much energy you get from the oxygen you breathe. C) A PFT measures how elastic your lungs are. D) A PFT measures whether oxygen and carbon dioxide move between your lungs and your blood.

Ans: A Feedback: PFTs are routinely used in patients with chronic respiratory disorders. They are performed to assess respiratory function and to determine the extent of dysfunction. Such tests include measurements of lung volumes, ventilatory function, and the mechanics of breathing, diffusion, and gas exchange. Lung elasticity and diffusion can often be implied from PFTs, but they are not directly assessed. Energy obtained from respiration is not measured directly.

26. The nurse is caring for a patient with a lower respiratory tract infection. When planning a focused respiratory assessment, the nurse should know that this type of infection most often causes what? A) Impaired gas exchange B) Collapsed bronchial structures C) Necrosis of the alveoli D) Closed bronchial tree

Ans: A Feedback: The lower respiratory tract consists of the lungs, which contain the bronchial and alveolar structures needed for gas exchange. A lower respiratory tract infection does not collapse bronchial structures or close the bronchial tree. An infection does not cause necrosis of lung tissues.

4. The ED nurse is assessing a patient complaining of dyspnea. The nurse auscultates the patients chest and hears wheezing throughout the lung fields. What might this indicate? A) The patient has a narrowed airway. B) The patient has pneumonia. C) The patient needs physiotherapy. D) The patient has a hemothorax.

Ans: A Feedback: Wheezing is a high-pitched, musical sound that is often the major finding in a patient with bronchoconstriction or airway narrowing. Wheezing is not normally indicative of pneumonia or hemothorax. Wheezing does not indicate the need for physiotherapy.

36. A nurse educator is reviewing the implications of the oxyhemoglobin dissociation curve with regard to the case of a current patient. The patient currently has normal hemoglobin levels, but significantly decreased SaO2 and PaO2 levels. What is an implication of this physiological state? A) The patients tissue demands may be met, but she will be unable to respond to physiological stressors. B) The patients short-term oxygen needs will be met, but she will be unable to expel sufficient CO2 . C) The patient will experience tissue hypoxia with no sensation of shortness of breath or labored breathing. D) The patient will experience respiratory alkalosis with no ability to compensate.

Ans: A Feedback: With a normal hemoglobin level of 15 mg/dL and a PaO2 level of 40 mm Hg (SaO2 75%), there is adequate oxygen available for the tissues, but no reserve for physiological stresses that increase tissue oxygen demand. If a serious incident occurs (e.g., bronchospasm, aspiration, hypotension, or cardiac dysrhythmias) that reduces the intake of oxygen from the lungs, tissue hypoxia results.

The nurse is assessing a patient whose respiratory disease in characterized by chronic hyperinflation of the lungs. What would the nurse most likely assess in this patient? A) Signs of oxygen toxicity B) Chronic chest pain C) A barrel chest D) Long, thin fingers

Ans: A barrel chest Feedback: In COPD patients with a primary emphysematous component, chronic hyperinflation leads to the barrel chest thorax configuration. The nurse most likely would not assess chest pain or long, thin fingers; these are not characteristic of emphysema. The patient would not show signs of oxygen toxicity unless he or she received excess supplementary oxygen.

A nurse educator is reviewing the indications for chest drainage systems with a group of medical nurses. What indications should the nurses identify? Select all that apply. A) Post thoracotomy B) Spontaneous pneumothorax C) Need for postural drainage D) Chest trauma resulting in pneumothorax E) Pleurisy

Ans: A, B, D Feedback: Chest drainage systems are used in treatment of spontaneous pneumothorax and trauma resulting in pneumothorax. Postural drainage and pleurisy are not criteria for use of a chest drainage system.

The rehabilitation nurse is working closely with a patient who has a new orthosis following a knee injury. What are the nurse's responsibilities to this patient? Select all that apply. A) Help the patient learn to apply and remove the orthosis. B) Teach the patient how to care for the skin that comes in contact with the orthosis. C) Assist in the initial fitting of the orthosis. D) Assist the patient in learning how to move the affected body part correctly. E) Collaborate with the physical therapist to set goals for care.

Ans: A, B, D, E Feedback: In addition to learning how to apply and remove the orthosis and maneuver the affected body part correctly, patients must learn how to properly care for the skin that comes in contact with the appliance. Skin problems or pressure ulcers may develop if the device is applied too tightly or too loosely or if it is adjusted improperly. Nurses do not perform the initial fitting of orthoses.

The occupational health nurse is assessing new employees at a company. What would be important to assess in employees with a potential occupational respiratory exposure to a toxin? Select all that apply. A) Time frame of exposure B) Type of respiratory protection used C) Immunization status D) Breath sounds E) Intensity of exposure

Ans: A, B, D, E Feedback: Key aspects of any assessment of patients with a potential occupational respiratory history include job and job activities, exposure levels, general hygiene, time frame of exposure, effectiveness of respiratory protection used, and direct versus indirect exposures. The patients current respiratory status would also be a priority. Occupational lung hazards are not normally influenced by immunizations.

A school nurse is caring for a 10-year-old girl who is having an asthma attack. What is the preferred intervention to alleviate this client's airflow obstruction? A) Administer corticosteroids by metered dose inhaler B) Administer inhaled anticholinergics C) Administer an inhaled beta-adrenergic agonist D) Utilize a peak flow monitoring device

Ans: Administer an inhaled beta-adrenergic agonist Feedback: Systemic corticosteroids may be necessary to decrease airway inflammation in patients who fail to respond to inhaled beta-adrenergic medication. A peak flow device will not resolve short-term shortness of breath.

A nurse is assessing a patient who is suspected of having bronchiectasis. The nurse should consider which of the following potential causes? Select all that apply. A) Pulmonary hypertension B) Airway obstruction C) Pulmonary infections D) Genetic disorders E) Atelectasis

Ans: Airway obstruction, Pulmonary infections, Genetic disorders B, C, D Feedback: Bronchiectasis is a chronic, irreversible dilation of the bronchi and bronchioles. Under the new definition of COPD, it is considered a disease process separate from COPD. Bronchiectasis may be caused by a variety of conditions, including airway obstruction, diffuse airway injury, pulmonary infections and obstruction of the bronchus or complications of long-term pulmonary infections, or genetic disorders such as cystic fibrosis. Bronchiectasis is not caused by pulmonary hypertension or atelectasis.

A nurse is caring for a patient who has been hospitalized with an acute asthma exacerbation. What drugs should the nurse expect to be ordered for this patient to gain underlying control of persistent asthma? A) Rescue inhalers B) Anti-inflammatory drugs C) Antibiotics D) Antitussives

Ans: Anti-inflammatory drugs Feedback: Because the underlying pathology of asthma is inflammation, control of persistent asthma is accomplished primarily with regular use of anti-inflammatory medications. Rescue inhalers, antibiotics, and antitussives do not aid in the first-line control of persistent asthma.

An interdisciplinary team is planning the care of a patient with bronchiectasis. What aspects of care should the nurse anticipate? Select all that apply. A) Occupational therapy B) Antimicrobial therapy C) Positive pressure isolation D) Chest physiotherapy E) Smoking cessation

Ans: Antimicrobial therapy, Chest physiotherapy, Smoking cessation Feedback: Chest physiotherapy, antibiotics, and smoking cessation are cornerstones of the care of patients with bronchiectasis. Occupational therapy and isolation are not normally indicated.

A nurse is preparing to perform an admission assessment on a patient with COPD. It is most important for the nurse to review which of the following? A) Social work assessment B) Insurance coverage C) Chloride levels D) Available diagnostic tests

Ans: Available diagnostic tests Feedback: In addition to the patient's history, the nurse reviews the results of available diagnostic tests. Social work assessment is not a priority for the majority of patients. Chloride levels are relevant to CF, not COPD. Insurance coverage is not normally the domain of the nurse.

The nurse is caring for a patient who is experiencing mild shortness of breath during the immediate postoperative period, with oxygen saturation readings between 89% and 91%. What method of oxygen delivery is most appropriate for the patients needs? A) Non-rebreathing mask B) Nasal cannula C) Simple mask D) Partial-rebreathing mask

Ans: B Feedback: A nasal cannula is used when the patient requires a low to medium concentration of oxygen for which precise accuracy is not essential. The Venturi mask is used primarily for patients with COPD because it can accurately provide an appropriate level of supplemental oxygen, thus avoiding the risk of suppressing the hypoxic drive. The patient's respiratory status does not require a partial- or non-rebreathing mask.

The acute medical nurse is preparing to wean a patient from the ventilator. Which assessment parameter is most important for the nurse to assess? A) Fluid intake for the last 24 hours B) Baseline arterial blood gas (ABG) levels C) Prior outcomes of weaning D) Electrocardiogram (ECG) results

Ans: B Feedback: Before weaning a patient from mechanical ventilation, it is most important to have baseline ABG levels. During the weaning process, ABG levels will be checked to assess how the patient is tolerating the procedure. Other assessment parameters are relevant, but less critical. Measuring fluid volume intake and output is always important when a patient is being mechanically ventilated. Prior attempts at weaning and ECG results are documented on the patient's record, and the nurse can refer to them before the weaning process begins.

The nurse is caring for a client with an endotracheal tube who is on a ventilator. When assessing the client, the nurse knows to maintain what cuff pressure to maintain appropriate pressure on the tracheal wall? A) Between 10 and 15 mm Hg B) Between 15 and 20 mm Hg C) Between 20 and 25 mm Hg D) Between 25 and 30 mm Hg

Ans: B Feedback: Complications can occur from pressure exerted by the cuff on the tracheal wall. Cuff pressures should be maintained between 15 and 20 mm Hg.

A patient is being admitted to the preoperative holding area for a thoracotomy. Preoperative teaching includes what? A) Correct use of a ventilator B) Correct use of incentive spirometry C) Correct use of a mini-nebulizer D) Correct technique for rhythmic breathing

Ans: B Feedback: Instruction in the use of incentive spirometry begins before surgery to familiarize the patient with its correct use. You do not teach a patient the use of a ventilator; you explain that he may be on a ventilator to help him breathe. Rhythmic breathing and mini-nebulizers are unnecessary.

While caring for a patient with an endotracheal tube, the nurses recognizes that suctioning is required how often? A) Every 2 hours when the patient is awake B) When adventitious breath sounds are auscultated C) When there is a need to prevent the patient from coughing D) When the nurse needs to stimulate the cough reflex

Ans: B Feedback: It is usually necessary to suction the patient's secretions because of the decreased effectiveness of the cough mechanism. Tracheal suctioning is performed when adventitious breath sounds are detected or whenever secretions are present. Unnecessary suctioning, such as scheduling every 2 hours, can initiate bronchospasm and cause trauma to the tracheal mucosa.

The nurse is caring for a patient with a severe nosebleed. The physician inserts a nasal sponge and tells the patient it may have to remain in place up to 6 days before it is removed. The nurse should identify that this patient is at increased risk for what? A) Viral sinusitis B) Toxic shock syndrome C) Pharyngitis D) Adenoiditis

Ans: B Feedback: A compressed nasal sponge may be used. Once the sponge becomes saturated with blood or is moistened with a small amount of saline, it will expand and produce tamponade to halt the bleeding. The packing may remain in place for 48 hours or up to 5 or 6 days if necessary to control bleeding. Antibiotics may be prescribed because of the risk of iatrogenic sinusitis and toxic shock syndrome.

A patient is being transferred from a rehabilitation setting to a long-term care facility. During this process, the nurse has utilized the referral system? Using this system achieves what goal of the patient's care? A) Minimizing costs of the patient's care B) Maintaining continuity of the patient's care C) Maintain the nursing care plan between diverse sites D) Keeping the primary care provider informed

Ans: B Feedback: A referral system maintains continuity of care when the patient is transferred to the home or to a long-term care facility. The interests of cost and of keeping the primary care provider informed are not primary. The nursing plan is likely to differ between sites.

What would the critical care nurse recognize as a condition that may indicate a patient's need to have a tracheostomy? A) A patient has a respiratory rate of 10 breaths per minute. B) A patient requires permanent ventilation. C) A patient exhibits symptoms of dyspnea. D) A patient has respiratory acidosis.

Ans: B Feedback: A tracheostomy permits long-term use of mechanical ventilation to prevent aspiration of oral and gastric secretions in the unconscious or paralyzed patient. Indications for a tracheostomy do not include a respiratory rate of 10 breaths per minute, symptoms of dyspnea, or respiratory acidosis.

A mother calls the clinic asking for a prescription for Amoxicillin for her 2-year-old son who has what the nurse suspects to be viral rhinitis. What should the nurse explain to this mother? A) I will relay your request promptly to the doctor, but I suspect that she wont get back to you if its a cold. B) Ill certainly inform the doctor, but if it is a cold, antibiotics wont be used because they do not affect the virus. C) Ill phone in the prescription for you since it can be prescribed by the pharmacist. D) Amoxicillin is not likely the best antibiotic, but Ill call in the right prescription for you.

Ans: B Feedback: Antimicrobial agents (antibiotics) should not be used because they do not affect the virus or reduce the incidence of bacterial complications. In addition, their inappropriate use has been implicated in development of organisms resistant to therapy. It would be inappropriate to tell the patient that the physician will not respond to her request.

The nurse is performing the health interview of a patient with chronic rhinosinusitis who experiences frequent nose bleeds. The nurse asks the patient about her current medication regimen. Which medication would put the patient at a higher risk for recurrent epistaxis? A) Afrin B) Beconase C) Sinustop Pro D) Singulair

Ans: B Feedback: Beconase should be avoided in patients with recurrent epistaxis, glaucoma, and cataracts. Sinustop Pro and Afrin are pseudoephedrine and do not have a side effect of epistaxis. Singulair is a bronchodilator and does not have epistaxis as a side effect.

A patient states that her family has had several colds during this winter and spring despite their commitment to handwashing. The high communicability of the common cold is attributable to what factor? A) Cold viruses are increasingly resistant to common antibiotics. B) The virus is shed for 2 days prior to the emergence of symptoms. C) A genetic predisposition to viral rhinitis has recently been identified. D) Overuse of OTC cold remedies creates a rebound susceptibility to future colds.

Ans: B Feedback: Colds are highly contagious because virus is shed for about 2 days before the symptoms appear and during the first part of the symptomatic phase. Antibiotic resistance is not relevant to viral illnesses and OTC medications do not have a rebound effect. Genetic factors do not exist.

The nurse is conducting a presurgical interview for a patient with laryngeal cancer. The patient states that he drinks approximately six to eight shots of vodka per day. It is imperative that the nurse inform the surgical team so the patient can be assessed for what? A) Increased risk for infection B) Delirium tremens C) Depression D) Nonadherence to postoperative care

Ans: B Feedback: Considering the known risk factors for cancer of the larynx, it is essential to assess the patients history of alcohol intake. Infection is a risk in the postoperative period, but not an appropriate answer based on the patients history. Depression and nonadherence are risks in the postoperative phase, but would not be critical short-term assessments.

The nurse recognizes that aspiration is a potential complication of a laryngectomy. How should the nurse best manage this risk? A) Facilitate total parenteral nutrition (TPN). B) Keep a complete suction setup at the bedside. C) Feed the patient several small meals daily. D) Refer the patient for occupational therapy.

Ans: B Feedback: Due to the risk for aspiration, the nurse keeps a suction setup available in the hospital and instructs the family to do so at home for use if needed. TPN is not indicated and small meals do not necessarily reduce the risk of aspiration. Physical therapists do not address swallowing ability.

You are the rehabilitation nurse caring for a 25-year-old patient who suffered extensive injuries in a motorcycle accident. During each patient contact, what action should you perform most frequently? A) Complete a physical assessment. B) Evaluate the patient's positioning. C) Plan nursing interventions. D) Assist the patient to ambulate.

Ans: B Feedback: During each patient contact, the nurse evaluates the patient's position and assists the patient to achieve and maintain proper positioning and alignment. The nurse does not complete a physical assessment during each patient contact. Similarly, the nurse does not plan nursing interventions or assist the patient to ambulate each time the nurse has contact with the patient.

The perioperative nurse has admitted a patient who has just underwent a tonsillectomy. The nurses postoperative assessment should prioritize which of the following potential complications of this surgery? A) Difficulty ambulating B) Hemorrhage C) Infrequent swallowing D) Bradycardia

Ans: B Feedback: Hemorrhage is a potential complication of a tonsillectomy. Increased pulse, fever, and restlessness may indicate a postoperative hemorrhage. Difficulty ambulating and bradycardia are not common complications in a patient after a tonsillectomy. Infrequent swallowing does not indicate hemorrhage; frequent swallowing does.

While assessing a newly admitted patient you note the following: impaired coordination, decreased muscle strength, limited range of motion, and reluctance to move. What nursing diagnosis do these signs and symptoms most clearly suggest? A) Ineffective health maintenance B) Impaired physical mobility C) Disturbed sensory perception: Kinesthetic D) Ineffective role performance

Ans: B Feedback: Impaired physical mobility is a limitation of physical movement that is identified by the characteristics found in this patient. The other listed diagnoses are not directly suggested by the noted assessment findings.

The nurse is providing care for an older adult man whose diagnosis of dementia has recently led to urinary incontinence. When planning this patient's care, what intervention should the nurse avoid? A) Scheduled toileting B) Indwelling catheter C) External condom catheter D) Incontinence pads

Ans: B Feedback: Indwelling catheters are avoided if at all possible because of the high incidence of urinary tract infections with their use. Intermittent self-catheterization is an appropriate alternative for managing reflex incontinence, urinary retention, and overflow incontinence related to an overdistended bladder. External catheters (condom catheters) and leg bags to collect spontaneous voiding are useful for male patients with reflex or total incontinence. Incontinence pads should be used as a last resort because they only manage, rather than solve, the incontinence.

A 74-year-old woman experienced a cerebrovascular accident 6 weeks ago and is currently receiving inpatient rehabilitation. You are coaching the patient to contract and relax her muscles while keeping her extremity in a fixed position. Which type of exercise is the patient performing? A) Passive B) Isometric C) Resistive D) Abduction

Ans: B Feedback: Isometric exercises are those in which there is alternating contraction and relaxation of a muscle while keeping the part in a fixed position. This exercise is performed by the patient. Passive exercises are carried out by the therapist or the nurse without assistance from the patient. Resistive exercises are carried out by the patient working against resistance produced by either manual or mechanical means. Abduction is movement of a part away from the midline of the body.

As a member of the rehabilitation team, the nurse is conscious of the need to perform the nursing role in collaboration with the other members of the team. Which of the following variables has the greatest bearing on the nurse's choice of actions and interventions during rehabilitative care? A) The skills of the other members of the team B) The circumstances of the patient C) The desires of the patient's family D) The nurse's education and experience level

Ans: B Feedback: Nurses assume an equal or, depending on the circumstances of the patient, a more critical role than other members of the health care team in the rehabilitation process. The nurse's role on the rehabilitation team does not depend primarily on other members of the team, the family's desires, or the nurse's education level.

The nurse is caring for a patient with chronic obstructive pulmonary disease (COPD). The patient has been receiving high-flow oxygen therapy for an extended time. What symptoms should the nurse anticipate if the patient were experiencing oxygen toxicity? A) Bradycardia and frontal headache B) Dyspnea and substernal pain C) Peripheral cyanosis and restlessness D) Hypotension and tachycardia

Ans: B Feedback: Oxygen toxicity can occur when patients receive too high a concentration of oxygen for an extended period. Symptoms of oxygen toxicity include dyspnea, substernal pain, restlessness, fatigue, and progressive respiratory difficulty. Bradycardia, frontal headache, cyanosis, hypotension, and tachycardia are not symptoms of oxygen toxicity.

You are admitting a patient into your rehabilitation unit after an industrial accident. The patient's nursing diagnoses include disturbed sensory perception and you assess that he has decreased strength and dexterity. You know that this patient may need what to accomplish self-care? A) Advice from his family B) Appropriate assistive devices C) A personal health care aide D) An assisted-living environment

Ans: B Feedback: Patients with impaired mobility, sensation, strength, or dexterity may need to use assistive devices to accomplish self-care. An assisted-living environment is less common than the use of assistive devices. Family involvement is imperative, but this may or may not take the form of advice. A healthcare aide is not needed by most patients.

The physician has ordered continuous positive airway pressure (CPAP) with the delivery of a patients high-flow oxygen therapy. The patient asks the nurse what the benefit of CPAP is. What would be the nurses best response? A) CPAP allows a higher percentage of oxygen to be safely used. B) CPAP allows a lower percentage of oxygen to be used with a similar effect. C) CPAP allows for greater humidification of the oxygen that is administered. D) CPAP allows for the elimination of bacterial growth in oxygen delivery systems.

Ans: B Feedback: Prevention of oxygen toxicity is achieved by using oxygen only as prescribed. Often, positive end- expiratory pressure (PEEP) or CPAP is used with oxygen therapy to reverse or prevent microatelectasis, thus allowing a lower percentage of oxygen to be used. Oxygen is moistened by passing through a humidification system. Changing the tubing on the oxygen therapy equipment is the best technique for controlling bacterial growth.

The nurse is caring for a patient who is scheduled to have a thoracotomy. When planning preoperative teaching, what information should the nurse communicate to the patient? A) How to milk the chest tubing B) How to splint the incision when coughing C) How to take prophylactic antibiotics correctly D) How to manage the need for fluid restriction

Ans: B Feedback: Prior to thoracotomy, the nurse educates the patient about how to splint the incision with the hands, a pillow, or a folded towel. The patient is not taught how to milk the chest tubing because this is performed by the nurse. Prophylactic antibiotics are not normally used and fluid restriction is not indicated following thoracotomy.

A patient has completed the acute treatment phase of care following a stroke and the patient will now begin rehabilitation. What should the nurse identify as the major goal of the rehabilitative process? A) To provide 24-hour, collaborative care for the patient B) To restore the patient's ability to function independently C) To minimize the patient's time spent in acute care settings D) To promote rapport between caregivers and the patient

Ans: B Feedback: The goal of rehabilitation is to restore the patient's ability to function independently or at a preillness or preinjury level of functioning as quickly as possible. Twenty-four hour care, rapport, and minimizing time in acute care are not central goals of rehabilitation.

A nurse has been asked to become involved in the care of an adult patient in his fifties who has experienced a new onset of urinary incontinence. During what aspect of the assessment should the nurse explore physiologic risk factors for elimination problems? A) Physical assessment B) Health history C) Genetic history D) Initial assessment

Ans: B Feedback: The health history is used to explore bladder and bowel function, symptoms associated with dysfunction, physiologic risk factors for elimination problems, perception of micturition (urination or voiding) and defecation cues, and functional toileting abilities. Elimination problems are not explored in the other listed aspects of assessment.

You are the nurse caring for a female patient who developed a pressure ulcer as a result of decreased mobility. The nurse on the shift before you has provided patient teaching about pressure ulcers and healing promotion. You assess that the patient has understood the teaching by observing what? A) Patient performs range-of-motion exercises. B) Patient avoids placing her body weight on the healing site. C) Patient elevates her body parts that are susceptible to edema. D) Patient demonstrates the technique for massaging the wound site.

Ans: B Feedback: The major goals of pressure ulcer treatment may include relief of pressure, improved mobility, improved sensory perception, improved tissue perfusion, improved nutritional status, minimized friction and shear forces, dry surfaces in contact with skin, and healing of pressure ulcer, if present. The other options do not demonstrate the achievement of the goal of the patient teaching.

The nurse is doing discharge teaching in the ED with a patient who had a nosebleed. What should the nurse include in the discharge teaching of this patient? A) Avoid blowing the nose for the next 45 minutes. B) In case of recurrence, apply direct pressure for 15 minutes. C) Do not take aspirin for the next 2 weeks. D) Seek immediate medical attention if the nosebleed recurs.

Ans: B Feedback: The nurse explains how to apply direct pressure to the nose with the thumb and the index finger for 15 minutes in case of a recurrent nosebleed. If recurrent bleeding cannot be stopped, the patient is instructed to seek additional medical attention. ASA is not contraindicated in most cases and the patient should avoiding blowing the nose for an extended period of time, not just 45 minutes.

An interdisciplinary team has been working collaboratively to improve the health outcomes of a young adult who suffered a spinal cord injury in a workplace accident. Which member of the rehabilitation team is the one who determines the final outcome of the process? A) Most-responsible nurse B) Patient C) Patient's family D) Primary care physician

Ans: B Feedback: The patient is the key member of the rehabilitation team. He or she is the focus of the team effort and the one who determines the final outcomes of the process. The nurse, family, and doctor are part of the rehabilitation team but do not determine the final outcome.

The nurse is providing care for a patient who has just been admitted to the postsurgical unit following a laryngectomy. What assessment should the nurse prioritize? A) The patients swallowing ability B) The patients airway patency C) The patients carotid pulses D) Signs and symptoms of infection

Ans: B Feedback: The patient with a laryngectomy is a risk for airway occlusion and respiratory distress. As in all nursing situations, assessment of the airway is a priority over other potential complications and assessment parameters.

The nurse has explained to the patient that after his thoracotomy, it will be important to adhere to a coughing schedule. The patient is concerned about being in too much pain to be able to cough. What would be an appropriate nursing intervention for this client? A) Teach him postural drainage. B) Teach him how to perform huffing. C) Teach him to use a mini-nebulizer. D) Teach him how to use a metered dose inhaler.

Ans: B Feedback: The technique of huffing may be helpful for the patient with diminished expiratory flow rates or for the patient who refuses to cough because of severe pain. Huffing is the expulsion of air through an open glottis. Inhalers, nebulizers, and postural drainage are not substitutes for performing coughing exercises.

A nurse is giving a talk to a local community group whose members advocate for disabled members of the community. The group is interested in emerging trends that are impacting the care of people who are disabled in the community. The nurse should describe an increasing focus on what aspect of care? A) Extended rehabilitation care B) Independent living C) Acute-care center treatment D) State institutions that provide care for life

Ans: B Feedback: There is a growing trend toward independent living for patients who are severely disabled, either alone or in groups. The goal is integration into the community. The nurse would be sure to mention this fact when talking to a local community group. The nurse would not describe extended rehabilitation care, acute-care center treatment, or state institutions because these are not increasing in importance.

The nurse is caring for a patient who has just been diagnosed with chronic rhinosinusitis. While being admitted to the clinic, the patient asks, Will this chronic infection hurt my new kidney? What should the nurse know about chronic rhinosinusitis in patients who have had a transplant? A) The patient will have exaggerated symptoms of rhinosinusitis due to immunosuppression. B) Taking immunosuppressive drugs can contribute to chronic rhinosinusitis. C) Chronic rhinosinusitis can damage the transplanted organ. D) Immunosuppressive drugs can cause organ rejection.

Ans: B Feedback: URIs, specifically chronic rhinosinusitis and recurrent acute rhinosinusitis, may be linked to primary or secondary immune deficiency or treatment with immunosuppressive therapy (i.e., for cancer or organ transplantation). Typical symptoms may be blunted or absent due to immunosuppression. No evidence indicates damage to the transplanted organ due to chronic rhinosinusitis. Immunosuppressive drugs do not cause organ rejection.

A patient has just been diagnosed with squamous cell carcinoma of the neck. While the nurse is doing health education, the patient asks, Does this kind of cancer tend to spread to other parts of the body? What is the nurses best response? A) In many cases, this type of cancer spreads to other parts of the body. B) This cancer usually does not spread to distant sites in the body. C) You will have to speak to your oncologist about that. D) Squamous cell carcinoma is nothing to be concerned about, so try to focus on your health.

Ans: B The incidence of distant metastasis with squamous cell carcinoma of the head and neck (including larynx cancer) is relatively low. The patients prognosis is determined by the oncologist, but the patient has asked a general question and it would be inappropriate to refuse a response. The nurse must not downplay the patients concerns.

A perioperative nurse is caring for a postoperative patient. The patient has a shallow respiratory pattern and is reluctant to cough or to begin mobilizing. The nurse should address the patients increased risk for what complication? A) Acute respiratory distress syndrome (ARDS) B) Atelectasis C) Aspiration D) Pulmonary embolism

Ans: B Feedback: A shallow, monotonous respiratory pattern coupled with immobility places the patient at an increased risk of developing atelectasis. These specific factors are less likely to result in pulmonary embolism or aspiration. ARDS involves an exaggerated inflammatory response and does not normally result from factors such as immobility and shallow breathing.

A firefighter was trapped in a fire and is admitted to the ICU for smoke inhalation. After 12 hours, the firefighter is exhibiting signs of ARDS and is intubated. What other supportive measures are initiated in a patient with ARDS? A) Psychological counseling B) Nutritional support C) High-protein oral diet D) Occupational therapy

Ans: B Feedback: Aggressive, supportive care must be provided to compensate for the severe respiratory dysfunction. This supportive therapy almost always includes intubation and mechanical ventilation. In addition, circulatory support, adequate fluid volume, and nutritional support are important. Oral intake is contraindicated by intubation. Counseling and occupational therapy would not be priorities during the acute stage of ARDS.

The nurse is caring for a patient suspected of having ARDS. What is the most likely diagnostic test ordered in the early stages of this disease to differentiate the patients symptoms from those of a cardiac etiology? A) Carboxyhemoglobin level B) Brain natriuretic peptide (BNP) level C) C-reactive protein (CRP) level D) Complete blood count

Ans: B Feedback: Common diagnostic tests performed for patients with potential ARDS include plasma brain natriuretic peptide (BNP) levels, echocardiography, and pulmonary artery catheterization. The BNP level is helpful in distinguishing ARDS from cardiogenic pulmonary edema. The carboxyhemoglobin level will be increased in a client with an inhalation injury, which commonly progresses into ARDS. CRP and CBC levels do not help differentiate from a cardiac problem.

A patient with thoracic trauma is admitted to the ICU. The nurse notes the patients chest and neck are swollen and there is a crackling sensation when palpated. The nurse consequently identifies the presence of subcutaneous emphysema. If this condition becomes severe and threatens airway patency, what intervention is indicated? A) A chest tube B) A tracheostomy C) An endotracheal tube D) A feeding tube

Ans: B Feedback: In severe cases in which there is widespread subcutaneous emphysema, a tracheostomy is indicated if airway patency is threatened by pressure of the trapped air on the trachea. The other listed tubes would neither resolve the subcutaneous emphysema nor the consequent airway constriction.

A hospital has been the site of an increased incidence of hospital-acquired pneumonia (HAP). What is an important measure for the prevention of HAP? A) Administration of prophylactic antibiotics B) Administration of pneumococcal vaccine to vulnerable individuals C) Obtaining culture and sensitivity swabs from all newly admitted patients D) Administration of antiretroviral medications to patients over age 65

Ans: B Feedback: Pneumococcal vaccination reduces the incidence of pneumonia, hospitalizations for cardiac conditions, and deaths in the general older adult population. A onetime vaccination of pneumococcal polysaccharide vaccine (PPSV) is recommended for all patients 65 years of age or older and those with chronic diseases. Antibiotics are not given on a preventative basis and antiretroviral medications do not affect the most common causative microorganisms. Culture and sensitivity testing by swabbing is not performed for pneumonia since the microorganisms are found in sputum.

A patient is receiving thrombolytic therapy for the treatment of pulmonary emboli. What is the best way for the nurse to assess the patients oxygenation status at the bedside? A) Obtain serial ABG samples. B) Monitor pulse oximetry readings. C) Test pulmonary function. D) Monitor incentive spirometry volumes.

Ans: B Feedback: The nurse assesses the patient with pulmonary emboli frequently for signs of hypoxemia and monitors the pulse oximetry values to evaluate the effectiveness of the oxygen therapy. ABGs are accurate indicators of oxygenation status, but are not analyzed at the bedside. PFTs and incentive spirometry volumes do not accurately reveal oxygenation status.

A patient who involved in a workplace accident suffered a penetrating wound of the chest that led to acute respiratory failure. What goal of treatment should the care team prioritize when planning this patients care? A) Facilitation of long-term intubation B) Restoration of adequate gas exchange C) Attainment of effective coping D) Self-management of oxygen therapy

Ans: B Feedback: The objectives of treatment are to correct the underlying cause of respiratory failure and to restore adequate gas exchange in the lung. This is priority over coping and self-care. Long-term ventilation may or may not be indicated.

A new employee asks the occupational health nurse about measures to prevent inhalation exposure of the substances. Which statement by the nurse will decrease the patients exposure risk to toxic substances? A) Position a fan blowing on the toxic substances to prevent the substance from becoming stagnant in the air. B) Wear protective attire and devices when working with a toxic substance. C) Make sure that you keep your immunizations up to date to prevent respiratory diseases resulting from toxins. D) Always wear a disposable paper face mask when you are working with inhalable toxins.

Ans: B Feedback: When working with toxic substances, the employee must wear or use protective devices such as face masks, hoods, or industrial respirators. Immunizations do not confer protection from toxins and a paper mask is normally insufficient protection. Never position a fan directly blowing on the toxic substance as it will disperse the fumes throughout the area.

13. The nurse is assessing a newly admitted medical patient and notes there is a depression in the lower portion of the patients sternum. This patients health record should note the presence of what chest deformity? A) A barrel chest B) A funnel chest C) A pigeon chest D) Kyphoscoliosis

Ans: B Feedback: A funnel chest occurs when there is a depression in the lower portion of the sternum, and this may lead to compression of the heart and great vessels, resulting in murmurs. A barrel chest is characterized by an increase in the anteroposterior diameter of the thorax and is a result of overinflation of the lungs. A pigeon chest occurs as a result of displacement of the sternum and includes an increase in the anteroposterior diameter. Kyphoscoliosis, which is characterized by elevation of the scapula and a corresponding S-shaped spine, limits lung expansion within the thorax.

27. The nurse is performing a respiratory assessment of a patient who has been experiencing episodes of hypoxia. The nurse is aware that this is ultimately attributable to impaired gas exchange. On what factor does adequate gas exchange primarily depend? A) An appropriate perfusiondiffusion ratio B) An adequate ventilationperfusion ratio C) Adequate diffusion of gas in shunted blood D) Appropriate blood nitrogen concentration

Ans: B Feedback: Adequate gas exchange depends on an adequate ventilationperfusion ratio. There is no perfusiondiffusion ratio. Adequate gas exchange does not depend on the diffusion of gas in shunted blood or a particular concentration of nitrogen.

10. A patient with a decreased level of consciousness is in a recumbent position. How should the nurse best assess the lung fields for a patient in this position? A) Inform that physician that the patient is in a recumbent position and anticipate an order for a portable chest x-ray. B) Turn the patient to enable assessment of all the patients lung fields. C) Avoid turning the patient, and assess the accessible breath sounds from the anterior chest wall. D) Obtain a pulse oximetry reading, and, if the reading is low, reposition the patient and auscultate breath sounds

Ans: B Feedback: Assessment of the anterior and posterior lung fields is part of the nurses routine evaluation. If the patient is recumbent, it is essential to turn the patient to assess all lung fields so that dependent areas can be assessed for breath sounds, including the presence of normal breath sounds and adventitious sounds. Failure to examine the dependent areas of the lungs can result in missing significant findings. This makes the other given options unacceptable.

28. The nurse is caring for a patient with lung metastases who just underwent a mediastinotomy. What should be the focus of the nurses postprocedure care? A) Assisting with pulmonary function testing (PFT) B) Maintaining the patients chest tube C) Administering oral suction as needed D) Performing chest physiotherapy

Ans: B Feedback: Chest tube drainage is required after mediastinotomy. PFT, chest physiotherapy, and oral suctioning would all be contraindicated because of the patients unstable health status

3. The nurse is assessing a patient who frequently coughs after eating or drinking. How should the nurse best follow up this assessment finding? A) Obtain a sputum sample. B) Perform a swallowing assessment. C) Inspect the patients tongue and mouth. D) Assess the patients nutritional status.

Ans: B Feedback: Coughing after food intake may indicate aspiration of material into the tracheobronchial tree; a swallowing assessment is thus indicated. Obtaining a sputum sample is relevant in cases of suspected infection. The status of the patients tongue, mouth, and nutrition is not directly relevant to the problem of aspiration.

29. The nurse is caring for a patient who has a pleural effusion and who underwent a thoracoscopic procedure earlier in the morning. The nurse should prioritize assessment for which of the following? A) Sputum production B) Shortness of breath C) Throat discomfort D) Epistaxis

Ans: B Feedback: Follow-up care in the health care facility and at home involves monitoring the patient for shortness of breath (which might indicate a pneumothorax). All of the listed options are relevant assessment findings, but shortness of breath is the most serious complication.

24. A patient has been diagnosed with pulmonary hypertension, in which the capillaries in the alveoli are squeezed excessively. The nurse should recognize a disturbance in what aspect of normal respiratory function? A) Acidbase balance B) Perfusion C) Diffusion D) Ventilation

Ans: B Feedback: Perfusion is influenced by alveolar pressure. The pulmonary capillaries are sandwiched between adjacent alveoli and, if the alveolar pressure is sufficiently high, the capillaries are squeezed. This does not constitute a disturbance in ventilation (air movement), diffusion (gas exchange), or acidbase balance

33. A sputum study has been ordered for a patient who has developed coarse chest crackles and a fever. At what time should the nurse best collect the sample? A) Immediately after a meal B) First thing in the morning C) At bedtime D) After a period of exercise

Ans: B Feedback: Sputum samples ideally are obtained early in the morning before the patient has had anything to eat or drink.

12. The nurse doing rounds at the beginning of a shift notices a sputum specimen in a container sitting on the bedside table in a patients room. The nurse asks the patient when he produced the sputum specimen and he states that the specimen is about 4 hours old. What action should the nurse take? A) Immediately take the sputum specimen to the laboratory. B) Discard the specimen and assist the patient in obtaining another specimen. C) Refrigerate the sputum specimen and submit it once it is chilled. D) Add a small amount of normal saline to moisten the specimen.

Ans: B Feedback: Sputum samples should be submitted to the laboratory as soon as possible. Allowing the specimen to stand for several hours in a warm room results in the overgrowth of contaminated organisms and may make it difficult to identify the pathogenic organisms. Refrigeration of the sputum specimen and the addition of normal saline are not appropriate actions.

30. A gerontologic nurse is analyzing the data from a patients focused respiratory assessment. The nurse is aware that the amount of respiratory dead space increases with age. What is the effect of this physiological change? A) Increased diffusion of gases B) Decreased diffusion capacity for oxygen C) Decreased shunting of blood D) Increased ventilation

Ans: B Feedback: The amount of respiratory dead space increases with age. Combined with other changes, this results in a decreased diffusion capacity for oxygen with increasing age, producing lower oxygen levels in the arterial circulation. Decreased shunting and increased ventilation do not occur with age.

1. A patient is having her tonsils removed. The patient asks the nurse what function the tonsils normally serve. Which of the following would be the most accurate response? A) The tonsils separate your windpipe from your throat when you swallow. B) The tonsils help to guard the body from invasion of organisms. C) The tonsils make enzymes that you swallow and which aid with digestion. D) The tonsils help with regulating the airflow down into your lungs.

Ans: B Feedback: The tonsils, the adenoids, and other lymphoid tissue encircle the throat. These structures are important links in the chain of lymph nodes guarding the body from invasion of organisms entering the nose and throat. The tonsils do not aid digestion, separate the trachea from the esophagus, or regulate airflow to the bronchi

20. While assessing a patient who has pneumonia, the nurse has the patient repeat the letter E while the nurses auscultates. The nurse notes that the patients voice sounds are distorted and that the letter A is audible instead of the letter E. How should this finding be documented? A) Bronchophony B) Egophony C) Whispered pectoriloquy D) Sonorous wheezes

Ans: B Feedback: This finding would be documented as egophony, which can be best assessed by instructing the patient to repeat the letter E. The distortion produced by consolidation transforms the sound into a clearly heard A rather than E. Bronchophony describes vocal resonance that is more intense and clearer than normal. Whispered pectoriloquy is a very subtle finding that is heard only in the presence of rather dense consolidation of the lungs. Sound is so enhanced by the consolidated tissue that even whispered words are heard. Sonorous wheezes are not defined as a voice sound, but rather as a breath sound.

37. A medical patient rings her call bell and expresses alarm to the nurse, stating, Ive just coughed up this blood. That cant be good, can it? How can the nurse best determine whether the source of the blood was the patients lungs? A) Obtain a sample and test the pH of the blood, if possible. B) Try to see if the blood is frothy or mixed with mucus. C) Perform oral suctioning to see if blood is obtained. D) Swab the back of the patients throat to see if blood is present.

Ans: B Feedback: Though not definitive, blood from the lung is usually bright red, frothy, and mixed with sputum. Testing the pH of nonarterial blood samples is not common practice and would not provide important data. Similarly, oral suctioning and swabbing the patients mouth would not reveal the source.

The nurse is performing an assessment on a patient who has been diagnosed with cancer of the larynx. Part of the nurses assessment addresses the patients general state of nutrition. Which laboratory values would be assessed when determining the nutritional status of the patient? Select all that apply. A) White blood cell count B) Protein level C) Albumin level D) Platelet count E) Glucose level

Ans: B, C, E Feedback: The nurse also assesses the patients general state of nutrition, including height and weight and body mass index, and reviews laboratory values that assist in determining the patients nutritional status (albumin, protein, glucose, and electrolyte levels). The white blood cell count and the platelet count would not normally assist in determining the patients nutritional status.

A patient is brought to the ED by ambulance after a motor vehicle accident in which the patient received blunt trauma to the chest. The patient is in acute respiratory failure, is intubated, and is transferred to the ICU. What parameters of care should the nurse monitor most closely? Select all that apply. A) Coping B) Level of consciousness C) Oral intake D) Arterial blood gases E) Vital signs

Ans: B, D, E Feedback: Patients are usually treated in the ICU. The nurse assesses the patients respiratory status by monitoring the level of responsiveness, ABGs, pulse oximetry, and vital signs. Oral intake and coping are not immediate priorities during the acute stage of treatment, but would become more important later during recovery.

A nurse is completing a focused respiratory assessment of a child with asthma. What assessment finding is most closely associated with the characteristic signs and symptoms of asthma? A) Shallow respirations B) Increased anterior-posterior (A-P) diameter C) Bilateral wheezes D) Bradypnea

Ans: Bilateral wheezes Feedback: The three most common symptoms of asthma are cough, dyspnea, and wheezing. There may be generalized wheezing (the sound of airflow through narrowed airways), first on expiration and then, possibly, during inspiration as well. Respirations are not usually slow and the child's A-P diameter does not normally change.

A nurse is reviewing the pathophysiology of cystic fibrosis (CF) in anticipation of a new admission. The nurse should identify what characteristic aspects of CF? A) Alveolar mucus plugging, infection, and eventual bronchiectasis B) Bronchial mucus plugging, inflammation, and eventual bronchiectasis C) Atelectasis, infection, and eventual COPD D) Bronchial mucus plugging, infection, and eventual COPD

Ans: Bronchial mucus plugging, inflammation, and eventual bronchiectasis Feedback: The hallmark pathology of CF is bronchial mucus plugging, inflammation, and eventual bronchiectasis. Commonly, the bronchiectasis begins in the upper lobes and progresses to involve all lobes. Infection, atelectasis, and COPD are not hallmark pathologies of CF.

While assessing the patient, the nurse observes constant bubbling in the water-seal chamber of the patients closed chest-drainage system. What should the nurse conclude? A) The system is functioning normally. B) The patient has a pneumothorax. C) The system has an air leak. D) The chest tube is obstructed.

Ans: C Feedback: Constant bubbling in the chamber often indicates an air leak and requires immediate assessment and intervention. The patient with a pneumothorax will have intermittent bubbling in the water-seal chamber. If the tube is obstructed, the nurse should notice that the fluid has stopped fluctuating in the water-seal chamber.

The critical care nurse is precepting a new nurse on the unit. Together they are caring for a patient who has a tracheostomy tube and is receiving mechanical ventilation. What action should the critical care nurse recommend when caring for the cuff? A) Deflate the cuff overnight to prevent tracheal tissue trauma. B) Inflate the cuff to the highest possible pressure in order to prevent aspiration. C) Monitor the pressure in the cuff at least every 8 hours D) Keep the tracheostomy tube plugged at all times.

Ans: C Feedback: Cuff pressure must be monitored by the respiratory therapist or nurse at least every 8 hours by attaching a handheld pressure gauge to the pilot balloon of the tube or by using the minimal leak volume or minimal occlusion volume technique. Plugging is only used when weaning the patient from tracheal support. Deflating the cuff overnight would be unsafe and inappropriate. High cuff pressure can cause tissue trauma.

The nurse is performing nasotracheal suctioning on a medical patient and obtains copious amounts of secretions from the patients airway, even after inserting and withdrawing the catheter several times. How should the nurse proceed? A) Continue suctioning the patient until no more secretions are obtained. B) Perform chest physiotherapy rather than nasotracheal suctioning. C) Wait several minutes and then repeat suctioning. D) Perform postural drainage and then repeat suctioning.

Ans: C Feedback: If additional suctioning is needed, the nurse should withdraw the catheter to the back of the pharynx, reassure the patient, and oxygenate for several minutes before resuming suctioning. Chest physiotherapy and postural drainage are not necessarily indicated.

The nurse is preparing to discharge a patient after thoracotomy. The patient is going home on oxygen therapy and requires wound care. As a result, he will receive home care nursing. What should the nurse include in discharge teaching for this patient? A) Safe technique for self-suctioning of secretions B) Technique for performing postural drainage C) Correct and safe use of oxygen therapy equipment D) How to provide safe and effective tracheostomy care

Ans: C Feedback: Respiratory care and other treatment modalities (oxygen, incentive spirometry, chest physiotherapy [CPT], and oral, inhaled, or IV medications) may be continued at home. Therefore, the nurse needs to instruct the patient and family in their correct and safe use. The scenario does not indicate the patient needs help with suctioning, postural drainage, or tracheostomy care.

A patient with a severe exacerbation of COPD requires reliable and precise oxygen delivery. Which mask will the nurse expect the physician to order? A) Non-rebreather air mask B) Tracheostomy collar C) Venturi mask D) Face tent

Ans: C Feedback: The Venturi mask provides the most accurate method of oxygen delivery. Other methods of oxygen delivery include the aerosol mask, tracheostomy collar, and face tents, but these do not match the precision of a Venturi mask.

A nurse has performed tracheal suctioning on a patient who experienced increasing dyspnea prior to a procedure. When applying the nursing process, how can the nurse best evaluate the outcomes of this intervention? A) Determine whether the patient can now perform forced expiratory technique (FET). B) Percuss the patients lungs and thorax. C) Measure the patients oxygen saturation. D) Have the patient perform incentive spirometry.

Ans: C Feedback: The patient's response to suctioning is usually determined by performing chest auscultation and by measuring the patient's oxygen saturation. FET, incentive spirometry, and percussion are not normally used as evaluative techniques.

The home care nurse is assessing a patient who requires home oxygen therapy. What criterion indicates that an oxygen concentrator will best meet the needs of the patient in the home environment? A) The patient desires a low-maintenance oxygen delivery system that delivers oxygen flow rates up to 6 L/min. B) The patient requires a high-flow system for use with a tracheostomy collar. C) The patient desires a portable oxygen delivery system that can deliver 2 L/min. D) The patients respiratory status requires a system that provides an FiO2 of 65%.

Ans: C Feedback: The use of oxygen concentrators is another means of providing varying amounts of oxygen, especially in the home setting. They can deliver oxygen flows from 1 to 10 L/min and provide an FiO2 of about 40%. They require regular maintenance and are not used for high-flow applications. The patient desiring a portable oxygen delivery system of 2L/min will benefit from the use of an oxygen concentrator.

The nurse is preparing to suction a patient with an endotracheal tube. What should be the nurses first step in the suctioning process? A) Explain the suctioning procedure to the patient and reposition the patient. B) Turn on suction source at a pressure not exceeding 120 mm Hg. C) Assess the patients lung sounds and SAO2 via pulse oximeter. D) Perform hand hygiene and don nonsterile gloves, goggles, gown, and mask.

Ans: C Assessment data indicate the need for suctioning and allow the nurse to monitor the effect of suction on the patients level of oxygenation. Explaining the procedure would be the second step; performing hand hygiene is the third step, and turning on the suction source is the fourth step.

A home care nurse performs the initial visit to a patient who is soon being discharged from a rehabilitation facility. This initial visit is to assess what the patient can do and to see what he will need when discharged home. What does this help ensure for the patient? A) Social relationships B) Family assistance C) Continuity of care D) Realistic expectations

Ans: C Feedback: A home care nurse may visit the patient in the hospital, interview the patient and the family, and review the ADL sheet to learn which activities the patient can perform. This helps ensure that continuity of care is provided and that the patient does not regress, but instead maintains the independence gained while in the hospital or rehabilitation setting. This initial visit does not ensure social relationships, family assistance, or realistic expectations.

A patient is being treated for bacterial pharyngitis. Which of the following should the nurse recommend when promoting the patients nutrition during treatment? A) A 1.5 L/day fluid restriction B) A high-potassium, low-sodium diet C) A liquid or soft diet D) A high-protein diet

Ans: C Feedback: A liquid or soft diet is provided during the acute stage of the disease, depending on the patients appetite and the degree of discomfort that occurs with swallowing. The patient is encouraged to drink as much fluid as possible (at least 2 to 3 L/day). There is no need for increased potassium or protein intake.

You are the nurse caring for an elderly adult who is bedridden. What intervention would you include in the care plan that would most effectively prevent pressure ulcers? A) Turn and reposition the patient a minimum of every 8 hours. B) Vigorously massage lotion into bony prominences. C) Post a turning schedule at the patient's bedside and ensure staff adherence. D) Slide, rather than lift, the patient when turning.

Ans: C Feedback: A turning schedule with a signing sheet will help ensure that the patient gets turned and, thus, help prevent pressure ulcers. Turning should occur every 1 to 2 hours, not every 8 hours, for patients who are in bed for prolonged periods. The nurse should apply lotion to keep the skin moist, but should avoid vigorous massage, which could damage capillaries. When moving the patient, the nurse should lift, rather than slide, the patient to avoid shearing.

A patient has been transferred to a rehabilitative setting from an acute care unit. What is the most important reason for the nurse to begin a program for activities of daily living (ADLs) as soon as the patient is admitted to a rehabilitation facility? A) The ability to perform ADLs may be the key to dependence. B) The ability to perform ADLs is essential to living in a group home. C) The ability to perform ADLs may be the key to reentry into the community. D) The ability to perform ADLs is necessary to function in an assisted-living situation.

Ans: C Feedback: An ADL program is started as soon as the rehabilitation process begins because the ability to perform ADLs is frequently the key to independence, return to the home, and reentry into the community. ADLs are frequently the key to independence, not dependence. The ability to perform ADLs is not always a criterion for admission to a group home or assisted-living facility.

An elderly woman diagnosed with osteoarthritis has been referred for care. The patient has difficulty ambulating because of chronic pain. When creating a nursing care plan, what intervention may the nurse use to best promote the patient's mobility? A) Motivate the patient to walk in the afternoon rather than the morning. B) Encourage the patient to push through the pain in order to gain further mobility. C) Administer an analgesic as ordered to facilitate the patient's mobility. D) Have another person with osteoarthritis visit the patient.

Ans: C Feedback: At times, mobility is restricted because of pain, paralysis, loss of muscle strength, systemic disease, an immobilizing device (e.g., cast, brace), or prescribed limits to promote healing. If mobility is restricted because of pain, providing pain management through the administration of an analgesic will increase the patient's level of comfort during ambulation and allow the patient to ambulate. Motivating the patent or having another person with the same diagnosis visit is not an intervention that will help with mobility. The patient should not be encouraged to "push through the pain."

It is cold season and the school nurse been asked to provide an educational event for the parent teacher organization of the local elementary school. What should the nurse include in teaching about the treatment of pharyngitis? A) Pharyngitis is more common in children whose immunizations are not up to date. B) There are no effective, evidence-based treatments for pharyngitis. C) Use of warm saline gargles or throat irrigations can relieve symptoms. D) Heat may increase the spasms in pharyngeal muscles.

Ans: C Feedback: Depending on the severity of the pharyngitis and the degree of pain, warm saline gargles or throat irrigations are used. The benefits of this treatment depend on the degree of heat that is applied. The nurse teaches about these procedures and about the recommended temperature of the solution: high enough to be effective and as warm as the patient can tolerate, usually 105F to 110F (40.6C to 43.3C). Irrigating the throat may reduce spasm in the pharyngeal muscles and relieve soreness of the throat. You would not tell the parent teacher organization that there is no real treatment of pharyngitis.

The nurse is caring for a patient in the ED for epistaxis. What information should the nurse include in patient discharge teaching as a way to prevent epistaxis? A) Keep nasal passages clear. B) Use decongestants regularly. C) Humidify the indoor environment. D) Use a tissue when blowing the nose.

Ans: C Feedback: Discharge teaching for prevention of epistaxis should include the following: avoid forceful nose bleeding, straining, high altitudes, and nasal trauma (nose picking). Adequate humidification may prevent drying of the nasal passages. Keeping nasal passages clear and using a tissue when blowing the nose are not included in discharge teaching for the prevention of epistaxis. Decongestants are not indicated.

The nurse is planning the care of a patient who is scheduled for a laryngectomy. The nurse should assign the highest priority to which postoperative nursing diagnosis? A) Anxiety related to diagnosis of cancer B) Altered nutrition related to swallowing difficulties C) Ineffective airway clearance related to airway alterations D) Impaired verbal communication related to removal of the larynx

Ans: C Feedback: Each of the listed diagnoses is valid, but ineffective airway clearance is the priority nursing diagnosis for all conditions.

A patient in the ICU has had an endotracheal tube in place for 3 weeks. The physician has ordered that a tracheostomy tube be placed. The patient's family wants to know why the endotracheal tube cannot be left in place. What would be the nurse's best response? A) "The physician may feel that mechanical ventilation will have to be used long-term." B) "Long-term use of an endotracheal tube diminishes the normal breathing reflex." C) "When an endotracheal tube is left in too long it can damage the lining of the windpipe." D) "It is much harder to breathe through an endotracheal tube than a tracheostomy."

Ans: C Feedback: Endotracheal intubation may be used for no longer than 2 to 3 weeks, by which time a tracheostomy must be considered to decrease irritation of and, trauma to, the tracheal lining, to reduce the incidence of vocal cord paralysis (secondary to laryngeal nerve damage), and to decrease the work of breathing. The need for long-term ventilation would not be the primary rationale for this change in treatment. Endotracheal tubes do not diminish the breathing reflex.

A female patient, 47 years old, visits the clinic because she has been experiencing stress incontinence when she sneezes or exercises vigorously. What is the best instruction the nurse can give the patient? A) Keep a record of when the incontinence occurs. B) Perform clean intermittent self-catheterization. C) Perform Kegel exercises four to six times per day. D) Wear a protective undergarment to address this age-related change.

Ans: C Feedback: For cognitively intact women who experience stress incontinence, the nurse should instruct the patient to perform Kegel exercises four to six times per day to strengthen the pubococcygeus muscle. Keeping a record of when the incontinence occurs or accepting incontinence as part of aging are incorrect answers because they are of no value in treating stress incontinence. Women with stress incontinence do not need clean intermittent catheterization. Protective undergarments hide the effects of urinary incontinence but they do not resolve the problem.

The nurse is providing care for a 90-year-old patient whose severe cognitive and mobility deficits result in the nursing diagnosis of risk for impaired skin integrity due to lack of mobility. When planning relevant assessments, the nurse should prioritize inspection of what area? A) The patient's elbows B) The soles of the patient's feet C) The patient's heels D) The patient's knees

Ans: C Feedback: Full inspection of the patient's skin is necessary, but the coccyx and the heels are the most susceptible areas for skin breakdown due to shear and friction.

The occupational health nurse is obtaining a patient history during a pre-employment physical. During the history, the patient states that he has hereditary angioedema. The nurse should identify what implication of this health condition? A) It will result in increased loss of work days. B) It may cause episodes of weakness due to reduced cardiac output. C) It can cause life-threatening airway obstruction. D) It is unlikely to interfere with the individuals health.

Ans: C Feedback: Hereditary angioedema is an inherited condition that is characterized by episodes of life-threatening laryngeal edema. No information supports lost days of work or reduced cardiac function.

The rehabilitation team has reaffirmed the need to maximize the independence of a patient in rehabilitation. When working toward this goal, what action should the nurse prioritize? A) Encourage families to become paraprofessionals in rehabilitation. B) Delegate care planning to the patient and family. C) Recognize the importance of informal caregivers. D) Make patients and families to work together.

Ans: C Feedback: In working toward maximizing independence, nurses affirm the patient as an active participant and recognize the importance of informal caregivers in the rehabilitation process. Nurses do not encourage families to become paraprofessionals in rehabilitation. The patient and family are central, but care planning is not their responsibility. Nurses do not "make" patients and families work together.

The nurse is performing preoperative teaching with a patient who has cancer of the larynx. After completing patient teaching, what would be most important for the nurse to do? A) Give the patient his or her cell phone number. B) Refer the patient to a social worker or psychologist. C) Provide the patient with audiovisual materials about the surgery. D) Reassure the patient and family that everything will be alright.

Ans: C Feedback: Informational materials (written and audiovisual) about the surgery are given to the patient and family for review and reinforcement. The nurse never gives personal contact information to the patient. Nothing in the scenario indicates that a referral to a social worker or psychologist is necessary. False reassurance must always be avoided.

The nurse is creating a plan of car for a patient diagnosed with acute laryngitis. What intervention should be included in the patients plan of care? A) Place warm cloths on the patients throat, as needed. B) Have the patient inhale warm steam three times daily. C) Encourage the patient to limit speech whenever possible. D) Limit the patients fluid intake to 1.5 L/day.

Ans: C Feedback: Management of acute laryngitis includes resting the voice, avoiding irritants (including smoking), resting, and inhaling cool steam or an aerosol. Fluid intake should be increased. Warm cloths on the throat will not help relieve the symptoms of acute laryngitis.

A nursing student is discussing a patient with viral pharyngitis with the preceptor at the walk-in clinic. What should the preceptor tell the student about nursing care for patients with viral pharyngitis? A) Teaching focuses on safe and effective use of antibiotics. B) The patient should be preliminarily screened for surgery. C) Symptom management is the main focus of medical and nursing care. D) The focus of care is resting the voice to prevent chronic hoarseness.

Ans: C Feedback: Nursing care for patients with viral pharyngitis focuses on symptomatic management. Antibiotics are not prescribed for viral etiologies. Surgery is not indicated in the treatment of viral pharyngitis. Chronic hoarseness is not a common sequela of viral pharyngitis, so teaching ways to prevent it would be of no use in this instance.

A 45-year-old obese man arrives in a clinic with complaints of daytime sleepiness, difficulty going to sleep at night, and snoring. The nurse should recognize the manifestations of what health problem? A) Adenoiditis B) Chronic tonsillitis C) Obstructive sleep apnea D) Laryngeal cancer

Ans: C Feedback: Obstructive sleep apnea occurs in men, especially those who are older and overweight. Symptoms include excessive daytime sleepiness, insomnia, and snoring. Daytime sleepiness and difficulty going to sleep at night are not indications of tonsillitis or adenoiditis. This patients symptoms are not suggestive of laryngeal cancer.

The nurse is teaching a patient with allergic rhinitis about the safe and effective use of his medications. What would be the most essential information to give this patient about preventing possible drug interactions? A) Prescription medications can be safely supplemented with OTC medications. B) Use only one pharmacy so the pharmacist can check drug interactions. C) Read drug labels carefully before taking OTC medications. D) Consult the Internet before selecting an OTC medication.

Ans: C Feedback: Patient education is essential when assisting the patient in the use of all medications. To prevent possible drug interactions, the patient is cautioned to read drug labels before taking any OTC medications. Some Web sites are reliable and valid information sources, but this is not always the case. Patients do not necessarily need to limit themselves to one pharmacy, though checking for potential interactions is important. Not all OTC medications are safe additions to prescription medication regimens.

A 52-year-old married man with two adolescent children is beginning rehabilitation following a motor vehicle accident. You are the nurse planning the patient's care. Who will the patient's condition affect? A) Himself B) His wife and any children that still live at home C) Him and his entire family D) No one, provided he has a complete recovery

Ans: C Feedback: Patients and families who suddenly experience a physically disabling event or the onset of a chronic illness are the ones who face several psychosocial adjustments, even if the patient recovers completely.

The nurse is working with a rehabilitation patient who has a deficit in mobility following a skiing accident. The nurse knows that preparation for ambulation is extremely important. What nursing action will best provide the foundation of preparation for ambulation? A) Stimulating the patient's desire to ambulate B) Assessing the patient's understanding of ambulation C) Helping the patient perform frequent exercise D) Setting realistic expectations

Ans: C Feedback: Regaining the ability to walk is a prime morale builder. However, to be prepared for ambulation—whether with brace, walker, cane, or crutches—the patient must strengthen the muscles required. Therefore, exercise is the foundation of preparation.

An elderly patient is brought to the emergency department with a fractured tibia. The patient appears malnourished, and the nurse is concerned about the patient's healing process related to insufficient protein levels. What laboratory finding would the floor nurse prioritize when assessing for protein deficiency? A) Hemoglobin B) Bilirubin C) Albumin D) Cortisol

Ans: C Feedback: Serum albumin is a sensitive indicator of protein deficiency. Albumin levels of less than 3 g/mL are indicative of hypoalbuminemia. Altered hemoglobin levels, cortisol levels, and bilirubin levels are not indicators of protein deficiency.

You are the nurse caring for a patient who has paraplegia following a hunting accident. You know to assess regularly for the development of pressure ulcers on this patient. What rationale would you cite for this nursing action? A) You know that this patient will have a decreased level of consciousness. B) You know that this patient may not be motivated to prevent pressure ulcers. C) You know that the risk for pressure ulcers is directly related to the duration of immobility. D) You know that the risk for pressure ulcers is related to what caused the immobility.

Ans: C Feedback: The development of pressure ulcers is directly related to the duration of immobility: If pressure continues long enough, small vessel thrombosis and tissue necrosis occur, and a pressure ulcer results. The cause of the immobility is not what is important in the development of a pressure ulcer; the duration of the immobility is what matters. Paraplegia does not result in a decreased level of consciousness and there is no reason to believe that the patient does not want to prevent pressure ulcers.

The nurse has noted the emergence of a significant amount of fresh blood at the drain site of a patient who is postoperative day 1 following total laryngectomy. How should the nurse respond to this development? A) Remove the patients drain and apply pressure with a sterile gauze. B) Assess the patient, reposition the patient supine, and apply wall suction to the drain. C) Rapidly assess the patient and notify the surgeon about the patients bleeding. D) Administer a STAT dose of vitamin K to aid coagulation.

Ans: C Feedback: The nurse promptly notifies the surgeon of any active bleeding, which can occur at a variety of sites, including the surgical site, drains, and trachea. The drain should not be removed or connected to suction. Supine positioning would exacerbate the bleeding. Vitamin K would not be administered without an order.

You have been referred to the care of an extended care resident who has been diagnosed with a stage III pressure ulcer. You are teaching staff at the facility about the role of nutrition in wound healing. What would be the best meal choice for this patient? A) Whole wheat macaroni with cheese B) Skim milk, oatmeal, and whole wheat toast C) Steak, baked potato, spinach and strawberry salad D) Eggs, hash browns, coffee, and an apple

Ans: C Feedback: The patient should be encouraged to eat foods high in protein, carbohydrates and vitamins A, B, and C. A meal of steak, baked potato, spinach and strawberry salad best exemplifies this dietary balance.

The nurse is creating a care plan for a patient who is status post-total laryngectomy. Much of the plan consists of a long-term postoperative communication plan for alaryngeal communication. What form of alaryngeal communication will likely be chosen? A) Esophageal speech B) Electric larynx C) Tracheoesophageal puncture D) American sign language (ASL)

Ans: C Feedback: Tracheoesophageal puncture is simple and has few complications. It is associated with high phonation success, good phonation quality, and steady long-term results. As a result, it is preferred over esophageal speech, and electric larynx or ASL.

A nurse is caring for a patient undergoing rehabilitation following a snowboarding accident. Within the interdisciplinary team, the nurse has been given the responsibility for coordinating the patient's total rehabilitative plan of care. What nursing role is this nurse performing? A) Patient educator B) Caregiver C) Case manager D) Patient advocate

Ans: C Feedback: When the nurse coordinates the patient's total rehabilitative plan of care, the nurse is functioning as a case manager. The nurse must coordinate services provided by all of the team members. The other answers are incorrect.

The nurse is providing discharge teaching for a patient who developed a pulmonary embolism after total knee surgery. The patient has been converted from heparin to sodium warfarin (Coumadin) anticoagulant therapy. What should the nurse teach the client? A) Coumadin will continue to break up the clot over a period of weeks B) Coumadin must be taken concurrent with ASA to achieve anticoagulation. C) Anticoagulant therapy usually lasts between 3 and 6 months. D) He should take a vitamin supplement containing vitamin K

Ans: C Feedback: Anticoagulant therapy prevents further clot formation, but cannot be used to dissolve a clot. The therapy continues for approximately 3 to 6 months and is not combined with ASA. Vitamin K reverses the effect of anticoagulant therapy and normally should not be taken.

When assessing for substances that are known to harm workers lungs, the occupational health nurse should assess their potential exposure to which of the following? A) Organic acids B) Propane C) Asbestos D) Gypsum

Ans: C Feedback: Asbestos is among the more common causes of pneumoconiosis. Organic acids, propane, and gypsum do not have this effect.

A patient presents to the ED stating she was in a boating accident about 3 hours ago. Now the patient has complaints of headache, fatigue, and the feeling that he just cant breathe enough. The nurse notes that the patient is restless and tachycardic with an elevated blood pressure. This patient may be in the early stages of what respiratory problem? A) Pneumoconiosis B) Pleural effusion C) Acute respiratory failure D) Pneumonia

Ans: C Feedback: Early signs of acute respiratory failure are those associated with impaired oxygenation and may include restlessness, fatigue, headache, dyspnea, air hunger, tachycardia, and increased blood pressure. As the hypoxemia progresses, more obvious signs may be present, including confusion, lethargy, tachycardia, tachypnea, central cyanosis, diaphoresis, and, finally, respiratory arrest. Pneumonia is infectious and would not result from trauma. Pneumoconiosis results from exposure to occupational toxins. A pleural effusion does not cause this constellation of symptoms.

A client presents to the walk-in clinic complaining of a dry, irritating cough and production of a minute amount of mucus-like sputum. The patient complains of soreness in her chest in the sternal area. The nurse should suspect that the primary care provider will assess the patient for what health problem? A) Pleural effusion B) Pulmonary embolism C) Tracheobronchitis D) Tuberculosis

Ans: C Feedback: Initially, the patient with tracheobronchitis has a dry, irritating cough and expectorates a scant amount of mucoid sputum. The patient may report sternal soreness from coughing and have fever or chills, night sweats, headache, and general malaise. Pleural effusion and pulmonary embolism do not normally cause sputum production and would likely cause acute shortness of breath. Hemoptysis is characteristic of TB.

The nurse is reviewing the electronic health record of a patient with an empyema. What health problem in the patients history is most likely to have caused the empyema? A) Smoking B) Asbestosis C) Pneumonia D) Lung cancer

Ans: C Feedback: Most empyemas occur as complications of bacterial pneumonia or lung abscess. Cancer, smoking, and asbestosis are not noted to be common causes.

The nurse is caring for a 46-year-old patient recently diagnosed with the early stages of lung cancer. The nurse is aware that the preferred method of treating patients with nonsmall cell tumors is what? A) Chemotherapy B) Radiation C) Surgical resection D) Bronchoscopic opening of the airway

Ans: C Feedback: Surgical resection is the preferred method of treating patients with localized nonsmall cell tumors with no evidence of metastatic spread and adequate cardiopulmonary function. The other listed treatment options may be considered, but surgery is preferred.

The nurse is caring for a patient in the ICU admitted with ARDS after exposure to toxic fumes from a hazardous spill at work. The patient has become hypotensive. What is the cause of this complication to the ARDS treatment? A) Pulmonary hypotension due to decreased cardiac output B) Severe and progressive pulmonary hypertension C) Hypovolemia secondary to leakage of fluid into the interstitial spaces D) Increased cardiac output from high levels of PEEP therapy

Ans: C Feedback: Systemic hypotension may occur in ARDS as a result of hypovolemia secondary to leakage of fluid into the interstitial spaces and depressed cardiac output from high levels of PEEP therapy. Pulmonary hypertension, not pulmonary hypotension, sometimes is a complication of ARDS, but it is not the cause of the patient becoming hypotensive.

A gerontologic nurse is teaching a group of medical nurses about the high incidence and mortality of pneumonia in older adults. What is a contributing factor to this that the nurse should describe? A) Older adults have less compliant lung tissue than younger adults. B) Older adults are not normally candidates for pneumococcal vaccination. C) Older adults often lack the classic signs and symptoms of pneumonia. D) Older adults often cannot tolerate the most common antibiotics used to treat pneumonia.

Ans: C Feedback: The diagnosis of pneumonia may be missed because the classic symptoms of cough, chest pain, sputum production, and fever may be absent or masked in older adult patients. Mortality from pneumonia in the elderly is not a result of limited antibiotic options or lower lung compliance. The pneumococcal vaccine is appropriate for older adults.

The nurse is caring for a patient who is receiving oxygen therapy for pneumonia. How should the nurse best assess whether the patient is hypoxemic? A) Assess the patients level of consciousness (LOC). B) Assess the patients extremities for signs of cyanosis. C) Assess the patients oxygen saturation level. D) Review the patients hemoglobin, hematocrit, and red blood cell levels.

Ans: C Feedback: The effectiveness of the patients oxygen therapy is assessed by the ABG analysis or pulse oximetry. ABG results may not be readily available. Presence or absence of cyanosis is not an accurate indicator of oxygen effectiveness. The patients LOC may be affected by hypoxia, but not every change in LOC is related to oxygenation. Hemoglobin, hematocrit, and red blood cell levels do not directly reflect current oxygenation status.

The nurse is caring for an 82-year-old patient with a diagnosis of tracheobronchitis. The patient begins complaining of right-sided chest pain that gets worse when he coughs or breathes deeply. Vital signs are within normal limits. What would you suspect this patient is experiencing? A) Traumatic pneumothorax B) Empyema C) Pleuritic pain D) Myocardial infarction

Ans: C Feedback: The key characteristic of pleuritic pain is its relationship to respiratory movement. Taking a deep breath, coughing, or sneezing worsens the pain. Pleuritic pain is limited in distribution rather than diffuse; it usually occurs only on one side. The pain may become minimal or absent when the breath is held. It may be localized or radiate to the shoulder or abdomen. Later, as pleural fluid develops, the pain decreases. The scenario does not indicate any trauma to the patient, so a traumatic pneumothorax is implausible. Empyema is unlikely as there is no fever indicative of infection. Myocardial infarction would affect the patients vital signs profoundly.

A patient has just been diagnosed with lung cancer. After the physician discusses treatment options and leaves the room, the patient asks the nurse how the treatment is decided upon. What would be the nurses best response? A) The type of treatment depends on the patients age and health status. B) The type of treatment depends on what the patient wants when given the options. C) The type of treatment depends on the cell type of the cancer, the stage of the cancer, and the patients health status. D) The type of treatment depends on the discussion between the patient and the physician of which treatment is best.

Ans: C Feedback: Treatment of lung cancer depends on the cell type, the stage of the disease, and the patients physiologic status (particularly cardiac and pulmonary status). Treatment does not depend solely on the patients age or the patients preference between the different treatment modes. The decision about treatment does not primarily depend on a discussion between the patient and the physician of which treatment is best, though this discussion will take place.

9. A patient asks the nurse why an infection in his upper respiratory system is affecting the clarity of his speech. Which structure serves as the patients resonating chamber in speech? A) Trachea B) Pharynx C) Paranasal sinuses D) Larynx

Ans: C Feedback: A prominent function of the sinuses is to serve as a resonating chamber in speech. The trachea, also known as the windpipe, serves as the passage between the larynx and the bronchi. The pharynx is a tubelike structure that connects the nasal and oral cavities to the larynx. The pharynx also functions as a passage for the respiratory and digestive tracts. The major function of the larynx is vocalization through the function of the vocal cords. The vocal cords are ligaments controlled by muscular movements that produce sound.

11. A patient is undergoing testing to see if he has a pleural effusion. Which of the nurses respiratory assessment findings would be most consistent with this diagnosis? A) Increased tactile fremitus, egophony, and a dull sound upon percussion of the chest wall B) Decreased tactile fremitus, wheezing, and a hyperresonant sound upon percussion of the chest wall C) Lung fields dull to percussion, absent breath sounds, and a pleural friction rub D) Normal tactile fremitus, decreased breath sounds, and a resonant sound upon percussion of the chest wall

Ans: C Feedback: Assessment findings consistent with a pleural effusion include affected lung fields being dull to percussion and absence of breath sounds. A pleural friction rub may also be present. The other listed signs are not typically associated with a pleural effusion.

19. The nurse is caring for an elderly patient in the PACU. The patient has had a bronchoscopy, and the nurse is monitoring for complications related to the administration of lidocaine. For what complication related to the administration of large doses of lidocaine in the elderly should the nurse assess? A) Decreased urine output and hypertension B) Headache and vision changes C) Confusion and lethargy D) Jaundice and elevated liver enzymes

Ans: C Feedback: Lidocaine may be sprayed on the pharynx or dropped on the epiglottis and vocal cords and into the trachea to suppress the cough reflex and minimize discomfort during a bronchoscopy. After the procedure, the nurse will assess for confusion and lethargy in the elderly, which may be due to the large doses of lidocaine administered during the procedure. The other listed signs and symptoms are not specific to this problem.

34. The ED nurse is assessing the respiratory function of a teenage girl who presented with acute shortness of breath. Auscultation reveals continuous wheezes during inspiration and expiration. This finding is most suggestive what? A) Pleurisy B) Emphysema C) Asthma D) Pneumonia

Ans: C Feedback: Sibilant wheezes are commonly associated with asthma. They do not normally accompany pleurisy, emphysema, or pneumonia.

38. The nurse is completing a patients health history with regard to potential risk factors for lung disease. What interview question addresses the most significant risk factor for respiratory diseases? A) Have you ever been employed in a factory, smelter, or mill? B) Does anyone in your family have any form of lung disease? C) Do you currently smoke, or have you ever smoked? D) Have you ever lived in an area that has high levels of air pollution?

Ans: C Feedback: Smoking the single most important contributor to lung disease, exceeds the significance of environmental, occupational, and genetic factors.

5. The nurse is caring for a patient admitted with an acute exacerbation of chronic obstructive pulmonary disease. During assessment, the nurse finds that the patient is experiencing increased dyspnea. What is the most accurate measurement of the concentration of oxygen in the patients blood? A) A capillary blood sample B) Pulse oximetry C) An arterial blood gas (ABG) study D) A complete blood count (CBC)

Ans: C Feedback: The arterial oxygen tension (partial pressure or PaO2 ) indicates the degree of oxygenation of the blood, and the arterial carbon dioxide tension (partial pressure or PaCO2 ) indicates the adequacy of alveolar ventilation. ABG studies aid in assessing the ability of the lungs to provide adequate oxygen and remove carbon dioxide and the ability of the kidneys to reabsorb or excrete bicarbonate ions to maintain normal body pH. Capillary blood samples are venous blood, not arterial blood, so they are not as accurate as an ABG. Pulse oximetry is a useful clinical tool but does not replace ABG measurement, because it is not as accurate. A CBC does not indicate the concentration of oxygen.

31. The nurse is assessing the respiratory status of a patient who is experiencing an exacerbation of her emphysema symptoms. When preparing to auscultate, what breath sounds should the nurse anticipate? A) Absence of breath sounds B) Wheezing with discontinuous breath sounds C) Faint breath sounds with prolonged expiration D) Faint breath sounds with fine crackles

Ans: C Feedback: The breath sounds of the patient with emphysema are faint or often completely inaudible. When they are heard, the expiratory phase is prolonged.

15. A medical nurse has admitted a patient to the unit with a diagnosis of failure to thrive. The patient has developed a fever and cough, so a sputum specimen has been obtained. The nurse notes that the sputum is greenish and that there is a large quantity of it. The nurse notifies the patients physician because these symptoms are suggestive of what? A) Pneumothorax B) Lung tumors C) Infection D) Pulmonary edema

Ans: C Feedback: The nature of the sputum is often indicative of its cause. A profuse amount of purulent sputum (thick and yellow, green, or rust-colored) or a change in color of the sputum is a common sign of a bacterial infection. Pink-tinged mucoid sputum suggests a lung tumor. Profuse, frothy, pink material, often welling up into the throat, may indicate pulmonary edema. A pneumothorax does not result in copious, green sputum.

18. While assessing an acutely ill patients respiratory rate, the nurse assesses four normal breaths followed by an episode of apnea lasting 20 seconds. How should the nurse document this finding? A) Eupnea B) Apnea C) Biots respiration D) Cheyne-Stokes

Ans: C Feedback: The nurse will document that the patient is demonstrating a Biots respiration pattern. Biots respiration is characterized by periods of normal breathing (three to four breaths) followed by varying periods of apnea (usually 10 seconds to 1 minute). Cheyne-Stokes is a similar respiratory pattern, but it involves a regular cycle where the rate and depth of breathing increase and then decrease until apnea occurs. Biots respiration is not characterized by the increase and decrease in the rate and depth, as characterized by Cheyne-Stokes. Eupnea is a normal breathing pattern of 12 to 18 breaths per minute. Bradypnea is a slower-than-normal rate (<10 breaths per minute), with normal depth and regular rhythm, and no apnea.

7. A patient with chronic lung disease is undergoing lung function testing. What test result denotes the volume of air inspired and expired with a normal breath? A) Total lung capacity B) Forced vital capacity C) Tidal volume D) Residual volume

Ans: C Feedback: Tidal volume refers to the volume of air inspired and expired with a normal breath. Total lung capacity is the maximal amount of air the lungs and respiratory passages can hold after a forced inspiration. Forced vital capacity is vital capacity performed with a maximally forced expiration. Residual volume is the maximal amount of air left in the lung after a maximal expiration.

A nurse is working with a child who is undergoing a diagnostic workup for suspected asthma. What are the signs and symptoms that are consistent with a diagnosis of asthma? Select all that apply. A) Chest tightness B) Crackles C) Bradypnea D) Wheezing E) Cough

Ans: Chest tightness, Wheezing, Cough Feedback: Asthma is a chronic inflammatory disease of the airways that causes airway hyperresponsiveness, mucosal edema, and mucus production. This inflammation ultimately leads to recurrent episodes of asthma symptoms: cough, chest tightness, wheezing, and dyspnea. Crackles and bradypnea are not typical symptoms of asthma.

A nurse is developing the teaching portion of a care plan for a patient with COPD. What would be the most important component for the nurse to emphasize? A) Smoking up to one-half of a pack of cigarettes weekly is allowable. B) Chronic inhalation of indoor toxins can cause lung damage. C) Minor respiratory infections are considered to be self-limited and are not treated. D) Activities of daily living (ADLs) should be clustered in the early morning hours.

Ans: Chronic inhalation of indoor toxins can cause lung damage. Feedback: Environmental risk factors for COPD include prolonged and intense exposure to occupational dusts and chemicals, indoor air pollution, and outdoor air pollution. Smoking cessation should be taught to all patients who are currently smoking. Minor respiratory infections that are of no consequence to the person with normal lungs can produce fatal disturbances in the lungs of the person with emphysema. ADLs should be paced throughout the day to permit patients to perform these without excessive distress.

A nurse is documenting the results of assessment of a patient with bronchiectasis. What would the nurse most likely include in documentation? A) Sudden onset of pleuritic chest pain B) Wheezes on auscultation C) Increased anterior-posterior (A-P) diameter D) Clubbing of the fingers

Ans: Clubbing of the fingers Feedback: Characteristic symptoms of bronchiectasis include chronic cough and production of purulent sputum in copious amounts. Clubbing of the fingers also is common because of respiratory insufficiency. Sudden pleuritic chest pain is a common manifestation of a pulmonary embolism. Wheezes on auscultation are common in patients with asthma. An increased A-P diameter is noted in patients with COPD.

A nurse is caring for a patient who has been admitted with an exacerbation of chronic bronchiectasis. The nurse should expect to assess the patient for which of the following clinical manifestations? A) Copious sputum production B) Pain on inspiration C) Pigeon chest D) Dry cough

Ans: Copious sputum production Feedback: Clinical manifestations of bronchiectasis include hemoptysis, chronic cough, copious purulent sputum, and clubbing of the fingers. Because of the copious production of sputum, the cough is rarely dry. A pigeon chest is not associated with the disease and patients do not normally experience pain on inspiration.

A nurse is teaching a patient with asthma about Azmacort, an inhaled corticosteroid. Which adverse effects should the nurse be sure to address in patient teaching? A) Dyspnea and increased respiratory secretions B) Nausea and vomiting C) Cough and oral thrush D) Fatigue and decreased level of consciousness

Ans: Cough and oral thrush Feedback: Azmacort has possible adverse effects of cough, dysphonia, oral thrush (candidiasis), and headache. In high doses, systemic effects may occur (e.g., adrenal suppression, osteoporosis, skin thinning, and easy bruising). The other listed adverse effects are not associated with this drug.

The home care nurse is planning to begin breathing retraining exercises with a client newly admitted to the home health service. The home care nurse knows that breathing retraining is especially indicated if the patient has what diagnosis? A) Asthma B) Pneumonia C) Lung cancer D) COPD

Ans: D Feedback: Breathing retraining is especially indicated in patients with COPD and dyspnea. Breathing retraining may be indicated in patients with other lung pathologies, but not to the extent indicated in patients with COPD.

The nurse is performing patient education for a patient who is being discharged on mini-nebulizer treatments. What information should the nurse prioritize in the patients discharge teaching? A) How to count her respirations accurately B) How to collect serial sputum samples C) How to independently wean herself from treatment D) How to perform diaphragmatic breathing

Ans: D Feedback: Diaphragmatic breathing is a helpful technique to prepare for proper use of the small-volume nebulizer. Patient teaching would not include counting respirations and the patient should not wean herself from treatment without the involvement of her primary care provider. Serial sputum samples are not normally necessary.

The nurse is assessing a patient who has a chest tube in place for the treatment of a pneumothorax. The nurse observes that the water level in the water seal rises and falls in rhythm with the patients respirations. How should the nurse best respond to this assessment finding? A) Gently reinsert the chest tube 1 to 2 cm and observe if the water level stabilizes. B) Inform the physician promptly that there is in imminent leak in the drainage system. C) Encourage the patient to do deep breathing and coughing exercises. D) Document that the chest drainage system is operating as it is intended.

Ans: D Feedback: Fluctuation of the water level in the water seal shows effective connection between the pleural cavity and the drainage chamber and indicates that the drainage system remains patent. No further action is needed.

A patients plan of care specifies postural drainage. What action should the nurse perform when providing this noninvasive therapy? A) Administer the treatment with the patient in a high Fowlers or semi-Fowlers position. B) Perform the procedure immediately following the patients meals. C) Apply percussion firmly to bare skin to facilitate drainage. D) Assist the patient into a position that will allow gravity to move secretions.

Ans: D Feedback: Postural drainage is usually performed two to four times per day. The patient uses gravity to facilitate postural draining. The skin should be covered with a cloth or a towel during percussion to protect the skin. Postural drainage is not administered in an upright position or directly following a meal.

The home care nurse is visiting a patient newly discharged home after a lobectomy. What would be most important for the home care nurse to assess? A) Resumption of the patients ADLs B) The familys willingness to care for the patient C) Nutritional status and fluid balance D) Signs and symptoms of respiratory complications

Ans: D Feedback: The nurse assesses the patient's adherence to the postoperative treatment plan and identifies acute or late postoperative complications. All options presented need assessment, but respiratory complications are the highest priority because they affect the patient's airway and breathing.

The nurse is discussing activity management with a patient who is postoperative following thoracotomy. What instructions should the nurse give to the patient regarding activity immediately following discharge? A) Walk 1 mile 3 to 4 times a week. B) Use weights daily to increase arm strength. C) Walk on a treadmill 30 minutes daily. D) Perform shoulder exercises five times daily.

Ans: D Feedback: The nurse emphasizes the importance of progressively increased activity. The nurse also instructs the patient on the importance of performing shoulder exercises five times daily. The patient should ambulate with limits and realize that the return of strength will likely be gradual and likely will not include weight lifting or lengthy walks.

The decision has been made to discharge a ventilator-dependent patient home. The nurse is developing a teaching plan for this patient and his family. What would be most important to include in this teaching plan? A) Administration of inhaled corticosteroids B) Assessment of neurologic status C) Turning and coughing D) Signs of pulmonary infection

Ans: D Feedback: The nurse teaches the patient and family about the ventilator, suctioning, tracheostomy care, signs of pulmonary infection, cuff inflation and deflation, and assessment of vital signs. Neurologic assessment and turning and coughing are less important than signs and symptoms of infection. Inhaled corticosteroids may or may not be prescribed.

The medical nurse is creating the care plan of an adult patient requiring mechanical ventilation. What nursing action is most appropriate? A) Keep the patient in a low Fowlers position. B) Perform tracheostomy care at least once per day. C) Maintain continuous bedrest. D) Monitor cuff pressure every 8 hours.

Ans: D Feedback: The cuff pressure should be monitored every 8 hours. It is important to perform tracheostomy care at least every 8 hours because of the risk of infection. The patient should be encouraged to ambulate, if possible, and a low Fowler's position is not indicated.

The nurse is caring for a patient who is postoperative day 2 following a total laryngectomy for supraglottic cancer. The nurse should prioritize what assessment? A) Assessment of body image B) Assessment of jugular venous pressure C) Assessment of carotid pulse D) Assessment of swallowing ability

Ans: D Feedback: A common postoperative complication from this type of surgery is difficulty in swallowing, which creates a potential for aspiration. Cardiovascular complications are less likely at this stage of recovery. The patients body image should be assessed, but dysphagia has the potential to affect the patients airway, and is a consequent priority.

The campus nurse at a university is assessing a 21-year-old student who presents with a severe nosebleed. The site of bleeding appears to be the anterior portion of the nasal septum. The nurse instructs the student to tilt her head forward and the nurse applies pressure to the nose, but the students nose continues to bleed. Which intervention should the nurse next implement? A) Apply ice to the bridge of her nose B) Lay the patient down on a cot C) Arrange for transfer to the local ED D) Insert a tampon in the affected nare

Ans: D Feedback: A cotton tampon may be used to try to stop the bleeding. The use of ice on the bridge of the nose has no scientific rationale for care. Laying the client down on the cot could block the clients airway. Hospital admission is necessary only if the bleeding becomes serious.

A patient is undergoing rehabilitation following a stroke that left him with severe motor and sensory deficits. The patient has been unable to ambulate since his accident, but has recently achieved the goals of sitting and standing balance. What is the patient now able to use? A) A cane B) Crutches C) A two-wheeled walker D) Parallel bars

Ans: D Feedback: After sitting and standing balance is achieved, the patient is able to use parallel bars. The patient must be able to use the parallel bars before he can safely use devices like a cane, crutches, or a walker.

A nurse is educating a patient in anticipation of a procedure that will require a water-sealed chest drainage system. What should the nurse tell the patient and the family that this drainage system is used for? A) Maintaining positive chest-wall pressure B) Monitoring pleural fluid osmolarity C) Providing positive intrathoracic pressure D) Removing excess air and fluid

Ans: D Feedback: Chest tubes and closed drainage systems are used to re-expand the lung involved and to remove excess air, fluid, and blood. They are not used to maintain positive chest-wall pressure, monitor pleural fluid, or provide positive intrathoracic pressure.

You are the nurse providing care for a patient who has limited mobility after a stroke. What would you do to assess the patient for contractures? A) Assess the patient's deep tendon reflexes (DTRs). B) Assess the patient's muscle size. C) Assess the patient for joint pain. D) Assess the patient's range of motion.

Ans: D Feedback: Each joint of the body has a normal range of motion. To assess a patient for contractures, the nurse should assess whether the patient can complete the full range of motion. Assessing DTRs, muscle size, or joint pain do not reveal the presence or absence of contractures.

A 93-year-old male patient with failure to thrive has begun exhibiting urinary incontinence. When choosing appropriate interventions, you know that various age-related factors can alter urinary elimination patterns in elderly patients. What is an example of these factors? A) Decreased residual volume B) Urethral stenosis C) Increased bladder capacity D) Decreased muscle tone

Ans: D Feedback: Factors that alter elimination patterns in the older adult include decreased bladder capacity, decreased muscle tone, increased residual volumes, and delayed perception of elimination cues. The other noted phenomena are atypical.

A patient visiting the clinic is diagnosed with acute sinusitis. To promote sinus drainage, the nurse should instruct the patient to perform which of the following? A) Apply a cold pack to the affected area. B) Apply a mustard poultice to the forehead. C) Perform postural drainage. D) Increase fluid intake.

Ans: D Feedback: For a patient diagnosed with acute sinusitis, the nurse should instruct the patient that hot packs, increasing fluid intake, and elevating the head of the bed can promote drainage. Applying a mustard poultice will not promote sinus drainage. Postural drainage is used to remove bronchial secretions.

The nurse is providing patient teaching to a young mother who has brought her 3-month-old infant to the clinic for a well-baby checkup. What action should the nurse recommend to the woman to prevent the transmission of organisms to her infant during the cold season? A) Take preventative antibiotics, as ordered. B) Gargle with warm salt water regularly. C) Dress herself and her infant warmly. D) Wash her hands frequently.

Ans: D Feedback: Handwashing remains the most effective preventive measure to reduce the transmission of organisms. Taking prescribed antibiotics, using warm salt-water gargles, and dressing warmly do not suppress transmission. Antibiotics are not prescribed for a cold.

A patient comes to the ED and is admitted with epistaxis. Pressure has been applied to the patients midline septum for 10 minutes, but the bleeding continues. The nurse should anticipate using what treatment to control the bleeding? A) Irrigation with a hypertonic solution B) Nasopharyngeal suction C) Normal saline application D) Silver nitrate application

Ans: D Feedback: If pressure to the midline septum does not stop the bleeding for epistaxis, additional treatment of silver nitrate application, Gelfoam, electrocautery, or vasoconstrictors may be used. Suction may be used to visualize the nasal septum, but it does not alleviate the bleeding. Irrigation with a hypertonic solution is not used to treat epistaxis.

A patient recovering from thoracic surgery is on long-term mechanical ventilation and becomes very frustrated when he tries to communicate. What intervention should the nurse perform to assist the patient? A) Assure the patient that everything will be all right and that remaining calm is the best strategy. B) Ask a family member to interpret what the patient is trying to communicate. C) Ask the physician to wean the patient off the mechanical ventilator to allow the patient to speak freely. D) Express empathy and then encourage the patient to write, use a picture board, or spell words with an alphabet board.

Ans: D Feedback: If the patient uses an alternative method of communication, he will feel in better control and likely be less frustrated. Assuring the patient that everything will be all right offers false reassurance, and telling him not to be upset minimizes his feelings. Neither of these methods helps the patient to communicate. In a patient with an endotracheal or tracheostomy tube, the family members are also likely to encounter difficulty interpreting the patients wishes. Making them responsible for interpreting the patients gestures may frustrate the family. The patient may be weaned off a mechanical ventilator only when the physiologic parameters for weaning have been met.

A 42-year-old patient is admitted to the ED after an assault. The patient received blunt trauma to the face and has a suspected nasal fracture. Which of the following interventions should the nurse perform? A) Administer nasal spray and apply an occlusive dressing to the patients face. B) Position the patients head in a dependent position. C) Irrigate the patients nose with warm tap water. D) Apply ice and keep the patients head elevated.

Ans: D Feedback: Immediately after the fracture, the nurse applies ice and encourages the patient to keep the head elevated. The nurse instructs the patient to apply ice packs to the nose to decrease swelling. Dependent positioning would exacerbate bleeding and the nose is not irrigated. Occlusive dressings are not used.

A patients total laryngectomy has created a need for alaryngeal speech which will be achieved through the use of tracheoesophageal puncture. What action should the nurse describe to the patient when teaching him about this process? A) Training on how to perform controlled belching B) Use of an electronically enhanced artificial pharynx C) Insertion of a specialized nasogastric tube D) Fitting for a voice prosthesis

Ans: D Feedback: In patients receiving transesophageal puncture, a valve is placed in the tracheal stoma to divert air into the esophagus and out the mouth. Once the puncture is surgically created and has healed, a voice prosthesis (Blom-Singer) is fitted over the puncture site. A nasogastric tube and belching are not required. An artificial pharynx is not used.

You are planning rehabilitation activities for a patient who is working toward discharge back into the community. During a care conference, the team has identified a need to focus on the patient's instrumental activities of daily living (IADLs). When planning the patient's subsequent care, you should focus particularly on which of the following? A) Dressing B) Bathing C) Feeding D) Meal preparation

Ans: D Feedback: Instrumental activities of daily living (IADLs) include grocery shopping, meal preparation, housekeeping, transportation, and managing finances. Activities of daily living (ADLs) include bathing dressing, feeding, and toileting.

You are caring for a 35-year-old man whose severe workplace injuries necessitate bilateral below-the-knee amputations. How can you anticipate that the patient will respond to this news? A) The patient will go through the stages of grief over the next week to 10 days. B) The patient will progress sequentially through five stages of the grief process. C) The patient will require psychotherapy to process his grief. D) The patient will experience grief in an individualized manner.

Ans: D Feedback: Loss of limb is a profoundly emotional experience, which the patient will experience in a subjective manner, and largely unpredictable, manner. Psychotherapy may or may not be necessary. It is not possible to accurately predict the sequence or timing of the patient's grief. The patient may or may not benefit from psychotherapy.

An elderly female patient who is bedridden is admitted to the unit because of a pressure ulcer that can no longer be treated in a community setting. During your assessment of the patient, you find that the ulcer extends into the muscle and bone. At what stage would document this ulcer? A) I B) II C) III D) IV

Ans: D Feedback: Stage III and IV pressure ulcers are characterized by extensive tissue damage. In addition to the interventions listed for stage I, these advanced draining, necrotic pressure ulcers must be cleaned (débrided) to create an area that will heal. Stage IV is an ulcer that extends to underlying muscle and bone. Stage III is an ulcer that extends into the subcutaneous tissue. With this type of ulcer, necrosis of tissue and infection may develop. Stage I is an area of erythema that does not blanch with pressure. Stage II involves a break in the skin that may drain.

As a clinic nurse, you are caring for a patient who has been prescribed an antibiotic for tonsillitis and has been instructed to take the antibiotic for 10 days. When you do a follow-up call with this patient, you are informed that the patient is feeling better and is stopping the medication after taking it for 4 days. What information should you provide to this patient? A) Keep the remaining tablets for an infection at a later time. B) Discontinue the medications if the fever is gone. C) Dispose of the remaining medication in a biohazard receptacle. D) Finish all the antibiotics to eliminate the organism completely.

Ans: D Feedback: The nurse informs the patient about the need to take the full course of any prescribed antibiotic. Antibiotics should be taken for the entire 10-day period to eliminate the microorganisms. A patient should never be instructed to keep leftover antibiotics for use at a later time. Even if the fever or other symptoms are gone, the medications should be continued. Antibiotics do not need to be disposed of in a biohazard receptacle, though they should be discarded appropriately.

A female patient has been achieving significant improvements in her ADLs since beginning rehabilitation from the effects of a brain hemorrhage. The nurse must observe and assess the patient's ability to perform ADLs to determine the patient's level of independence in self-care and her need for nursing intervention. Which of the following additional considerations should the nurse prioritize? A) Liaising with the patient's insurer to describe the patient's successes. B) Teaching the patient about the pathophysiology of her functional deficits. C) Eliciting ways to get the patient to express a positive attitude. D) Appraising the family's involvement in the patient's ADLs.

Ans: D Feedback: The nurse should also be aware of the patient's medical conditions or other health problems, the effect that they have on the ability to perform ADLs, and the family's involvement in the patient's ADLs. It is not normally necessary to teach the patient about the pathophysiology of her functional deficits. A positive attitude is beneficial, but creating this is not normally within the purview of the nurse. The nurse does not liaise with the insurance company.

A nurse is teaching a patient how to perform flow type incentive spirometry prior to his scheduled thoracic surgery. What instruction should the nurse provide to the patient? A) Hold the spirometer at your lips and breathe in and out like you normally would. B) When youre ready, blow hard into the spirometer for as long as you can. C) Take a deep breath and then blow short, forceful breaths into the spirometer. D) Breathe in deeply through the spirometer, hold your breath briefly, and then exhale.

Ans: D Feedback: The patient should be taught to lace the mouthpiece of the spirometer firmly in the mouth, breathe air in through the mouth, and hold the breath at the end of inspiration for about 3 seconds. The patient should then exhale slowly through the mouthpiece.

The nurse is explaining the safe and effective administration of nasal spray to a patient with seasonal allergies. What information is most important to include in this teaching? A) Finish the bottle of nasal spray to clear the infection effectively. B) Nasal spray can only be shared between immediate family members. C) Nasal spray should be administered in a prone position. D) Overuse of nasal spray may cause rebound congestion.

Ans: D Feedback: The use of topical decongestants is controversial because of the potential for a rebound effect. The patient should hold his or her head back for maximal distribution of the spray. Only the patient should use the bottle.

The nurse caring for a patient with an endotracheal tube recognizes several disadvantages of an endotracheal tube. What would the nurse recognize as a disadvantage of endotracheal tubes? A) Cognition is decreased. B) Daily arterial blood gases (ABGs) are necessary. C) Slight tracheal bleeding is anticipated. D) The cough reflex is depressed.

Ans: D Feedback: There are several disadvantages of an endotracheal tube. Disadvantages include suppression of the patients cough reflex, thickening of secretions, and depressed swallowing reflexes. Ulceration and stricture of the larynx or trachea may develop, but bleeding is not an expected finding. The tube should not influence cognition and daily ABGs are not always required.

A patient in the ICU is status post embolectomy after a pulmonary embolus. What assessment parameter does the nurse monitor most closely on a patient who is postoperative following an embolectomy? A) Pupillary response B) Pressure in the vena cava C) White blood cell differential D) Pulmonary arterial pressure

Ans: D Feedback: If the patient has undergone surgical embolectomy, the nurse measures the patients pulmonary arterial pressure and urinary output. Pressure is not monitored in a patients vena cava. White cell levels and pupillary responses would be monitored, but not to the extent of the patients pulmonary arterial pressure.

The nurse caring for a patient recently diagnosed with lung disease encourages the patient not to smoke. What is the primary rationale behind this nursing action? A) Smoking decreases the amount of mucus production. B) Smoke particles compete for binding sites on hemoglobin. C) Smoking causes atrophy of the alveoli. D) Smoking damages the ciliary cleansing mechanism.

Ans: D Feedback: In addition to irritating the mucous cells of the bronchi and inhibiting the function of alveolar macrophage (scavenger) cells, smoking damages the ciliary cleansing mechanism of the respiratory tract. Smoking also increases the amount of mucus production and distends the alveoli in the lungs. It reduces the oxygen-carrying capacity of hemoglobin, but not by directly competing for binding sites.

An 87-year-old patient has been hospitalized with pneumonia. Which nursing action would be a priority in this patients plan of care? A) Nasogastric intubation B) Administration of probiotic supplements C) Bedrest D) Cautious hydration

Ans: D Feedback: Supportive treatment of pneumonia in the elderly includes hydration (with caution and with frequent assessment because of the risk of fluid overload in the elderly); supplemental oxygen therapy; and assistance with deep breathing, coughing, frequent position changes, and early ambulation. Mobility is not normally discouraged and an NG tube is not necessary in most cases. Probiotics may or may not be prescribed for the patient.

A 54-year-old man has just been diagnosed with small cell lung cancer. The patient asks the nurse why the doctor is not offering surgery as a treatment for his cancer. What fact about lung cancer treatment should inform the nurses response? A) The cells in small cell cancer of the lung are not large enough to visualize in surgery. B) Small cell lung cancer is self-limiting in many patients and surgery should be delayed. C) Patients with small cell lung cancer are not normally stable enough to survive surgery. D) Small cell cancer of the lung grows rapidly and metastasizes early and extensively.

Ans: D Feedback: Surgery is primarily used for NSCLCs, because small cell cancer of the lung grows rapidly and metastasizes early and extensively. Difficult visualization and a patients medical instability are not the limiting factors. Lung cancer is not a self-limiting disease.

The home care nurse is monitoring a patient discharged home after resolution of a pulmonary embolus. For what potential complication would the home care nurse be most closely monitoring this patient? A) Signs and symptoms of pulmonary infection B) Swallowing ability and signs of aspiration C) Activity level and role performance D) Residual effects of compromised oxygenation

Ans: D Feedback: The home care nurse should monitor the patient for residual effects of the PE, which involved a severe disruption in respiration and oxygenation. PE has a noninfectious etiology; pneumonia is not impossible, but it is a less likely sequela. Swallowing ability is unlikely to be affected; activity level is important, but secondary to the effects of deoxygenation.

The nurse is caring for a patient who has been in a motor vehicle accident and the care team suspects that the patient has developed pleurisy. Which of the nurses assessment findings would best corroborate this diagnosis? A) The patient is experiencing painless hemoptysis. B) The patients arterial blood gases (ABGs) are normal, but he demonstrates increased work of breathing. C) The patients oxygen saturation level is below 88%, but he denies shortness of breath. D) The patients pain intensifies when he coughs or takes a deep breath.

Ans: D Feedback: The key characteristic of pleuritic pain is its relationship to respiratory movement. Taking a deep breath, coughing, or sneezing worsens the pain. The patients ABGs would most likely be abnormal and shortness of breath would be expected.

16. A patient has been diagnosed with heart failure that has not yet responded to treatment. What breath sound should the nurse expect to assess on auscultation? A) Expiratory wheezes B) Inspiratory wheezes C) Rhonchi D) Crackles

Ans: D Feedback: Crackles reflect underlying inflammation or congestion and are often present in such conditions as pneumonia, bronchitis, and congestive heart failure. Rhonchi and wheezes are associated with airway obstruction, which is not a part of the pathophysiology of heart failure.

35. The nurse is caring for a patient who has been scheduled for a bronchoscopy. How should the nurse prepare the patient for this procedure? A) Administer a bolus of IV fluids. B) Arrange for the insertion of a peripherally inserted central catheter. C) Administer nebulized bronchodilators every 2 hours until the test. D) Withhold food and fluids for several hours before the test

Ans: D Feedback: Food and fluids are withheld for 4 to 8 hours before the test to reduce the risk of aspiration when the cough reflex is blocked by anesthesia. IV fluids, bronchodilators, and a central line are unnecessary.

32. The patient has just had an MRI ordered because a routine chest x-ray showed suspicious areas in the right lung. The physician suspects bronchogenic carcinoma. An MRI would most likely be order to assess for what in this patient? A) Alveolar dysfunction B) Forced vital capacity C) Tidal volume D) Chest wall invasion

Ans: D Feedback: MRI is used to characterize pulmonary nodules; to help stage bronchogenic carcinoma (assessment of chest wall invasion); and to evaluate inflammatory activity in interstitial lung disease, acute pulmonary embolism, and chronic thrombolytic pulmonary hypertension. Imaging would not focus on the alveoli since the problem in the bronchi. A static image such as MRI cannot inform PFT.

22. A patient is being treated for a pulmonary embolism and the medical nurse is aware that the patient suffered an acute disturbance in pulmonary perfusion. This involved an alteration in what aspect of normal physiology? A) Maintenance of constant osmotic pressure in the alveoli B) Maintenance of muscle tone in the diaphragm C) pH balance in the pulmonary veins and arteries D) Adequate flow of blood through the pulmonary circulation.

Ans: D Feedback: Pulmonary perfusion is the actual blood flow through the pulmonary circulation. Perfusion is not defined in terms of pH balance, muscle tone, or osmotic pressure.

8. In addition to heart rate, blood pressure, respiratory rate, and temperature, the nurse needs to assess a patients arterial oxygen saturation (SaO2 ). What procedure will best accomplish this? A) Incentive spirometry B) Arterial blood gas (ABG) measurement C) Peak flow measurement D) Pulse oximetry

Ans: D Feedback: Pulse oximetry is a noninvasive procedure in which a small sensor is positioned over a pulsating vascular bed. It can be used during transport and causes the patient no discomfort. An incentive spirometer is used to assist the patient with deep breathing after surgery. ABG measurement can measure SaO2, but this is an invasive procedure that can be painful. Some patients with asthma use peak flow meters to measure levels of expired air.

2. The nurse is caring for a patient who has just returned to the unit after a colon resection. The patient is showing signs of hypoxia. The nurse knows that this is probably caused by what? A) Nitrogen narcosis B) Infection C) Impaired diffusion D) Shunting

Ans: D Feedback: Shunting appears to be the main cause of hypoxia after thoracic or abdominal surgery and most types of respiratory failure. Impairment of normal diffusion is a less common cause. Infection would not likely be present at this early stage of recovery and nitrogen narcosis only occurs from breathing compressed air.

25. A patient is scheduled to have excess pleural fluid aspirated with a needle in order to relieve her dyspnea. The patient inquires about the normal function of pleural fluid. What should the nurse describe? A) It allows for full expansion of the lungs within the thoracic cavity. B) It prevents the lungs from collapsing within the thoracic cavity. C) It limits lung expansion within the thoracic cavity. D) It lubricates the movement of the thorax and lungs.

Ans: D Feedback: The visceral pleura cover the lungs; the parietal pleura line the thorax. The visceral and parietal pleura and the small amount of pleural fluid between these two membranes serve to lubricate the thorax and lungs and permit smooth motion of the lungs within the thoracic cavity with each breath. The pleura do not allow full expansion of the lungs, prevent the lungs from collapsing, or limit lung expansion within the thoracic cavity.

17. A patient has a diagnosis of multiple sclerosis. The nurse is aware that neuromuscular disorders such as multiple sclerosis may lead to a decreased vital capacity. What does vital capacity measure? A) The volume of air inhaled and exhaled with each breath B) The volume of air in the lungs after a maximal inspiration C) The maximal volume of air inhaled after normal expiration D) The maximal volume of air exhaled from the point of maximal inspiration

Ans: D Feedback: Vital capacity is measured by having the patient take in a maximal breath and exhale fully through a spirometer. Vital lung capacity is the maximal volume of air exhaled from the point of maximal inspiration, and neuromuscular disorders such as multiple sclerosis may lead to a decreased vital capacity. Tidal volume is defined as the volume of air inhaled and exhaled with each breath. The volume of air in the lungs after a maximal inspiration is the total lung capacity. Inspiratory capacity is the maximal volume of air inhaled after normal expiration.

A student nurse is preparing to care for a patient with bronchiectasis. The student nurse should recognize that this patient is likely to experience respiratory difficulties related to what pathophysiologic process? A) Intermittent episodes of acute bronchospasm B) Alveolar distention and impaired diffusion C) Dilation of bronchi and bronchioles D) Excessive gas exchange in the bronchioles

Ans: Dilation of bronchi and bronchioles Feedback: Bronchiectasis is a chronic, irreversible dilation of the bronchi and bronchioles that results from destruction of muscles and elastic connective tissue. It is not characterized by acute bronchospasm, alveolar distention, or excessive gas exchange.

An admitting nurse is assessing a patient with COPD. The nurse auscultates diminished breath sounds, which signify changes in the airway. These changes indicate to the nurse to monitor the patient for what? A) Kyphosis and clubbing of the fingers B) Dyspnea and hypoxemia C) Sepsis and pneumothorax D) Bradypnea and pursed lip breathing

Ans: Dyspnea and hypoxemia Feedback: These changes in the airway require that the nurse monitor the patient for dyspnea and hypoxemia. Kyphosis is a musculoskeletal problem. Sepsis and pneumothorax are atypical complications. Tachypnea is much more likely than bradypnea. Pursed lip breathing can relieve dyspnea.

An asthma nurse educator is working with a group of adolescent asthma patients. What intervention is most likely to prevent asthma exacerbations among these patients? A) Encouraging patients to carry a corticosteroid rescue inhaler at all times B) Educating patients about recognizing and avoiding asthma triggers C) Teaching patients to utilize alternative therapies in asthma management D) Ensuring that patients keep their immunizations up to date

Ans: Educating patients about recognizing and avoiding asthma triggers Feedback: Asthma exacerbations are best managed by early treatment and education, including the use of written action plans as part of any overall effort to educate patients about self-management techniques, especially those with moderate or severe persistent asthma or with a history of severe exacerbations. Corticosteroids are not used as rescue inhalers. Alternative therapies are not normally a high priority, though their use may be appropriate in some cases. Immunizations should be kept up to date, but this does not necessarily prevent asthma exacerbations.

A nurse is evaluating the diagnostic study data of a patient with suspected cystic fibrosis (CF). Which of the following test results is associated with a diagnosis of cystic fibrosis? A) Elevated sweat chloride concentration B) Presence of protein in the urine C) Positive phenylketonuria D) Malignancy on lung biopsy

Ans: Elevated sweat chloride concentration Feedback: Gene mutations affect transport of chloride ions, leading to CF, which is characterized by thick, viscous secretions in the lungs, pancreas, liver, intestine, and reproductive tract as well as increased salt content in sweat gland secretions.

An asthma educator is teaching a patient newly diagnosed with asthma and her family about the use of a peak flow meter. The educator should teach the patient that a peak flow meter measures what value? A) Highest airflow during a forced inspiration B) Highest airflow during a forced expiration C) Airflow during a normal inspiration D) Airflow during a normal expiration

Ans: Highest airflow during a forced expiration Feedback: Peak flow meters measure the highest airflow during a forced expiration.

A patient's severe asthma has necessitated the use of a long-acting beta2-agonist (LABA). Which of the patient's statements suggests a need for further education? A) "I know that these drugs can sometimes make my heart beat faster." B) "I've heard that this drug is particularly good at preventing asthma attacks during exercise." C) "I'll make sure to use this each time I feel an asthma attack coming on." D) "I've heard that this drug sometimes gets less effective over time."

Ans: I'll make sure to use this each time I feel an asthma attack coming on. Feedback: LABAs are not used for management of acute asthma symptoms. Tachycardia is a potential adverse effect and decreased protection against exercise-induced bronchospasm may occur with regular use.

A nurse is caring for a patient with COPD. The patient's medication regimen has been recently changed and the nurse is assessing for therapeutic effect of a new bronchodilator. What assessment parameters suggest a consequent improvement in respiratory status? Select all that apply. A) Negative sputum culture B) Increased viscosity of lung secretions C) Increased respiratory rate D) Increased expiratory flow rate E) Relief of dyspnea

Ans: Increased expiratory flow rate, Relief of dyspnea Feedback: The relief of bronchospasm is confirmed by measuring improvement in expiratory flow rates and volumes (the force of expiration, how long it takes to exhale, and the amount of air exhaled) as well as by assessing the dyspnea and making sure that it has lessened. Increased respiratory rate and viscosity of secretions would suggest a worsening of the patient's respiratory status. Bronchodilators would not have a direct result on the patient's infectious process.

A nurse has been asked to give a workshop on COPD for a local community group. The nurse emphasizes the importance of smoking cessation because smoking has what pathophysiologic effect? A) Increases the amount of mucus production B) Destabilizes hemoglobin C) Shrinks the alveoli in the lungs D) Collapses the alveoli in the lungs

Ans: Increases the amount of mucus production Feedback: Smoking irritates the goblet cells and mucous glands, causing an increased accumulation of mucus, which, in turn, produces more irritation, infection, and damage to the lung.

The case manager for a group of patients with COPD is providing health education. What is most important for the nurse to assess when providing instructions on self-management to these patients? A) Knowledge of alternative treatment modalities B) Family awareness of functional ability and activities of daily living (ADLs) C) Knowledge of the pathophysiology of the disease process D) Knowledge about self-care and their therapeutic regimen

Ans: Knowledge about self-care and their therapeutic regimen Feedback: When providing instructions about self-management, it is important for the nurse to assess the knowledge of patients and family members about self-care and the therapeutic regimen. This supersedes knowledge of alternative treatments or the pathophysiology of the disease, neither of which is absolutely necessary for patients to know. The patient's own knowledge is more important than that of the family.

A nurse is caring for a young adult patient whose medical history includes an alpha1-antitrypsin deficiency. This deficiency predisposes the patient to what health problem? A) Pulmonary edema B) Lobular emphysema C) Cystic fibrosis (CF) D) Empyema

Ans: Lobular emphysema Feedback: A host risk factor for COPD is a deficiency of alpha1-antitrypsin, an enzyme inhibitor that protects the lung parenchyma from injury. This deficiency predisposes young patients to rapid development of lobular emphysema even in the absence of smoking. This deficiency does not influence the patient's risk of pulmonary edema, CF, or empyema.

A patient is having pulmonary-function studies performed. The patient performs a spirometry test, revealing an FEV1/FVC ratio of 60%. How should the nurse interpret this assessment finding? A) Strong exercise tolerance B) Exhalation volume is normal C) Respiratory infection D) Obstructive lung disease

Ans: Obstructive lung disease Feedback: Spirometry is used to evaluate airflow obstruction, which is determined by the ratio of forced expiration volume in 1 second to forced vital capacity. Obstructive lung disease is apparent when an FEV1/FVC ratio is less than 70%.

A nurse is caring for a 6-year-old patient with cystic fibrosis. In order to enhance the child's nutritional status, what intervention should most likely be included in the plan of care? A) Pancreatic enzyme supplementation with meals B) Provision of five to six small meals per day rather than three larger meals C) Total parenteral nutrition (TPN) D) Magnesium, thiamine, and iron supplementation

Ans: Pancreatic enzyme supplementation with meals Feedback: Nearly 90% of patients with CF have pancreatic exocrine insufficiency and require oral pancreatic enzyme supplementation with meals. Frequent, small meals or TPN are not normally indicated. Vitamin supplements are required, but specific replacement of magnesium, thiamine, and iron is not typical.

A pediatric nurse practitioner is caring for a child who has just been diagnosed with asthma. The nurse has provided the parents with information that includes potential causative agents for an asthmatic reaction. What potential causative agent should the nurse describe? A) Pets B) Lack of sleep C) Psychosocial stress D) Bacteria

Ans: Pets Feedback: Common causative agents that may trigger an asthma attack are as follows: dust, dust mites, pets, soap, certain foods, molds, and pollens. Lack of sleep, stress, and bacteria are not common triggers for asthma attacks.

A patient arrives in the emergency department with an attack of acute bronchiectasis. Chest auscultation reveals the presence of copious secretions. What intervention should the nurse prioritize in this patient's care? A) Oral administration of diuretics B) Intravenous fluids to reduce the viscosity of secretions C) Postural chest drainage D) Pulmonary function testing

Ans: Postural chest drainage Feedback: Postural drainage is part of all treatment plans for bronchiectasis, because draining of the bronchiectatic areas by gravity reduces the amount of secretions and the degree of infection. Diuretics and IV fluids will not aid in the mobilization of secretions. Lung function testing may be indicated, but this assessment will not relieve the patient's symptoms.

A nurse is creating a health promotion intervention focused on chronic obstructive pulmonary disease (COPD). What should the nurse identify as a complication of COPD? A) Lung cancer B) Cystic fibrosis C) Respiratory failure D) Hemothorax

Ans: Respiratory failure Feedback: Complications of COPD include respiratory failure, pneumothorax, atelectasis, pneumonia, and pulmonary hypertension (corpulmonale). Lung cancer, cystic fibrosis, and hemothorax are not common complications.

A nurse is developing a teaching plan for a patient with COPD. What should the nurse include as the most important area of teaching? A) Avoiding extremes of heat and cold B) Setting and accepting realistic short- and long-range goals C) Adopting a lifestyle of moderate activity D) Avoiding emotional disturbances and stressful situations

Ans: Setting and accepting realistic short- and long-range goals Feedback: A major area of teaching involves setting and accepting realistic short-term and long-range goals. The other options should also be included in the teaching plan, but they are not areas that are as high a priority as setting and accepting realistic goals.

A nursing is planning the care of a patient with emphysema who will soon be discharged. What teaching should the nurse prioritize in the plan of care? A) Taking prophylactic antibiotics as ordered B) Adhering to the treatment regimen in order to cure the disease C) Avoiding airplanes, buses, and other crowded public places D) Setting realistic short-term and long-range goals

Ans: Setting realistic short-term and long-range goals Feedback: A major area of teaching involves setting and accepting realistic short-term and long-range goals. Emphysema is not considered curable and antibiotics are not used on a preventative basis. The patient does not normally need to avoid public places.

A patient with emphysema is experiencing shortness of breath. To relieve this patient's symptoms, the nurse should assist her into what position? A) Sitting upright, leaning forward slightly B) Low Fowler's, with the neck slightly hyperextended C) Prone D) Trendelenburg

Ans: Sitting upright, leaning forward slightly Feedback: The typical posture of a person with COPD is to lean forward and use the accessory muscles of respiration to breathe.

A student nurse is developing a teaching plan for an adult patient with asthma. Which teaching point should have the highest priority in the plan of care that the student is developing? A) Gradually increase levels of physical exertion. B) Change filters on heaters and air conditioners frequently. C) Take prescribed medications as scheduled. D) Avoid goose-down pillows.

Ans: Take prescribed medications as scheduled. Feedback: Although all of the measures are appropriate for a client with asthma, taking prescribed medications on time is the most important measure in preventing asthma attacks.

A nurse is admitting a new patient who has been admitted with a diagnosis of COPD exacerbation. How can the nurse best help the patient achieve the goal of maintaining effective oxygenation? A) Teach the patient strategies for promoting diaphragmatic breathing. B) Administer supplementary oxygen by simple face mask. C) Teach the patient to perform airway suctioning. D) Assist the patient in developing an appropriate exercise program.

Ans: Teach the patient strategies for promoting diaphragmatic breathing. Feedback: The breathing pattern of most people with COPD is shallow, rapid, and inefficient; the more severe the disease, the more inefficient the breathing pattern. With practice, this type of upper chest breathing can be changed to diaphragmatic breathing, which reduces the respiratory rate, increases alveolar ventilation, and sometimes helps expel as much air as possible during expiration. Suctioning is not normally necessary in patients with COPD. Supplementary oxygen is not normally delivered by simple face mask and exercise may or may not be appropriate.

A nurse is providing health education to the family of a patient with bronchiectasis. What should the nurse teach the patient's family members? A) The correct technique for chest palpation and auscultation B) Techniques for assessing the patient's fluid balance C) The technique for providing deep nasotracheal suctioning D) The correct technique for providing postural drainage

Ans: The correct technique for providing postural drainage Feedback: A focus of the care of bronchiectasis is helping patients clear pulmonary secretions; consequently, patients and families are taught to perform postural drainage. Chest palpation and auscultation and assessment of fluid balance are not prioritized over postural drainage. Nasotracheal suctioning is not normally necessary.

A clinic nurse is caring for a patient who has just been diagnosed with chronic obstructive pulmonary disease (COPD). The patient asks the nurse what he could have done to minimize the risk of contracting this disease. What would be the nurse's best answer? A) "The most important risk factor for COPD is exposure to occupational toxins." B) "The most important risk factor for COPD is inadequate exercise." C) "The most important risk factor for COPD is exposure to dust and pollen." D) "The most important risk factor for COPD is cigarette smoking."

Ans: The most important risk factor for COPD is cigarette smoking. Feedback: The most important risk factor for COPD is cigarette smoking. Lack of exercise and exposure to dust and pollen are not risk factors for COPD. Occupational risks are significant but are far exceeded by smoking.

An older adult patient has been diagnosed with COPD. What characteristic of the patient's current health status would preclude the safe and effective use of a metered-dose inhaler (MDI)? A) The patient has not yet quit smoking. B) The patient has severe arthritis in her hands. C) The patient requires both corticosteroids and beta2-agonists. D) The patient has cataracts.

Ans: The patient has severe arthritis in her hands. Feedback: Safe and effective MDI use requires the patient to be able to manipulate the device independently, which may be difficult if the patient has arthritis. Smoking does not preclude MDI use. A modest loss of vision does not preclude the use of an MDI and a patient can safely use more than one MDI.

A nurse is planning the care of a client with bronchiectasis. What goal of care should the nurse prioritize? A) The patient will successfully mobilize pulmonary secretions. B) The patient will maintain an oxygen saturation level of ≥98%. C) The patient's pulmonary blood pressure will decrease to within reference ranges. D) The patient will resume prediagnosis level of function within 72 hours.

Ans: The patient will successfully mobilize pulmonary secretions. Feedback: Nursing management focuses on alleviating symptoms and helping patients clear pulmonary secretions. Pulmonary pressures are not a central focus in the care of the patient with bronchiectasis. Rapid resumption of prediagnosis function and oxygen saturation above 98% are unrealistic goals.

A nurse is explaining to a patient with asthma what her new prescription for prednisone is used for. What would be the most accurate explanation that the nurse could give? A) To ensure long-term prevention of asthma exacerbations B) To cure any systemic infection underlying asthma attacks C) To prevent recurrent pulmonary infections D) To gain prompt control of inadequately controlled, persistent asthma

Ans: To gain prompt control of inadequately controlled, persistent asthma Feedback: Prednisone is used for a short-term (3-10 days) "burst" to gain prompt control of inadequately controlled, persistent asthma. It is not used to treat infection or to prevent exacerbations in the long term.

A nurse is providing discharge teaching for a client with COPD. When teaching the client about breathing exercises, what should the nurse include in the teaching? A) Lie supine to facilitate air entry B) Avoid pursed lip breathing C) Use diaphragmatic breathing D) Use chest breathing

Ans: Use diaphragmatic breathing Feedback: Inspiratory muscle training and breathing retraining may help improve breathing patterns in patients with COPD. Training in diaphragmatic breathing reduces the respiratory rate, increases alveolar ventilation, and, sometimes, helps expel as much air as possible during expiration. Pursed-lip breathing helps slow expiration, prevents collapse of small airways, and controls the rate and depth of respiration. Diaphragmatic breathing, not chest breathing, increases lung expansion. Supine positioning does not aid breathing.

A nurse's assessment reveals that a client with COPD may be experiencing bronchospasm. What assessment finding would suggest that the patient is experiencing bronchospasm? A) Fine or coarse crackles on auscultation B) Wheezes or diminished breath sounds on auscultation C) Reduced respiratory rate or lethargy D) Slow, deliberate respirations

Ans: Wheezes or diminished breath sounds on auscultation Feedback: Wheezing and diminished breath sounds are consistent with bronchospasm. Crackles are usually attributable to other respiratory or cardiac pathologies. Bronchospasm usually results in rapid, inefficient breathing and agitation.

A patient diagnosed with Bells palsy is being cared for on an outpatient basis. During health education, the nurse should promote which of the following actions?

Applying a protective eye shield at nigh

A nurse is caring for a patient who experiences debilitating cluster headaches. The patient should be taught to take appropriate medications at what point in the course of the onset of a new headache?

As soon as the patient senses the onset of symptoms

The nurse is reviewing the medication administration record of a female patient who possesses numerous risk factors for stroke. Which of the womans medications carries the greatest potential for reducing her risk of stroke?

Aspirin 81 mg PO o.d.

The nurse is performing a neurologic assessment of a patient whose injuries have rendered her unable to follow verbal commands. How should the nurse proceed with assessing the patients level of consciousness (LOC)?

Assess the patients eye opening and response to stimuli.

The nurse is caring for a patient whose recent health history includes an altered LOC. What should be the nurses first action when assessing this patient?

Assessing the patients verbal response

The nurse is conducting a focused neurologic assessment. When assessing the patients cranial nerve function, the nurse would include which of the following assessments?

Assessment of gag reflex

A preceptor is discussing stroke with a new nurse on the unit. The preceptor would tell the new nurse which cardiac dysrhythmia is associated with cardiogenic embolic strokes?

Atrial fibrillation

The nurse caring for a patient in ICU diagnosed with Guillain-Barr syndrome should prioritize monitoring for what potential complication?

Autonomic dysfunction

The nurse is discharging a patient home after surgery for trigeminal neuralgia. What advice should the nurse provide to this patient in order to reduce the risk of injury?

Avoid rubbing the eye on the affected side of the face.

Research has corroborated an experienced nurses observation that the incidence and prevalence of chronic conditions is increasing in the United States. What health promotion initiative most directly addresses the factor that has been shown to contribute to this increase? A) A program to link residents with primary care providers B) A community-based weight-loss program C) A stress management workshop D) A cancer screening campaign

B) A community-based weight-loss program

A 19-year-old patient with a diagnosis of Down syndrome is being admitted to your unit for the treatment of community-acquired pneumonia. When planning this patients care, the nurse recognizes that this patient's disability is categorized as what? A) A sensory disability B) A developmental disability C) An acquired disability D) An age-associated disability

B) A developmental disability

A nurse knows that patients with invisible disabilities like chronic pain often feel that their chronic conditions are more challenging to deal with than more visible disabilities. Why would they feel this way? A) Invisible disabilities create negative attitudes in the health care community. B) Despite appearances, invisible disabilities can be as disabling as visible disabilities. C) Disabilities, such as chronic pain, are apparent to the general population. D) Disabilities. Such as chronic pain, may not be curable, unlike visible disabilities.

B) Despite appearances, invisible disabilities can be as disabling as visible disabilities.

A nurse is planning the care of a patient who has been diagnosed with renal failure, which the nurse recognizes as being a chronic condition. Which of the following descriptors apply to chronic conditions? Select all that apply. A) Diseases that resolve slowly B) Diseases where complete cures are rare C) Diseases that have a short, unpredictable course D) Diseases that do not resolve spontaneously E) Diseases that have a prolonged course

B) Diseases where complete cures are rare D) Diseases that do not resolve spontaneously E) Diseases that have a prolonged course

A patient with end-stage lung cancer has been admitted to hospice care. The hospice team is meeting with the patient and her family to establish goals for care. What is likely to be a first priority in goal setting for the patient? A) Maintenance of activities of daily living B) Pain control C) Social interaction D) Promotion of spirituality

B) Pain Control

A medical-surgical nurse is teaching a patient about the health implications of her recently diagnosed type 2 diabetes. The nurse should teach the patient to be proactive with her glycemic control in order to reduce her risk of what health problem? A) Arthritis B) Renal failure C) Pancreatic cancer D) Asthma

B) Renal Failure

A 39-year-old patient with paraplegia has been admitted to the hospital for the treatment of a sacral ulcer. The nurse is aware that the patient normally lives alone in an apartment and manages his ADLs independently. Before creating the patients plan of care, how should the nurse best identify the level of assistance that the patient will require in the hospital? A) Make referrals for assessment to occupational therapy and physical therapy. B) Talk with the patient about the type and level of assistance that he desires. C) Obtain the patients previous medical record and note what was done during his most recent admission. D) Apply a standardized care plan that addresses the needs of a patient with paraplegia.

B) Talk with the patient about the type and level of assistance that he desires.

The staff development nurse is presenting a class on the importance of incorporating people-first language into daily practice as well as documentation. What is an example of the use of people-first language when giving a verbal report? A) The schizophrenic B) The patient with schizophrenia C) The schizophrenic patient D) The schizophrenic client

B) The patient with schizophrenia

An initiative has been launched in a large hospital to promote the use of people-first language in formal and informal communication. What is the significance to the patient when the nurse uses people-first language? A) The nurse knows more clearly who the patient is. B) The person is of more importance to the nurse than the disability. C) The patients disability is the defining characteristic of the patients life. D) The nurse knows that the patients disability is a curable condition.

B) The person is of more importance to the nurse than the disability.

During the care conference for a patient who has multiple chronic conditions, the case manager has alluded to the principles of the interface model of disability. What statement is most characteristic of this model? A) This patient should be free to plan his care without our interference. B) This patient can be empowered and doesn't have to be dependent. C) This patient was a very different person before the emergence of these health problems. D) This patients physiological problems are the priority over his psychosocial status.

B) This patient can be empowered and doesn't have to be dependent.

A patient with a new diagnosis of ischemic stroke is deemed to be a candidate for treatment with tissue plasminogen activator (t-PA) and has been admitted to the ICU. In addition to closely monitoring the patients cardiac and neurologic status, the nurse monitors the patient for signs of what complication?

Bleeding

To alleviate pain associated with trigeminal neuralgia, a patient is taking Tegretol (carbamazepine). What health education should the nurse provide to the patient before initiating this treatment?

Blood levels of the drug must be monitored.

A 33-year-old patient presents at the clinic with complaints of weakness, incoordination, dizziness, and loss of balance. The patient is hospitalized and diagnosed with MS. What sign or symptom, revealed during the initial assessment, is typical of MS?

Blurred vision, intention tremor, and urinary hesitancy

A patient who has recently been diagnosed with chronic heart failure is being taught by the nurse how to live successfully with her chronic condition. Her ability to meet this goal will primarily depend on her ability to do which of the following? A) Lower her expectations for quality of life and level of function. B) Access community services to eventually cure her disease. C) Adapt her lifestyle to accommodate her symptoms. D) Establish good rapport with her primary care provider.

C) Adapt her lifestyle to accommodate her symptoms.

The nurse is reviewing the importance of preventative health care with a patient who has a disability. The patient states that she will not have the money to pay for her annual gynecologic exams or mammograms due to the cost of this hospitalization. What information would be appropriate for the nurse to share with the patient? A) Limited finances are a common problem for patients with a disability. Since you were hospitalized this year, you can likely forego the gynecologic exam and mammogram. B) These are very important health preventative measures, so you will need to borrow the money to pay for the exam and mammogram. C) I'll look into federal assistance programs that provide financial assistance for health-related expenses for people with disabling conditions. D) These preventative measures should likely be tax deductible, so you should consult with your accountant and then make your appointments.

C) I'll look into federal assistance programs that provide financial assistance for health-related expenses for people with disabling conditions.

You are the case manager who oversees the multidisciplinary care of patients living with chronic conditions. Two of your patients are living with spina bifida. You recognize that the center of care for these two patients typically exists where? A) In the hospital B) In the physicians office C) In the home D) In the rehabilitation facility

C) In the home

The interface model of disability is being used to plan the care of a patient who is living with the effects of a stroke. Why should the nurse prioritize this model? A) It fosters dependency and rapport between the caregiver and the patient. B) It encourages the provision of care that is based specifically on the disability. C) It promotes interactions with patients focused on the root cause of the disability. D) It promotes the idea that patients are capable and responsible.

C) It promotes the idea that patients are capable and responsible.

The nurse is caring for a young adult male with a traumatic brain injury and severe disabilities caused by a motor vehicle accident when he was an adolescent. Where does the nurse often provide care for patients like this young adult? A) Adult day-care facilities B) Step-down units C) Medical-surgical units D) Pediatric units

C) Medical Surgical units

A patient who is recovering from a stroke expresses frustration about his care to the nurse, stating, It seems like everyone sees me as just a problem that needs fixing. This patients statement is suggestive of what model of disability? A) Biopsychosocial model B) Social model C) Rehabilitation model D) Interface model

C) Rehabilitation model

You are the nurse caring for a young mother who has a longstanding diagnosis of multiple sclerosis (MS). She was admitted to your unit with a postpartum infection 3 days ago. You are planning to discharge her home when she has finished 5 days of IV antibiotic therapy. With what information would it be most important for you to provide this patient? A) A succinct overview of postpartum infections B) How the response to infection differs in patients with multiple sclerosis C) The same information you would provide to a patient without a chronic condition D) Information on effective management of multiple sclerosis in the home setting

C) The same information you would provide to a patient without a chronic condition

You are presenting patient teaching to a 48-year-old man who was just diagnosed with type 2 diabetes. The patient has a BMI of 35 and leads a sedentary lifestyle. You give the patient information on the risk factors for his diagnosis and begin talking with him about changing behaviors around diet and exercise. You know that further patient teaching is needed when the patient tells you what? A) I need to start slow on an exercise program approved by my doctor. B) I know theres a chance I could have avoided this if Id always eaten better and exercised more. C) There is nothing that can be done anyway, because chronic diseases like diabetes cannot be prevented. D) I want to have a plan in place before I start making a lot of changes to my lifestyle.

C) There is nothing that can be done anyway, because chronic diseases like diabetes cannot be prevented.

A man and woman are in their early eighties and have provided constant care for their 44-year-old son who has Down syndrome. When planning this family's care, the nurse should be aware that the parents most likely have what concerns around what question? A) What could we have done better for our son? B) Why was our son born with Down syndrome while our other children are healthy? C) Who will care for our son once were unable? D) Will we experience the effects of developmental disabilities late in life?

C) Who will care for our son once were unable?

A patient diagnosed with a cerebral aneurysm reports a severe headache to the nurse. What action is a priority for the nurse?

Call the physician immediately.

A patient who just suffered a suspected ischemic stroke is brought to the ED by ambulance. On what should the nurses primary assessment focus?

Cardiac and respiratory status

A patient exhibiting an uncoordinated gait has presented at the clinic. Which of the following is the most plausible cause of this patients health problem?

Cerebellar dysfunction

A nurse is assessing reflexes in a patient with hyperactive reflexes. When the patients foot is abruptly dorsiflexed, it continues to beat two to three times before settling into a resting position. How would the nurse document this finding?

Clonus

A patient is admitted through the ED with suspected St. Louis encephalitis. The unique clinical feature of St. Louis encephalitis will make what nursing action a priority?

Close monitoring of fluid balance

While completing a health history on a patient who has recently experienced a seizure, the nurse would assess for what characteristic associated with the postictal state?

Confusion

A patient is being given a medication that stimulates her parasympathetic system. Following administration of this medication, the nurse should anticipate what effect?

Constricted pupils

A major cause of health-related problems is the increase in the incidence of chronic conditions. This is the case not only in developed countries like the United States but also in developing countries. What factor has contributed to the increased incidence of chronic diseases in developing countries? A) Developing countries are experiencing an increase in average life span. B) Increasing amounts of health research are taking place in developing countries. C) Developing countries lack the health infrastructure to manage illness. D) Developing countries are simultaneously coping with emerging infectious diseases.

D) Developing countries are simultaneously coping with emerging infectious diseases.

A 37-year-old woman with multiple sclerosis is married and has three children. The nurse has worked extensively with the woman and her family to plan appropriate care. What is the nurses most important role with this patient? A) Ensure the patient adheres to all treatments B) Provide the patient with advice on alternative treatment options C) Provide a detailed plan of activities of daily living (ADLs) for the patient D) Help the patient develop strategies to implement treatment regimens

D) Help the patient develop strategies to implement treatment regimens

You are caring for a young woman who has Down syndrome and who has just been diagnosed with type 2 diabetes. What consideration should you prioritize when planning this patients nursing care? A) How her new diagnosis affects her health attitudes B) How her diabetes affects the course of her Down syndrome C) How her chromosomal disorder affects her glucose metabolism D) How her developmental disability influences her health management

D) How her developmental disability influences her health management

A patient has recently been diagnosed with type 2 diabetes. The patient is clinically obese and has a sedentary lifestyle. How can the nurse best begin to help the patient increase his activity level? A) Set up appointment times at a local fitness center for the patient to attend. B) Have a family member ensure the patient follows a suggested exercise plan. C) Construct an exercise program and have the patient follow it. D) Identify barriers with the patient that inhibit his lifestyle change.

D) Identify barriers with the patient that inhibit his lifestyle change.

A nurse is aware that the number of people in the United States who are living with disabilities is expected to continue increasing. What is considered to be one of the factors contributing to this increase? A) The decrease in the number of people with early-onset disabilities B) The increased inability to cure chronic disorders C) Changes in infection patterns resulting from antibiotic resistance D) Increased survival rates among people who experience trauma

D) Increased survival rates among people who experience trauma

During their prime employable years between ages 21 and 64, 77% of those with a non severe disability are employed. What has research shown about this employed population? A) Their salaries are commensurate with their experience. B) They enjoy their jobs more than people who do not have disabilities. C) Employment rates are higher among people with a disability than those without. D) People with disabilities earn less money than people without disabilities.

D) People with disabilities earn less money than people without disabilities.

The nurse is caring for a patient diagnosed with cancer of the liver who has chosen to remain in his home as long as he is able. The nurse reviews the care plan for the patient and notes that it focuses on palliative measures. The nurse also notes that over the last 3 weeks, the patient's condition has continued to deteriorate. What is the nurse's best response to this clinical information? A) Recognize that death will most likely occur in the next week. B) Recognize that the patient is in the trajectory phase of chronic illness and should be kept pain-free. C) Recognize that the patient is in the downward phase of chronic illness and should be reassessed. D) Recognize that the patient should immediately be admitted into the hospital.

D) Recognize that the patient should immediately be admitted into the hospital.

A patient who undergoes hemodialysis three times weekly is on a fluid restriction of 1000 mL/day. The nurse sees the patient drinking a 355-mL (12 ounce) soft drink after the patient has already reached the maximum intake of fluid for the day. What action should the nurse take? A) Take the soft drink away from the patient and inform the dialysis nurse to remove extra fluid from the patient during the next dialysis treatment B) Document the patients behavior as noncompliant and notify the physician C) Further restrict the patients fluid for the following day and communicate this information to the charge nurse D) Reinforce the importance of the fluid restriction and document the teaching and the intake of extra fluid

D) Reinforce the importance of fluid restriction and document the teaching and the intake of extra fluid

A community health nurse has drafted a program that will address the health promotion needs of members of the community who live with one or more disabilities. Which of the following areas of health promotion education is known to be neglected among adults with disabilities? A) Blood pressure screening B) Diabetes testing C) Nutrition D) Sexual health

D) Sexual Health

You have admitted a new patient to your unit with a diagnosis of stage IV breast cancer. This woman has a comorbidity of myasthenia gravis. While you are doing the initial assessment, the patient tells you that she felt the lump in her breast about 9 months ago. You ask the patient why she did not see her health care provider when she first found the lump in her breast. What would be a factor that is known to influence the patient in seeking health care services? A) Lack of insight due to the success of self-managing a chronic condition B) Lack of knowledge about treatment options C) Overly sensitive patient reactions to health care services D) Unfavorable interactions with health care providers

D) Unfavorable interactions with health care providers

The nurse is planning the care of a patient with Parkinsons disease. The nurse should be aware that treatment will focus on what pathophysiological phenomenon?

Decreased availability of dopamine

A patient diagnosed with MS has been admitted to the medical unit for treatment of an MS exacerbation. Included in the admission orders is baclofen (Lioresal). What should the nurse identify as an expected outcome of this treatment?

Decreased muscle spasms in the lower extremities

The physician has ordered a somatosensory evoked responses (SERs) test for a patient for whom the nurse is caring. The nurse is justified in suspecting that this patient may have a history of what type of neurologic disorder?

Demyelinating disease

The nurse is discharging home a patient who suffered a stroke. He has a flaccid right arm and leg and is experiencing problems with urinary incontinence. The nurse makes a referral to a home health nurse because of an awareness of what common patient response to a change in body image?

Depression

A patient diagnosed with myasthenia gravis has been hospitalized to receive plasmapheresis for a myasthenic exacerbation. The nurse knows that the course of treatment for plasmapheresis in a patient with myasthenia gravis is what?

Determined by the patients response

The nurse is caring for a patient with a brain tumor. What drug would the nurse expect to be ordered to reduce the edema surrounding the tumor?

Dexamethasone

What should the nurse suspect when hourly assessment of urine output on a patient postcraniotomy exhibits a urine output from a catheter of 1,500 mL for two consecutive hours?

Diabetes insipidus

A middle-aged woman has sought care from her primary care provider and undergone diagnostic testing that has resulted in a diagnosis of MS. What sign or symptom is most likely to have prompted the woman to seek care?

Difficulty in coordination

A patient is being admitted to the neurologic ICU with suspected herpes simplex virus encephalitis. What nursing action best addresses the patients complaints of headache?

Dimming the lights and reducing stimulation

The nurse has created a plan of care for a patient who is at risk for increased ICP. The patients care plan should specify monitoring for what early sign of increased ICP?

Disorientation and restlessness

A nursing student is writing a care plan for a newly admitted patient who has been diagnosed with a stroke. What major nursing diagnosis should most likely be included in the patients plan of care?

Disturbed sensory perception

During the performance of the Romberg test, the nurse observes that the patient sways slightly. What is the nurses most appropriate action?

Document successful completion of the assessment.

A patient with suspected Creutzfeldt-Jakob disease (CJD) is being admitted to the unit. The nurse would expect what diagnostic test to be ordered for this patient?

EEG

A trauma patient in the ICU has been declared brain dead. What diagnostic test is used in making the determination of brain death?

Electroencephalography (EEG)

When caring for a patient who has had a stroke, a priority is reduction of ICP. What patient position is most consistent with this goal?

Elevation of the head of the bed

The nurse is caring for a patient who is hospitalized with an exacerbation of MS. To ensure the patients safety, what nursing action should be performed?

Ensure that suction apparatus is set up at the bedside.

A patient with MS has been admitted to the hospital following an acute exacerbation. When planning the patients care, the nurse addresses the need to enhance the patients bladder control. What aspect of nursing care is most likely to meet this goal?

Establish a timed voiding schedule.

A patient is brought by ambulance to the ED after suffering what the family thinks is a stroke. The nurse caring for this patient is aware that an absolute contraindication for thrombolytic therapy is what?

Evidence of hemorrhagic stroke

What should be included in the patients care plan when establishing an exercise program for a patient affected by a stroke?

Exercise the affected extremities passively four or five times a day.

The nurse is assessing a patient with a suspected stroke. What assessment finding is most suggestive of a stroke?

Facial droop

The nurse is developing a plan of care for a patient newly diagnosed with Bells palsy. The nurses plan of care should address what characteristic manifestation of this disease?

Facial paralysis

When caring for a patient with increased ICP the nurse knows the importance of monitoring for possible secondary complications, including syndrome of inappropriate antidiuretic hormone (SIADH). What nursing interventions would the nurse most likely initiate if the patient developed SIADH?

Fluid restriction

When preparing to discharge a patient home, the nurse has met with the family and warned them that the patient may exhibit unexpected emotional responses. The nurse should teach the family that these responses are typically a result of what cause?

Frustration around changes in function and communication

A patient is admitted to the medical unit with an exacerbation of multiple sclerosis. When assessing this patient, the nurse has the patient stick out her tongue and move it back and forth. What is the nurse assessing?

Function of the hypoglossal nerve

A patient has experienced a seizure in which she became rigid and then experienced alternating muscle relaxation and contraction. What type of seizure does the nurse recognize?

Generalized seizure

A patient exhibiting an altered level of consciousness (LOC) due to blunt-force trauma to the head is admitted to the ED. The physician determines the patients injury is causing increased intracranial pressure (ICP). The nurse should gauge the patients LOC on the results of what diagnostic tool?

Glasgow Coma Scale

The neurologic nurse is testing the function of a patients cerebellum and basal ganglia. What action will most accurately test these structures?

Guide the patient through the performance of rapid, alternating movements.

The nurse is teaching a patient with Guillain-Barr syndrome about the disease. The patient asks how he can ever recover if demyelination of his nerves is occurring. What would be the nurses best response?

Guillain-Barr spares the Schwann cell, which allows for remyelination in the recovery phase of the disease.

A patient has recently begun mobilizing during the recovery from an ischemic stroke. To protect the patients safety during mobilization, the nurse should perform what action?

Have a colleague follow the patient closely with a wheelchair.

A nurse is caring for a patient diagnosed with Mnires disease. While completing a neurologic examination on the patient, the nurse assesses cranial nerve VIII. The nurse would be correct in identifying the function of this nerve as what?

Hearing and equilibrium

A 69-year-old patient is brought to the ED by ambulance because a family member found him lying on the floor disoriented and lethargic. The physician suspects bacterial meningitis and admits the patient to the ICU. The nurse knows that risk factors for an unfavorable outcome include what? Select all that apply.

Heart rate greater than 120 bpm Older age Low Glasgow Coma Scale

A gerontologic nurse educator is providing practice guidelines to unlicensed care providers. Because reaction to painful stimuli is sometimes blunted in older adults, what must be used with caution?

Hot or cold packs

A patient diagnosed with a hemorrhagic stroke has been admitted to the neurologic ICU. The nurse knows that teaching for the patient and family needs to begin as soon as the patient is settled on the unit and will continue until the patient is discharged. What will family education need to include?

How to correctly modify the home environment

A patient in the OR goes into malignant hyperthermia due to an abnormal reaction to the anesthetic. The nurse knows that the area of the brain that regulates body temperature is which of the following?

Hypothalamus

A nurse is planning the care of a 28-year-old woman hospitalized with a diagnosis of myasthenia gravis. What approach would be most appropriate for the care and scheduling of diagnostic procedures for this patient?

In the morning, with frequent rest periods

A patient is having a fight or flight response after receiving bad news about his prognosis. What affect will this have on the patients sympathetic nervous system?

Increase in the secretion of sweat

The nurse has admitted a new patient to the unit. One of the patients admitting orders is for an adrenergic medication. The nurse knows that this medication will have what effect on the circulatory system?

Increased heart rate

You are the clinic nurse caring for a patient with a recent diagnosis of myasthenia gravis. The patient has begun treatment with pyridostigmine bromide (Mestinon). What change in status would most clearly suggest a therapeutic benefit of this medication?

Increased muscle strength

The nurse is caring for a patient who sustained a moderate head injury following a bicycle accident. The nurses most recent assessment reveals that the patients respiratory effort has increased. What is the nurses most appropriate response?

Inform the care team and assess for further signs of possible increased ICP.

A 35-year-old woman is diagnosed with a peripheral neuropathy. When making her plan of care, the nurse knows to include what in patient teaching? Select all that apply.

Inspect the lower extremities for skin breakdown. Footwear needs to be accurately sized. Assistive devices may be needed to reduce the risk of falls.

The nurse is creating a plan of care for a patient who has a recent diagnosis of MS. Which of the following should the nurse include in the patients care plan?

Instruct the patient on daily muscle stretching.

The nurse is caring for a patient who is in status epilepticus. What medication does the nurse know may be given to halt the seizure immediately?

Intravenous diazepam (Valium)

During the examination of an unconscious patient, the nurse observes that the patients pupils are fixed and dilated. What is the most plausible clinical significance of the nurses finding?

It indicates an injury at the midbrain level.

A patient with a documented history of seizure disorder experiences a generalized seizure. What nursing action is most appropriate?

Loosen the patients restrictive clothing.

A patient has a poor prognosis after being involved in a motor vehicle accident resulting in a head injury. As the patients ICP increases and condition worsens, the nurse knows to assess for indications of approaching death. These indications include which of the following?

Loss of brain stem reflexes

The nurse is caring for a patient with an upper motor neuron lesion. What clinical manifestations should the nurse anticipate when planning the patients neurologic assessment?

Loss of voluntary control of movement

A patient is scheduled for a myelogram and the nurse explains to the patient that this is an invasive procedure, which assesses for any lesions in the spinal cord. The nurse should explain that the preparation is similar to which of the following neurologic tests?

Lumbar puncture

The nurse is participating in the care of a patient with increased ICP. What diagnostic test is contraindicated in this patients treatment?

Lumbar puncture

The nurse is working with a patient who is newly diagnosed with MS. What basic information should the nurse provide to the patient?

MS is a progressive demyelinating disease of the nervous system.

A nurse is caring for a patient diagnosed with a hemorrhagic stroke. When creating this patients plan of care, what goal should be prioritized?

Maintain and improve cerebral tissue perfusion.

The neurologic ICU nurse is admitting a patient following a craniotomy using the supratentorial approach. How should the nurse best position the patient?

Maintain head of bed (HOB) elevated at 30 to 45 degrees.

A patient has been admitted to the ICU after being recently diagnosed with an aneurysm and the patients admission orders include specific aneurysm precautions. What nursing action will the nurse incorporate into the patients plan of care?

Maintain the patient on complete bed rest.

The nurse is providing care for a patient who is unconscious. What nursing intervention takes highest priority?

Maintaining a patent airway

A patient is being admitted to the neurologic ICU following an acute head injury that has resulted in cerebral edema. When planning this patients care, the nurse would expect to administer what priority medication?

Mannitol (Osmitrol)

A 73-year-old man comes to the clinic complaining of weakness and loss of sensation in his feet and legs. Assessment of the patient shows decreased reflexes bilaterally. Why would it be a challenge to diagnose a peripheral neuropathy in this patient?

Many symptoms can be the result of normal aging process.

A patient with increased ICP has a ventriculostomy for monitoring ICP. The nurses most recent assessment reveals that the patient is now exhibiting nuchal rigidity and photophobia. The nurse would be correct in suspecting the presence of what complication?

Meningitis

The nurse is caring for a patient who is postoperative following a craniotomy. When writing the plan of care, the nurse identifies a diagnosis of deficient fluid volume related to fluid restriction and osmotic diuretic use. What would be an appropriate intervention for this diagnosis?

Monitor serum electrolytes.

The critical care nurse is caring for 25-year-old man admitted to the ICU with a brain abscess. What is a priority nursing responsibility in the care of this patient?

Monitoring neurologic status closely

In the course of a focused neurologic assessment, the nurse is palpating the patients major muscle groups at rest and during passive movement. Data gleaned from this assessment will allow the nurse to describe which of the following aspects of neurologic function?

Muscle tone

A patient with possible bacterial meningitis is admitted to the ICU. What assessment finding would the nurse expect for a patient with this diagnosis?

Neck flexion produces flexion of knees and hips

The nurse is caring for a patient with increased intracranial pressure (ICP). The patient has a nursing diagnosis of ineffective cerebral tissue perfusion. What would be an expected outcome that the nurse would document for this diagnosis?

Obeys commands with appropriate motor responses.

When caring for a patient with an altered level of consciousness, the nurse is preparing to test cranial nerve VII. What assessment technique would the nurse use to elicit a response from cranial nerve VII?

Observe for facial movement symmetry, such as a smile.

A patient is brought to the ER following a motor vehicle accident in which he sustained head trauma. Preliminary assessment reveals a vision deficit in the patients left eye. The nurse should associate this abnormal finding with trauma to which of the following cerebral lobes?

Occipital

The nurse is doing an initial assessment on a patient newly admitted to the unit with a diagnosis of cerebrovascular accident (CVA). The patient has difficulty copying a figure that the nurse has drawn and is diagnosed with visual-receptive aphasia. What brain region is primarily involved in this deficit?

Parietal-occipital area

The nurse is caring for a patient in the ICU who has a brain stem herniation and who is exhibiting an altered level of consciousness. Monitoring reveals that the patients mean arterial pressure (MAP) is 60 mm Hg with an intracranial pressure (ICP) reading of 5 mm Hg. What is the nurses most appropriate action?

Participate in interventions to increase cerebral perfusion pressure.

The nurse is admitting a patient to the unit who is diagnosed with a lower motor neuron lesion. What entry in the patients electronic record is most consistent with this diagnosis?

Patient demonstrates an absence of deep tendon reflexes.

A patient presents at the clinic complaining of pain and weakness in her hands. On assessment, the nurse notes diminished reflexes in the upper extremities bilaterally and bilateral loss of sensation. The nurse knows that these findings are indicative of what?

Peripheral nerve disorder

The nurse is caring for a patient who has undergone supratentorial removal of a pituitary mass. What medication would the nurse expect to administer prophylactically to prevent seizures in this patient?

Phenytoin

What term is used to describe the fibrous connective tissue that hugs the brain closely and extends into every fold of the brains surface?

Pia mater

A patient is recovering from intracranial surgery that was performed using the transsphenoidal approach. The nurse should be aware that the patient may have required surgery on what neurologic structure?

Pituitary gland

A patient has had an ischemic stroke and has been admitted to the medical unit. What action should the nurse perform to best prevent joint deformities?

Place a pillow in the axilla when there is limited external rotation.

A hospital patient has experienced a seizure. In the immediate recovery period, what action best protects the patients safety?

Place the patient in a side-lying position.

A female patient is diagnosed with a right-sided stroke. The patient is now experiencing hemianopsia. How might the nurse help the patient manage her potential sensory and perceptional difficulties?

Place the patients extremities where she can see them.

The patient in the ED has just had a diagnostic lumbar puncture. To reduce the incidence of a post-lumbar puncture headache, what is the nurses most appropriate action?

Position the patient prone.

A patient with MS has developed dysphagia as a result of cranial nerve dysfunction. What nursing action should the nurse consequently perform?

Position the patient upright during feeding.

A patient with lower back pain is scheduled for myelography using metrizamide (a water-soluble contrast dye). After the test, the nurse should prioritize what action?

Positioning the patient with the head of the bed elevated 45 degrees

After a major ischemic stroke, a possible complication is cerebral edema. Nursing care during the immediate recovery period from an ischemic stroke should include which of the following?

Positioning to avoid hypoxia

A male patient presents to the clinic complaining of a headache. The nurse notes that the patient is guarding his neck and tells the nurse that he has stiffness in the neck area. The nurse suspects the patient may have meningitis. What is another well-recognized sign of this infection?

Positive Kernigs sign

The nurse is caring for a 77-year-old woman with MS. She states that she is very concerned about the progress of her disease and what the future holds. The nurse should know that elderly patients with MS are known to be particularly concerned about what variables? Select all that apply.

Possible nursing home placement Increasing disability Becoming a burden on the family

After a subarachnoid hemorrhage, the patients laboratory results indicate a serum sodium level of less than 126 mEq/L. What is the nurses most appropriate action?

Prepare to administer 3% NaCl by IV as ordered.

A patient with Guillain-Barr syndrome has experienced a sharp decline in vital capacity. What is the nurses most appropriate action?

Prepare to assist with intubation.

A patient with diabetes presents to the clinic and is diagnosed with a mononeuropathy. This patients nursing care should involve which of the following?

Protection of the affected limb from injury

The patient has been diagnosed with aphasia after suffering a stroke. What can the nurse do to best make the patients atmosphere more conducive to communication?

Provide a board of commonly used needs and phrases.

The critical care nurse is admitting a patient in myasthenic crisis to the ICU. The nurse should prioritize what nursing action in the immediate care of this patient?

Providing ventilatory assistance

A patient is postoperative day 1 following intracranial surgery. The nurses assessment reveals that the patients LOC is slightly decreased compared with the day of surgery. What is the nurses best response to this assessment finding?

Recognize that this may represent the peak of post-surgical cerebral edema.

A gerontologic nurse planning the neurologic assessment of an older adult is considering normal, age-related changes. Of what phenomenon should the nurse be aware?

Reduction in cerebral blood flow

A patient scheduled for magnetic resonance imaging (MRI) has arrived at the radiology department. The nurse who prepares the patient for the MRI should prioritize which of the following actions?

Removing all metal-containing objects

A nurse in the ICU is providing care for a patient who has been admitted with a hemorrhagic stroke. The nurse is performing frequent neurologic assessments and observes that the patient is becoming progressively more drowsy over the course of the day. What is the nurses best response to this assessment finding?

Report this to the physician as a possible sign of clinical deterioration.

The nurse is caring for a patient with multiple sclerosis (MS). The patient tells the nurse the hardest thing to deal with is the fatigue. When teaching the patient how to reduce fatigue, what action should the nurse suggest?

Resting in an air-conditioned room whenever possible

A family member brings the patient to the clinic for a follow-up visit after a stroke. The family member asks the nurse what he can do to decrease his chance of having another stroke. What would be the nurses best answer?

Stop smoking as soon as possible.

A nurse is admitting a patient with a severe migraine headache and a history of acute coronary syndrome. What migraine medication would the nurse question for this patient?

Sumatriptan succinate (Imitrex)

Following diagnostic testing, a patient has been admitted to the ICU and placed on cerebral aneurysm precautions. What nursing action should be included in patients plan of care?

Supervise the patients activities of daily living closely.

A trauma patient was admitted to the ICU with a brain injury. The patient had a change in level of consciousness, increased vital signs, and became diaphoretic and agitated. The nurse should recognize which of the following syndromes as the most plausible cause of these symptoms?

Sympathetic storm

The nurse is preparing health education for a patient who is being discharged after hospitalization for a hemorrhagic stroke. What content should the nurse include in this education?

Take antihypertensive medication as ordered.

The nurse is caring for a patient recovering from an ischemic stroke. What intervention best addresses a potential complication after an ischemic stroke?

Teaching the patient to perform deep breathing and coughing exercises

The nursing students are learning how to assess function of cranial nerve VIII. To assess the function of cranial nerve VIII the students would be correct in completing which of the following assessment techniques?

Test for air and bone conduction (Rinne test).

A rehabilitation nurse caring for a patient who has had a stroke is approached by the patients family and asked why the patient has to do so much for herself when she is obviously struggling. What would be the nurses best answer?

The focus on care in a rehabilitation facility is to help the patient to resume as much self-care as possible.

A patient is currently being stimulated by the parasympathetic nervous system. What effect will this nervous stimulation have on the patients bladder?

The parasympathetic nervous system makes the bladder contract.

Following a traumatic brain injury, a patient has been in a coma for several days. Which of the following statements is true of this patients current LOC?

The patient may occasionally make nonpurposeful movements.

A 48-year-old patient has been diagnosed with trigeminal neuralgia following recent episodes of unilateral face pain. The nurse should recognize what implication of this diagnosis?

The patient needs to be assessed for MS.

A patient who suffered an ischemic stroke now has disturbed sensory perception. What principle should guide the nurses care of this patient?

The patient should be approached on the side where visual perception is intact.

A patient with metastatic cancer has developed trigeminal neuralgia and is taking carbamazepine (Tegretol) for pain relief. What principle applies to the administration of this medication?

The patient should be monitored for bone marrow depression.

The nurse is caring for a patient diagnosed with an ischemic stroke and knows that effective positioning of the patient is important. Which of the following should be integrated into the patients plan of care?

The patient should be placed in a prone position for 15 to 30 minutes several times a day.

A patient recovering from a stroke has severe shoulder pain from subluxation of the shoulder and is being cared for on the unit. To prevent further injury and pain, the nurse caring for this patient is aware of what principle of care?

The patient should be taught to interlace fingers, place palms together, and slowly bring scapulae forward to avoid excessive force to shoulder.

A patient who has experienced an ischemic stroke has been admitted to the medical unit. The patients family in adamant that she remain on bed rest to hasten her recovery and to conserve energy. What principle of care should inform the nurses response to the family?

The patient should mobilize as soon as she is physically able.

A nurse is collaborating with the interdisciplinary team to help manage a patients recurrent headaches. What aspect of the patients health history should the nurse identify as a potential contributor to the patients headaches?

The patient takes vasodilators for the treatment of angina.

The nurse is providing care for a patient who is withdrawing from heavy alcohol use. The nurse and other members of the care team are present at the bedside when the patient has a seizure. In preparation for documenting this clinical event, the nurse should note which of the following?

The patients activities immediately prior to the seizure.

A patient for whom the nurse is caring has positron emission tomography (PET) scheduled. In preparation, what should the nurse explain to the patient?

The test may result in dizziness or lightheadedness.

A 72-year-old man has been brought to his primary care provider by his daughter, who claims that he has been experiencing uncharacteristic lapses in memory. What principle should underlie the nurses assessment and management of this patient?

Thorough assessment is necessary because changes in cognition are always considered to be pathologic.

A patient diagnosed with transient ischemic attacks (TIAs) is scheduled for a carotid endarterectomy. The nurse explains that this procedure will be done for what purpose?

To remove atherosclerotic plaques blocking cerebral flow

The nurse is developing a plan of care for a patient with Guillain-Barr syndrome. Which of the following interventions should the nurse prioritize for this patient?

Using the incentive spirometer as prescribed

A patient has developed diabetes insipidus after having increased ICP following head trauma. What nursing assessment best addresses this complication?

Vigilant monitoring of fluid balance

The nurse caring for a patient diagnosed with Guillain-Barr syndrome is planning care with regard to the clinical manifestations associated this syndrome. The nurses communication with the patient should reflect the possibility of what sign or symptom of the disease?

Vocal paralysis

The nurse is planning discharge education for a patient with trigeminal neuralgia. The nurse knows to include information about factors that precipitate an attack. What would the nurse be correct in teaching the patient to avoid?

Washing his face

A patient had a lumbar puncture performed at the outpatient clinic and the nurse has phoned the patient and family that evening. What does this phone call enable the nurse to determine?

Whether the patient has had any complications of the test

A patient diagnosed with Bells palsy is having decreased sensitivity to touch of the involved nerve. What should the nurse recommend to prevent atrophy of the muscles?

Whistling

The nurse is performing stroke risk screenings at a hospital open house. The nurse has identified four patients who might be at risk for a stroke. Which patient is likely at the highest risk for a hemorrhagic stroke?

White male, age 60, with history of uncontrolled hypertension

A patient is scheduled for CT scanning of the head because of a recent onset of neurologic deficits. What should the nurse tell the patient in preparation for this test?

You will need to lie still throughout the procedure.


Conjuntos de estudio relacionados

Prep U- Ch. 24: Asepsis and Infection Control

View Set

Principles of Microeconomics FGCU CH.5

View Set

Streetcar named desire- light quotes

View Set

Physics Practice Test - Gravity and Tides

View Set